不等式への招待 第3章

このエントリーをはてなブックマークに追加
1132人目の素数さん
ある人は蝶を集め、ある人は切手を収集し、ある人は不等式を集める…
          ___          ----- 参考文献〔3〕 P.65 -----
    |┃三 ./  ≧ \   
    |┃   |::::  \ ./ | 
    |┃ ≡|::::: (● (● |  不等式と聞ゐちゃぁ
____.|ミ\_ヽ::::... .ワ......ノ     黙っちゃゐられねゑ…
    |┃=__    \           ハァハァ
    |┃ ≡ )  人 \ ガラッ

過去スレ
・不等式スレッド (Part1)  http://science3.2ch.net/test/read.cgi/math/1072510082/
・不等式への招待 第2章 http://science6.2ch.net/test/read.cgi/math/1105911616/

過去スレのミラー置き場:http://briefcase.yahoo.co.jp/bc/loveinequality/

まとめWiki http://wiki.livedoor.jp/loveinequality/

姉妹サイト(?)
Yahoo! 掲示板 「出題 不等式」 http://messages.yahoo.co.jp/bbs?.mm=GN&action=l&board=1835554&tid=bdpbja1jiteybc0a1k&sid=1835554&mid=10000
2132人目の素数さん:2007/05/13(日) 05:02:00
不等式の本
[1] 不等式,ハーディ・リトルウッド・ポリヤ,シュプリンガー,2003年
   http://amazon.co.jp/o/ASIN/4431710566
[2] 不等式,大関信雄・青木雅計,槇書店,1967年(絶版)
[3] 不等式への招待,大関信雄・大関清太,近代科学社,1987年(絶版)
[4] 不等式入門,渡部隆一,森北出版,2005年
   http://amazon.co.jp/o/ASIN/4627010494
[5] 不等式の工学への応用、海津聰、森北出版,2004年
   http://amazon.co.jp/o/ASIN/4627075812
[6] 不等式(モノグラフ4),染取弘,科学新興新社,1990年
   http://amazon.co.jp/o/ASIN/4894281740
[7] 数理科学 No.386 特集「現代の不等式」,サイエンス社,1995年8月号(絶版)
[8] 数学トレッキングツアー第3章「相加平均≧相乗平均」,東京理科大学数学教育研究所,教育出版,2006年
   http://amazon.co.jp/o/ASIN/4316801988
[9] 数学オリンピック事典,数学オリンピック財団,朝倉書店、2001年
   http://amazon.co.jp/o/ASIN/4254110871
[10] The Cauchy-Schwarz Master Class: An Introduction to the Art of Mathematical Inequalities,J. M. Steele,Cambridge Univ. Pr.,2004年
   http://amazon.co.jp/o/ASIN/052154677X
3132人目の素数さん:2007/05/13(日) 05:03:00
不等式の埋蔵地
[1] RGMIA http://rgmia.vu.edu.au/
[2] Crux Mathematicorum Synopses http://www.journals.cms.math.ca/CRUX/synopses/
[3] Maths problems http://www.kalva.demon.co.uk/
[4] Mathematical Inequalities & Applications http://www.ele-math.com/
[5] American Mathematical Monthly http://www.maa.org/pubs/monthly.html
[6] Problems in the points contest of KöMaL http://www.komal.hu/verseny/feladatok.e.shtml
[7] IMO リンク集 http://imo.math.ca/
[9] Mathematical Olympiads Correspondence Program http://www.cms.math.ca/Competitions/MOCP/
[10] Mathematical Excalibur http://www.math.ust.hk/excalibur/
[11] MathLinks Contest http://www.mathlinks.ro/Forum/contest.html
[12] MATH PROBLEM SOLVING WEB PAGE http://www.math.northwestern.edu/~mlerma/problem_solving/#proposed_problems (要自動登録)
[13] Wolfram MathWorld http://mathworld.wolfram.com/

海外不等式ヲタの生息地
[1] Journal of Inequalities in Pure and Applied Mathematics http://jipam.vu.edu.au/
[2] MIA Journal http://www.mia-journal.com/
[3] MathLinks Math Forum http://www.mathlinks.ro/Forum/forum-55.html
4132人目の素数さん:2007/05/13(日) 05:38:57
         __,,......,,,,___
        ,.'7'::::::::::::::::!:::::`ヽ.
       /::::!:::::::::::::::::::!:::::::::::::i
      ,.!:::::i:::::::o:::::::_」:::_;;:::: ) ( 、
     ./`.7-i-r‐ r‐ 'i_! !ハ ⌒ ヽ.
     i  !/!,_,ハ_ハ,.-' -‐'‐ i__! i.  ',
     ! i  !.´ __、    ',.-- 、 i i i   !
     `'7ヽ!.'´ `      "ノ / i イ
      !. ハ"    '___  くン 、/'´      乙でございます
       ヽ,ヘ.>..  ヾ ..ノ ,.イ/
          .`>, -=´_,.!-、
  r-、       _,く`' ーrr-'":::::〈ヽ、.__
,,..-ヽ;:`ヽ.  ,.イ´::::::>-‐-<-‐'":::::::/ `ヽ.
-‐::::::i::::::::i / !;:::::::! i´ ̄`i i::::::::::::;::!   i
   _  / ____!___ヽヽ| ヽヽ ── .____|__
5132人目の素数さん:2007/05/13(日) 09:34:59
Cinco!!!!!!!!!!!!!!!!!!!!!!!!!!!!!!!!!!!!!!!!!!!!!!!!!!!!!!!!!!!!!!!!!!!!!!!!!!!!!!!!!!!!!!!!!!!!!!!!!!!!!
6132人目の素数さん:2007/05/13(日) 09:38:09
sex
7132人目の素数さん:2007/05/19(土) 19:39:52
>>992
とっとと落せ?
8132人目の素数さん:2007/05/24(木) 12:22:16
任意の正の実数x、y、zに対して次の不等式が成立する実数wの最大値を求めよ。

√(x/(y+z))+√(y/(z+x))+√(z/(x+y))>w
9132人目の素数さん:2007/05/24(木) 21:13:03
x=1≦y≦z としてよい
y+z=k(≧2)とすると、kが一定ならy=1の時が最小値f(k)をとり、
f(k)は単調減少で、f(k)→2 (k→∞)
よってw=2
10132人目の素数さん:2007/05/29(火) 14:10:59
nを2以上の整数とするとき
((n+1)^(1+(1/n))-n^(1+(1/n)))/((n+1)^(n+1)-n^(n+1))^n)^(1/n)>1/n
を示せ
11132人目の素数さん:2007/05/29(火) 14:36:46
>>10
ちんスレの人?
答えが必要な訳じゃないんだよね?
12132人目の素数さん:2007/05/29(火) 23:08:13
>>10
帰納法でいけるか
13132人目の素数さん:2007/05/30(水) 04:10:25
正の数 x、y が x+y+xy=1 をみたすとき、1/x + 1/y + 1/(x+y) のとりうる値の範囲を求めよ。

( ゚∀゚) テヘッ
14132人目の素数さん:2007/05/30(水) 05:16:12
発掘しますた!
http://www.math.northwestern.edu/~mlerma/problem_solving/old_problems/old_problems.pdf

19 はどうやるんでしょう?
27 は三角関数ヲタ向け?
15132人目の素数さん:2007/05/30(水) 05:27:16
C844、M1764、C846、C847、M1769、C851、C854、C855 に (;´Д`)'`ァ'`ァ
http://www.mat.uniroma2.it/~tauraso/GRA20/main.html

たぶん、The American Mathematical Monthly (http://www.maa.org/) って雑誌の問題だと思う。
うちのDQN底辺大学の図書館、2007年度から購読を止めたので読めなくなった…。
ド田舎だから、県内に他に理系大学ないから、もう読めないぜ…
定期購読するしかないのか?
16132人目の素数さん:2007/05/30(水) 22:56:41
>8
 √{x/(y+z)} + √{y/(z+x)} + √{z/(x+y)} ≧ (√x + √y + √z)^2 / [√{x(y+z)} + √{y(z+x)} + √{z(x+y)}] > 2.

(略証)
左側:
 コーシーで簡単。
右側:
 x, y ≦ z としてもよい。
 f(z') = √z' のグラフは上に凸だから、接線の下側にある。
 √(z+y) < √{z+y+(y^2)/(4z)} = √z + y/(2√z) < √z + (√y)/2,
 √(z+x) < √{z+x+(x^2)/(4z)} = √z + x/(2√z) < √z + (√x)/2,
 √{z(x+y)} < {(x+y)+z}/2,  (← 相加・相乗平均)
これらに √x, √y, 1 を掛けてたすと、
 √{x(y+z)} + √{y(z+x)} + √z(x+y)} < √(zx) + √(zy) + √(xy) + (x+y+z)/2 = (1/2)(√x + √y + √z)^2,
よって上式を得る。 (終)

ハァハァ ゼェゼェ…
17132人目の素数さん:2007/05/31(木) 00:56:20
>13
x+y=s, xy=t とおく(基本対称式),
 題意より s+t=1, s>0, t>0,
 絶対不等式 s^2 -4t = (x-y)^2 ≧0,
より
 0.828427… = 2(√2 -1) ≦ s < 1,
 (与式) = s/t + 1/s = s/(1-s) + 1/s = 1/{s(1-s)} -1 ≧ 5(1+√2)/2 = 6.0355339…
 等号は x = y = s/2 = √2 -1 のとき。
18132人目の素数さん:2007/05/31(木) 10:48:17
実数x,y,zについて以下の不等式が成り立つことを示せ。また、等号成立条件を求めよ。

(x^3 + y^3 + z^3 - 3xyz)^2 ≦ (x^2 + y^2 + z^2)^3
19132人目の素数さん:2007/06/01(金) 00:29:33
>>18
(右辺)-(左辺)=(xy+yz+zx)^2(x^2 + y^2 + z^2 + (x-y)^2 + (y-z)^2 + (z-x)^2)≧0
より不等式は成立。
この表式より,等号成立条件は xy+yz+zx=0 または x=y=z=0 だが,まとめると xy+yz+zx=0
20132人目の素数さん:2007/06/01(金) 01:03:57
>14
[19.] Series involving e. Find the sum of the following series:
Σ[n=1,∞) {e -(1 +1/n)^n}


n*log(1 +1/n) = n*{1/n -1/(2n^2) +O(1/n^3)} = 1 -1/(2n) +O(1/n^2),
(1 +1/n)^n = e*{1 -1/(2n) +O(1/n^2)},
e -(1 +1/n)^n = e/(2n) + O(1/n^2). 
(e/2)Σ 1/n 〜 (e/2)log(n) より対数発散…


蛇足だが、↓ならば収束すると思われ…
 Σ[n=1,∞) {e -(1 +1/n)^(n +1/2)}
21132人目の素数さん:2007/06/01(金) 21:58:36
>>16
> 左側:
>  コーシーで簡単。

シュワちゃんをつかうと、分子は (x+y+z)^2 にならないですか?
22132人目の素数さん:2007/06/01(金) 21:59:37
>>16
> 左側:
>  コーシーで簡単。

シュワちゃんをつかうと、分子は (x+y+z)^2 にならないですか?
23132人目の素数さん:2007/06/01(金) 23:29:44
>19 には負領域がない。 ゼロ面(node)は↓

http://science6.2ch.net/test/read.cgi/math/1179324003/921-922
さくらスレ217

http://science6.2ch.net/test/read.cgi/math/1180169999/64, 109
さくらスレ218

>21-22
 ま〜たまた…
24132人目の素数さん:2007/06/01(金) 23:36:54
>>23
解説してね。
25132人目の素数さん:2007/06/02(土) 00:00:26
>>23
解説してよん
26132人目の素数さん:2007/06/02(土) 04:18:27
キモ
27132人目の素数さん:2007/06/02(土) 04:24:38
>>24-26
荒らしは自重しましょう
28132人目の素数さん:2007/06/02(土) 06:38:36
>>16
左側の証明が分からないのでお願いします。
29132人目の素数さん:2007/06/02(土) 11:43:22
知恵遅れはこのスレに来るなよ。目障り
30132人目の素数さん:2007/06/02(土) 12:16:19
>29 荒らすなよ.このスレでは仲良くしろ.
>23 性格変わった?意地悪せずに答えてやれよ.俺が分かるなら答えてやるんだけど.
31132人目の素数さん:2007/06/02(土) 12:55:10
つ Cauchy-Schwarzの不等式 (a^2+b^2+c^2)(x^2+y^2+z^2) ≧ (ax+by+cz)^2

[√{x/(y+z)} + √{y/(z+x)} + √{z/(x+y)} ]・[√{x(y+z)} + √{y(z+x)} + √{z(x+y)}]
 ≧ (√x + √y + √z)^2

不等式ヲタは仲良くしよう ( ゚∀゚) テヘッ
32132人目の素数さん:2007/06/02(土) 13:11:00
a_1,a_2,a_3,・・・・a_n のk次基本対称式をe_k (k=1,2,...n)

F(k)≡(e_k/nCk)^(1/k) とするとき

F(k)≧F(k+1)
33132人目の素数さん:2007/06/02(土) 13:27:45
>>32
まとめwikiの過去スレミラーから探せ。
34132人目の素数さん:2007/06/02(土) 15:13:37
>32
 a_1,a_2,…,a_n >0 のとき、…

* nに関する帰納法
 Part1 の 257, 263(1), 269, 271
 数セミ増刊「数学の問題 = 第@集」日本評論社 (1977/02) No.21 の解説(本文)

* 対称式, Muirhead
 第2章の 800, 810-818
 http://www.ams.org/proc/2004-132-09/S0002-9939-04-07384-8/home.html

* 微分法
 Part.1 の 480-481
 数セミ増刊「数学の問題 = 第@集」日本評論社 (1977/02) No.21 の解説(追補)
 E.F.Beckenbach and R.Bellman: "Inequalities, Ergebnisse der Mathematik und ihrer Grenzgebiete", Band 30, Springer-Verlag, Berlin (1961) p.11
3523:2007/06/02(土) 15:27:17
>24-25
 亀レスだが…

Q. ゼロ面(node) xy+yz+zx =0 はどんな形?

A. 座標軸を回して Z=(x+y+z)/√3 とし、(X,Y,Z)で直交系をなすようにすれば
 xy+yz+zx = {(x+y+z)^2 -x^2 -y^2 -z^2}/2 = {3Z^2 -X^2 -Y^2 -Z^2}/2 = Z^2 -(X^2 +Y^2)/2,
 よって、円錐面で、主軸はZ軸 すなわち x+y+z の方向。
36132人目の素数さん:2007/06/02(土) 18:31:58
>>35
ナルホドナー
3716:2007/06/02(土) 21:30:14
>21-22,28

http://science6.2ch.net/test/read.cgi/math/1110615777/
シュワちゃんスレ
38132人目の素数さん:2007/06/03(日) 11:25:41
>8
 √{z/(x+y)} = z/√{z(x+y)} > 2z/{(x+y)+z},   (← 相乗・調和平均)
循環的にたす.
39132人目の素数さん:2007/06/03(日) 12:20:48
>>8
見事。
40132人目の素数さん:2007/06/03(日) 19:53:07
>>38
(*゚∀゚)=3 ハァハァ…
41132人目の素数さん:2007/06/04(月) 03:02:02
>15

[C.851]
 b^2 +c^2 +bc = (3/4)(b+c)^2 + (1/4)(b-c)^2 ≧ (3/4)(b+c)^2,
 a√(b^2 +c^2 +bc) ≧ {(√3)/2}(ab+ca),
循環的にたす。


[C.844]
Σ[k=1,n] 1/k -γ -log(n) = ε(n) とおくと,
Σ[n=1,N] ε(n)/n = log(N)ε(N) + (1/2)ε(N)^2 - (1/2)γ^2 + (1/2)Σ[n=1,N] (1/n^2) + {(1/2)log(N)^2 - Σ[k=1,N] log(k)/k }
 → -(1/2)γ^2 + (1/2)ζ(2) - L (N→∞).
42132人目の素数さん:2007/06/04(月) 15:02:00
何者だ一体?
すげー実力w
43132人目の素数さん:2007/06/04(月) 17:59:41
照れるぜ・・・
44132人目の素数さん:2007/06/04(月) 18:26:30
>>42
不等式マニア
45132人目の素数さん:2007/06/04(月) 18:37:23
>>42
不等式ヲタは共同体で連続体で群生体だから、無限の知識と無尽蔵の体力を持ってるんだYO!
46132人目の素数さん:2007/06/04(月) 18:46:09
恒等式ヲタ出現きぼん
47132人目の素数さん:2007/06/04(月) 21:51:55
実数x,y,zについて以下の不等式が成り立つことを示せ。また、等号成立条件を求めよ。

 (x^3 + y^3 + z^3 - 3xyz)^2 + (xy+yz+zx)^3 ≦ (x^2 + y^2 + z^2)^3.
48132人目の素数さん:2007/06/05(火) 01:12:09
>15

[C.846]
 (1/n)Σ[k=1,n] {C[n,k]}^(-k) ≧ {(n+1)/(2^n)}^((n+1)/2).

(略解)
・n=1,2 は直接確かめる。
・n≧3 のとき、k=n の項だけ残す。
 n^2 ≦ 2^(n+1),
 n+1 ≦ 2^(n-1),
 (左辺) ≧ 1/n ≧ (1/2)^((n+1)/2) ≧ {(n+1)/(2^n)}^((n+1)/2) = (右辺).
49132人目の素数さん:2007/06/05(火) 01:52:56
>>47
(右辺)-(左辺)=(3/2)(xy+yz+zx)^2((x-y)^2 + (y-z)^2 + (z-x)^2)
より不等式は成立。
等号成立条件は xy+yz+zx=0 または x=y=z
50132人目の素数さん:2007/06/05(火) 02:32:32
>>15  [C854]
H_k=∫[0,1]Σ[j=0,k-1]t^j dt=∫[0,1](1-t^k)/(1-t) dt
n!/Π[j=0,n](k+j)=Γ(n+1)Γ(k)/Γ(n+k+1)=Β(k,n+1)=∫[0,1]t^(k-1)*(1-t)^n dt
これらから
H_k*n!/Π[j=0,n](k+j)
=∫[0,1](1-t^k)/(1-t) dt*∫[0,1]t^(k-1)*(1-t)^n dt
=[∫[0,x](1-t^k)/(1-t) dt*∫[0,x]t^(k-1)*(1-t)^n dt]_[x=0,1]
部分積分を使うことで
H_k*n!/Π[j=0,n](k+j)
=∫[0,1](1-x^k)/(1-x)∫[0,x]t^(k-1)*(1-t)^n dtdx
 +∫[0,1]∫[0,x](1-t^k)/(1-t) dt*x^(k-1)*(1-x)^n dx
を得る。
kについてこれらの和を取る。(極限の順序交換の大雑把さは大目に見てください)
Σ[k=1,∞]∫[0,1](1-x^k)/(1-x)∫[0,x]t^(k-1)*(1-t)^n dtdx
=∫[0,1]∫[0,x] Σ[k=1,∞] {t^k-(tx)^k}/{t(1-x)}*(1-t)^n dtdx
=∫[0,1]∫[0,x](1-t)^(n-1)/(1-tx) dtdx
=∫[0,1]∫[t,1](1-t)^(n-1)/(1-tx) dxdt (積分の順序交換)
=∫[0,1]log(1+t)*(1-t)^(n-1)/t dt
同様に
Σ[k=1,∞]∫[0,1]∫[0,x](1-t^k)/(1-t) dt*x^(k-1)*(1-x)^n dx
=-∫[0,1]log(1-x^2)*(1-x)^(n-1)/x dx
=-∫[0,1]log(1-t^2)*(1-t)^(n-1)/t dt (変数をtに書き換えた)

以上から
Σ[k=1,∞]H_k*n!/Π[j=0,n](k+j)
=∫[0,1]log(1+t)*(1-t)^(n-1)/t dt -∫[0,1]log(1-t^2)*(1-t)^(n-1)/t dt
=-∫[0,1]log(1-t)*(1-t)^(n-1)/t dt
=∫[0,∞]y*e^(-ny)/{1-e^(-y)} dy ( y=-log(1-t) と変数変換)
=∫[0,∞]Σ[j=n,∞]y*e^(-jy) dy
=Σ[j=n,∞]1/j^2=π^2/6-Σ[j=1,n-1]1/j^2
ゆえ
Σ[k=1,∞]H_k/Π[j=0,n](k+j)=1/(n!)*{π^2/6-Σ[j=1,n-1]1/j^2}
51132人目の素数さん:2007/06/05(火) 04:11:21
凄い! こんなにガンガン解ければ楽しいだろうな…

A.422、B.3987、B.3989、C.892 (C.892は昔、入試問題で解いたような…)
http://www.komal.hu/verseny/feladat.cgi?a=honap&h=200703&t=mat&l=en
A.425、B.3997、B.4000
http://www.komal.hu/verseny/feladat.cgi?a=honap&h=200704&t=mat&l=en
52132人目の素数さん:2007/06/05(火) 23:57:18
三角関数の問題
△ABCが sin2A/5 = sin2B/4 = sin2C/3 をみたすとき、 Aの値を求めよ。

問54_3
http://www.asahi-net.or.jp/~nj7h-ktr/kadai06-07.html
53132人目の素数さん:2007/06/06(水) 01:32:46
>52
 A,B,C は同時に鋭角、直角または鈍角。
 3つとも鈍角、直角は不合理なので、鋭角3角形。
 A=arctan(1)=45゚, B=arctan(2), C=arctan(3).
54132人目の素数さん:2007/06/06(水) 13:40:18
何所が不等式や
55132人目の素数さん:2007/06/06(水) 16:42:24
>54
ハァハァできればいいのさ
>53
過程がよくワカリマセン
56132人目の素数さん:2007/06/07(木) 01:28:47
>>15  [C.854] 別法

 S_n = (n!)Σ[k=1,∞) (H_k)/[k(k+1)……(k+n)] とおき、
 S_1 = (π^2)/6 と S_n - S_(n+1) = 1/(n^2) から
 S_n = (π^2)/6 -1 -1/(2^2) - … - 1/{(n-1)^2}
を示す。

S_1 = Σ[k=1,∞) (H_k)/[k(k+1)]
  = Σ[k=1,∞) (H_k){1/k - 1/(k+1)}
  = Σ[k=1,∞) {(H_k)/k - H_(k-1)/k}        (← H_0 = 0 )
  = Σ[k=1,∞) 1/(k^2)
  = ζ(2)
  = (π^2)/6.

S_n - S_(n+1) = Σ[k=1,∞) (H_k){ (n!)/[k(k+1)…(k+n)] - (n+1)!/[k(k+1)…(k+n+1)] }
  = (n!)Σ[k=1,∞) (H_k)/[(k+1)……(k+n+1)]
  = (n-1)!Σ[k=1,∞) (H_k){1/[(k+1)…(k+n)] - 1/[(k+2)…(k+n+1)] }
  = (n-1)!Σ[k=1,∞) { (H_k)/[(k+1)…(k+n)] - H_(k-1)/[(k+1)…(k+n)] }  (← H_0 =0)
  = (n-1)!Σ[k=1,∞) 1/[k(k+1)…(k+n)]
  = (n-1)!Σ[k=1,∞) (1/n){ 1/[k(k+1)…(k+n-1)] - 1/[(k+1)…(k+n)] }
  = (n-1)!(1/n)(1/n!)
  = 1/(n^2).
57132人目の素数さん:2007/06/07(木) 21:52:14
>51 上

[A.422]
Let x_1,x_2,…,x_n,x_(n+1) be positive real numbers with x_1+x_2+…+x_n = x_(n+1).
Prove that
 Σ[i=1,n] √{x_i[x_(n+1) - x_i]} ≦ √{ Σ[i=1,n] x_(n+1)[x_(n+1) - x_i]},
(略解)
(左辺) ≦ (1/n){Σ[i=1,n] √x_i)}{Σ[j=1,n] √(x_{n+1} - x_j)}  (← 逆順序積 ≦ 乱順序積)
  ≦ √{Σ[i=1,n] x_i}・√{Σ[j=1,n] (x_{n+1} - x_j)}       (← コーシー)
  = √(x_{n+1})・√{Σ[j=1,n] (x_{n+1} - x_j)}
  = √{Σ[j=1,n] x_{n+1}[x_{n+1} - x_j]}
  = (右辺).

[B.3989]
 a,b,c are positive real numbers, such that a^2 +b^2 +c^2 +abc = 4.
 Prove that a+b+c ≦ 3.
(略解)
 1-a,1-b,1-c のうち2つは同符号、よって (1-b)(1-c) ≧0 としてもよい。
 3(3-a-b-c) + (a^2 +b^2 +c^2 +abc -4)
 = (1/2)(2-a-b)^2 + (1/2)(2-b-c)^2 + (1/2)(2-c-a)^2 -(1-a)(1-b)(1-c)
 = (1/4)(3-2c-a)^2 + (1/4)(3-2b-a)^2 + (1/2)(1-a)^2 + a(1-b)(1-c)
 ≧ a(1-b)(1-c).

[C.892]
Prove that if x,y,z are positive real numbers and xyz=1, the values of the expressions
  1/(1+x+xy), y/(1+y+yz), xz/(1+z+xz)
cannot all be greater than 1/3.
(略解)
  1/(1+x+xy) = yz/(1+y+yz) = z/(1+z+xz),
 xy/(1+x+xy) = y/(1+y+yz) = 1/(1+z+xz),
  x/(1+x+xy) = 1/(1+y+yz) = xz/(1+z+xz),
辺々たす.
58132人目の素数さん:2007/06/07(木) 21:58:46
>51 下

[B.3997]
 x,y,z are real numbers. Prove that if xyz=u, then
 x^4 + y^4 + z^4 + x^2・y^2 + y^2・z^2 + z^2・x^2 ≧ 2u(x+y+z).

(略解)
 (左辺) - (右辺) = (1/2)(x^2 -y^2)^2 + (1/2)(y^2 -z^2)^2 + (1/2)(z^2 -x^2)^2
         + (x^2)(y-z)^2 + (y^2)(z-x)^2 + (z^2)(x-y)^2.

ハァハァ
59132人目の素数さん:2007/06/07(木) 22:47:53
>>57
[C.892]の解答みて思い出した。
俺は この問題を解いて(いや解けずに答えを見て)、
不等式の世界に入ったんだ(いや囚われの身になったんだ)!
ハァハァ・・・
60132人目の素数さん:2007/06/08(金) 15:06:36
以前あった問題
a,b,c>0
ab+bc+ca=1で
(1+a^2+b^2)/(a+b)^2+(1+b^2+c^2)/(b+c)^2+(1+c^2a^2)/(c+a)^2≧5/2
を誰か解決してくれ。もうノート2冊分くらいぐるぐるしてる。
ちなみにいじってるうちに
(a-b)^2+(b-c)^2+(c-a)^2≧(a-b)^2/(a+b)^2+(b-c)^2/(b+c)^2+(c-a)^2/(c+a)^2
と同値とか、わきにそれてばかりいる。

大体、2文字対称不等式だと、大抵そんなむずかしくはないし、使う式も大体
決まってるんだが、どうも、3文字対称不等式はめんどくさい。誰か解いてくれ。
それから、不等式もそろそろ分類してもいい頃あいだと思うんだが、、、。
むずかしそうで、ルーチンで解ける物、それ以外(これが多いから収集したりする
訳だが、、、)。

結局、相加相乗平均を使うだけの問題も多いと思う。
61132人目の素数さん:2007/06/08(金) 16:44:59
対称不等式だから
(1+a^2+b^2)/(a+b)^2+(1+b^2+c^2)/(b+c)^2+(1+c^2 + a^2)/(c+a)^2≧5/2
でいいんだよな

a+b+c=s, ab+bc+ca=t, abc=uとおいて
(t^2+a^2+b^2)/(a+b)^2+(t^2+b^2+c^2)/(b+c)^2+(t^2+c^2+a^2)/(c+a)^2≧15/4
を示す
t^2+a^2+b^2=t^2-2ab+(a+b)^2等を使えば
(t^2-2ab)/(a+b)^2+(t^2-2bc)/(b+c)^2+(t^2-2ca)/(c+a)^2≧3/4と同値
左辺=f/(a+b)^2(b+c)^2(c+a)^2とすれば、根性で対称式で書いて
4f-3(a+b)^2(b+c)^2(c+a)^2
=4s(st-u)(s^2-3t)+(18stu+s^2t^2-27u^2-4t^3-4su)
st-u≧0は容易、
18stu+s^2t^2-27u^2-4t^3-4su≧0は
3次方程式x^3-sx^2+tx-u=0が実数解のみを持つ条件
s^2-3t≧0はその解がすべて0以上の条件で、終り

間違ってるかもしれん。正しい/いい解法は実力者を待て
62132人目の素数さん:2007/06/08(金) 16:55:22
18stu+s^2t^2-27u^2-4t^3-4su≧0と
s^2-3t≧0
合わせて実数解条件だったかもしれん
正の解の条件ではないな、
まあなんか本を見ておくれ
63132人目の素数さん:2007/06/08(金) 17:07:40
手許にあった論文
a new look at newton's inequalities (結構面白い、webで拾えると思う)
によると
18stu+s^2t^2-27u^2-4t^3-4su≧0は実数解条件でwell-knownだそうだ
s^2-3t=(1/2){(a-b)^2+(b-c)^2+(c-a)^2}
は理くつ不要だった

スレ汚しすまん
64132人目の素数さん:2007/06/08(金) 17:13:51
18stu+s^2t^2-27u^2-4t^3-4s ^3 u
なんかボロボロorz
65132人目の素数さん:2007/06/08(金) 19:30:22
またt^2とか間違いを見つけたので直したの貼り直しときます

a,b,c>0に対し、a+b+c=s, ab+bc+ca=t, abc=uとおいて
(t+a^2+b^2)/(a+b)^2+(t+b^2+c^2)/(b+c)^2+(t+c^2+a^2)/(c+a)^2≧15/4
を示す。これはt+a^2+b^2=t-2ab+(a+b)^2等を使えば
(t-2ab)/(a+b)^2+(t-2bc)/(b+c)^2+(t-2ca)/(c+a)^2≧3/4と同値。
左辺=f/((a+b)^2(b+c)^2(c+a)^2)とすれば、
4f-3(a+b)^2(b+c)^2(c+a)^2
=4s(st-u)(s^2-3t)+(18stu+s^2t^2-27u^2-4t^3-4s^3u)
と基本対称式で書ける。(一応乱数入れてチェックしてみた)
ここで、s^2-3t=(1/2)((a-b)^2+(b-c)^2+(c-a)^2)≧0...(*)、
st-u=(a+b+c)(ab+bc+ca)-abc
≧3(abc)^(1/3)・3(ab・bc・ca)^(1/3)-abc=8abc>0であり、
D_3=18stu+s^2t^2-27u^2-4t^3-4s^3u≧0は
3次方程式x^3-sx^2+tx-u=0が実数解のみを持つ条件で
a,b,cがこの方程式の3解だから満たされている
等号条件は(*)よりa=b=cが必要で、このとき実際成立する。
66132人目の素数さん:2007/06/08(金) 21:44:08
>>60
あまり考えてないけど、x=b+c, y=c+a, z=a+b と置き換えて

(ab+bc+ca+a^2+b^2)/(a+b)^2

をx,y,zで表したらどうなる?
67132人目の素数さん:2007/06/08(金) 22:49:39
>>63
これでおじゃるかな?
( ゚∀゚)つ http://www.emis.de/journals/JIPAM/article111.html?sid=111
68132人目の素数さん:2007/06/09(土) 04:33:02
>>67
そうです、ありがと

やってみたらたまたまできた?だけなんで
素人の力づく解法でみっともないかもしれない

このスレすごい人がいるから期待してる
69132人目の素数さん:2007/06/09(土) 06:09:48
>>66
それでうまく行きますね、俺何やってんだか・・・やっぱりみっともなかったw
70132人目の素数さん:2007/06/09(土) 14:26:36
>15

[M.1769]
Determine a formula for the coefficient of x^i・y^j in
P_n(x,y) = Σ[k=0,n] C[2n+1,2k+1] x^(n-k)・(x+y)^k.

(略解)
 {(1+u)^(2n+1) - (1-u)^(2n+1)}/(2u) = Σ[k=0,n] C[2n+1,2k+1] (u^2)^k = Q(u^2)
とおくと,
 P_n(x,y) = (x^n)Q({x+y}/x) = Σ[i=0,n] C[n+i,n-i] (4x)^i・y^(n-i) = Σ[j=0,n] C[2n-j,j] (4x)^(n-j)・(y^j).

テヘッ
71132人目の素数さん:2007/06/09(土) 19:52:22
>>70
キタッ!wヘ√レv-(゚∀゚)-ヘ√レ- !! スンバラスィ!
72訂正>>60:2007/06/10(日) 16:03:35
(1+a^2*b^2)/(a+b)^2+(1+b^2*c^2)/(b+c)^2+(1+c^2*a^2)/(c+a)^2≧5/2
ごめんなさい。
73132人目の素数さん:2007/06/10(日) 19:28:51
>>72
( ゚д゚)ポカーン
74132人目の素数さん:2007/06/10(日) 20:28:38
>>51

[A.425]
 Let n≧2 and let a_1,a_2,…,a_n, x_1,x_2,…,x_n be positive real numbers such that a_1+a_2+ … +a_n =A, x_1+x_2+ … + x_n =X.
Prove that
 2Σ[1≦i<j≦n] x_i・x_j ≦ {(n-2)/(n-1)}X^2 + Σ[i=1,n] {a_i/(A-a_i)}(x_i)^2.

(略解)
コーシーより
 Σ[i=1,n] {A/(A-a_i)} (x_i)^2 ≧{Σ[k=1,n] x_k}^2 /{Σ[j=1,n] (A-a_j)/A } = {1/(n-1)}X^2,
よって
 (右辺) = {(n-2)/(n-1)}X^2 + Σ[i=1,n] {A/(A-a_i)}(x_i)^2 - Σ[i=1,n] (x_i)^2
  ≧ {(n-2)/(n-1)}X^2 + {1/(n-1)}X^2 - Σ[i=1,n] (x_i)^2
  = X^2 - Σ[i=1,n] (x_i)^2
  = 2Σ[1≦i<j≦n] x_i・x_j
  = (左辺).

[B.3997] >>58

[B.4000]
 Find the smallest possible value of x^2 +y^2, given that x and y are real numbers, x≠0 and xy(x^2 -y^2) = x^2 +y^2.

(略解)
 (1/4)(x^2 +y^2)^2 = (1/4){(2xy)^2 + (x^2 -y^2)^2} ≧ (1/2)(2xy)(x^2 -y^2) = xy(x^2 -y^2)
と与式から
 x^2 +y^2 ≧ 4,
等号成立は 2xy = x^2 -y^2,
 (x,y) = (±√(2+√2), ±√(2-√2)), (±√(2-√2), 干√(2+√2)) 〔複号同順〕

ハァハァ
75132人目の素数さん:2007/06/15(金) 09:35:26
△ABCが鋭角三角形のとき,
tanA tanB tanC ≧ 3√3
を示せ。
76132人目の素数さん:2007/06/15(金) 10:23:38
>>75
tanA=x tanB=y tanC=zとおく。
△ABCが鋭角三角形な事からx,y,zは全て正の数。
また z=tan(π-(A+B))=-tan(A+B)=-(x+y)/(1-xy) ゆえ
z-xyz=-x-y すなわちx+y+z=xyzが成り立つ。
x/(xyz)^(1/3)>0 ,y/(xyz)^(1/3)>0, z/(xyz)^(1/3)>0
について、相加相乗平均から
(x+y+z)/(xyz)^(1/3)≧3{xyz/(xyz)}^(1/3)=3
x+y+z=xyzから (xyz)^(2/3)≧3 ゆえxyz≧3√3
等号成立はx=y=zのとき、それは
0<θ<π/2でのtanθの狭義単調増加性から
A=B=Cのときなので△ABCが正三角形のとき。
77132人目の素数さん:2007/06/15(金) 11:19:19
>>75
tanA=x tanB=y tanC=zとおく。
△ABCが鋭角三角形な事からx,y,zは全て正の数。
また z=tan(π-(A+B))=-tan(A+B)=-(x+y)/(1-xy) ゆえ
z-xyz=-x-y すなわちx+y+z=xyzが成り立つ。

…までは>>76と同じで,ここからはtan xの凸不等式でおしまい。
78132人目の素数さん:2007/06/15(金) 11:51:32
>>77
コピペの上に「〜でおしまい。」って。
人の事馬鹿にしてるのじゃなければ
もう少し書き様って物があるんじゃないですか?
79132人目の素数さん:2007/06/15(金) 12:28:50
>>75
解析的な証明。

tanA=x tanB=yとおく。
まず,A,B<π/2よりx,yは正の数。
また,C<π/2 なので,tan(π-(A+B))=-(x+y)/(1-xy)>0 だから xy>1
つまり,x,y>0, xy>1 の条件下において,xy(x+y)/(xy-1)≧3√3 を示すことになる。

s=x+y, t=xy とおくと,s,t の変域は s>0,t>1,s^2-4t≧0.
この条件下で,st/(t-1)≧3√3 を示せばよい。

言い換えれば,t>1, s≧2√t ⇒ s≧3√3 (1-1/t) を示せばよい。
つまり,t>1 ⇒ 2√t≧3√3 (1-1/t) を示せばよい。

f(t)=2√t - 3√3 (1-1/t) とおけば,f'(t)=0 となるのは t=3 のときで,このとき最小値をとる。
よって f(t)≧f(3)=0 なので示された。
80132人目の素数さん:2007/06/15(金) 12:39:16
ごめん,怒られてる理由がいまいちわかんない。
だってほんとにtan xの凸不等式でおしまいじゃん。w
81132人目の素数さん:2007/06/15(金) 12:49:29
>>78
凸不等式を知っているかね? オービーくんッ!
82132人目の素数さん:2007/06/15(金) 18:07:05
>>80
見通しの悪い駄解答を書き込んですまんかったな。
わざわざコピペ引用までして晒し上げたうえに
あてつけの一言まで頂いたけるとは思わなかったよ。
83132人目の素数さん:2007/06/15(金) 23:55:34
仲良くしようぜ ('A`)
84132人目の素数さん:2007/06/16(土) 07:16:19
       ★★小泉純一郎と安部は朝鮮人★★
コピペして各板に貼り付けよう 知人にも話そう 政治板もたまには覗こう
小泉純一郎 
・戦前大臣を務めた祖父小泉又次郎は純粋な日本人とされる。だが、純一郎の帰化朝鮮人である父が鮫島姓を買い取り
 又次郎の娘をたぶらかして婿として小泉家に入る そこで小泉家は帰化朝鮮人である純一郎の父に乗っ取られた
 参照http://ja.wikipedia.org/wiki/%E5%B0%8F%E6%B3%89%E7%B4%94%E4%B9%9F
・父親の純也は、鹿児島加世田の朝鮮部落の出身者といわれる 日大卒業名簿には、純也の日本名はなく、
 見知らぬ朝鮮名が書かれているという  
 純也は朝鮮人の帰国事業、地上の楽園計画の初代会長であった
・結婚後、子供をもうけ即離婚した宮本佳代子は在日企業エスエス製薬創業者の孫
・小泉の元秘書官の名前は飯島勲←注目 帰化朝鮮人
・派閥のドン森喜朗も生粋の朝鮮人 ←森も帰化人がよく使う通名
・小泉は、横須賀のヤクザ、稲川会と関係が深い
安倍晋三
・岸家 毛利元就が陶晴賢と厳島沖で戦い大勝を収めた際、寝返って毛利方についた船の
 調達人が「ガン」と称する帰化人であったという 毛利はその功績によって「ガン」を
 田布施周辺の代官に召したてた このガンを岸家の先祖とする説がある
・祖父岸信介が文鮮明と共に 反共団体 国際勝共連合(統一教会)を設立
・官房長官時代統一教会「合同結婚式」に祝電を送り、話題に
・安倍のスポンサーは、下関の朝鮮人パチンコ業者である
・グリコ森永事件時、明らかになった帰化朝鮮人企業森永のご令嬢と結婚
・そのわが国のファーストレディーは電通(会長成田豊、半島生まれの帰化人)勤務という分かりやすい
 経歴の持ち主の朝鮮の血筋
・韓国、中国の留学生に日本の企業に入ってもらうために住居費分、学費免除分、生活費など月計20万〜30万円相当の支給
 日本人のワーキングプア層を全く省みない また帰化系在日系朝鮮人が日本の企業で技術を盗み、半島の現代などの企業に
 伝授していることが深刻な問題になっている 
・多くの朝鮮人が差別を主張し、警察、原発、自衛隊で職を得ている
85132人目の素数さん:2007/06/18(月) 00:26:37
>15

[C.847]
 面積を とおくと、
 r=/s, 4R = 2a/sin(A) =abc/, 
 (竸2)/s = (s-a)(s-b)(s-c)   (← ヘロン)
より
 (左辺) = (r^2・s)^(1/3) = {(竸2)/s}^(1/3) = {(s-a)(s-b)(s-c)}^(1/3),
 (中辺) = √{(4R+r)r /3} = √{[abc + (竸2)/s] /(3s)} = √{[(s-a)(s-b)+(s-b)(s-c)+(s-c)(s-a)] /3},
 (右辺) = s /3 = {(s-a) + (s-b) + (s-c)} /3.
以下ry)

まとめ
 [C.844] >>41 [C.846] >>48 [C.847] ↑ [C.851] >>41 [C.854] >>50, >>56 [M.1769] >>70

ハァハァ
8657:2007/06/18(月) 00:46:37
>51 上

[B.3987]
 Let n≧4 be an integer, and let a_1,a_2,…,a_n denote non-negative real numbers.
Prove that
 Π[k=1,n] (a_k + a_{k+1} + a_{k+2})^2 ≧ (2^n)Π[k=1,n] (a_k + a_{k+1})^2,
where a_{n+1}=a_1, a_{n+2}=a_2.

{略解(in Hungarian)}
(a+t)(t+d) = t(a+t+d) + ad ≧ t(a+t+d),
に t=b+c を入れて
(a+b+c)(b+c+d) ≧ (b+c){(a+b)+(c+d)} ≧ (b+c)・2√{(a+b)(c+d)}.  (←相加・相乗平均)
循環的に掛ける。

http://www.komal.hu/verseny/feladat.cgi?a=feladat&f=B3987&l=en

ゴホゴホ
87132人目の素数さん:2007/06/21(木) 01:36:20
( ゚∀゚)つ問題投下
a,b,c>0 のとき,
 (2/3)(a^2/(b+c)+b^2/(c+a)+c^2/(a+b)) ≧ √((a^2+b^2+c^2)/3)
が成立することを示せ。
88132人目の素数さん:2007/06/21(木) 14:06:45
まず(b+c)/2≦√((b^2+c^2)/2)等だから、
(2/3)(a^2/(b+c)+b^2/(c+a)+c^2/(a+b)) ≧(√2/3)(a^2/√(b^2+c^2)+...)
よってa^2=x等とおいて、
(√2/3)(x/√(y+z)+y/√(z+x)+z/√(x+y))≧√((x+y+z)/3)
を示せばよい。
k>0に対し、f(x)=x/√(k-x)は0<x<kで凸であるから、k=x+y+zとしておけば、
(√2/3)(x/√(y+z)+y/√(z+x)+z/√(x+y))=√2・(f(x)+f(y)+f(z))/3
≧√2f((x+y+z)/3)=√((x+y+z)/3)
89132人目の素数さん:2007/06/22(金) 00:19:04
>87
 (2/3){a^2/(b+c)+b^2/(c+a)+c^2/(a+b)} ≧ (a^2 +b^2 +c^2)/(a+b+c) ≧ √{(a^2 +b^2 +c^2)/3}.

(略証)
左側
 (左辺)*{(b+c)a^2 + (c+a)b^2 + (a+b)c^2} ≧ (2/3)(a^2 +b^2 +c^2)^2,  (←コーシー)
 (b+c)a^2 + (c+a)b^2 + (a+b)c^2 ≦ (2/3)(a+b+c)(a^2 +b^2 +c^2),  (← 逆順序積 ≦ 乱順序積)
 辺々割る。
右側
 (a+b+c)^2 = 3(a^2 +b^2 +c^2) -(a-b)^2 -(b-c)^2 -(c-a)^2 ≦ 3(a^2 +b^3 +c^2),
 ∴ 1/(a+b+c) ≧ 1/√{3(a^2 +b^2 +c^2)}.

ハァハァ
9088:2007/06/22(金) 00:25:00
>>89
すげーな、プロの味がするw
9189:2007/06/22(金) 00:39:29
>89 の左側の別解

 (左辺) ≧ (2/9)(a^2 +b^2 +c^2){1/(b+c) + 1/(c+a) + 1/(a+b)} (← 同順序積 ≧ 乱順序積)
     ≧ 2(a^2 +b^2 +c^2) / {(b+c) + (c+a) + (a+b)}     (← 相加・調和平均)
     = (a^2 +b^2 +c^2) / (a+b+c).
92132人目の素数さん:2007/06/24(日) 01:39:02
>87

〔系〕 a,b,c>0 のとき
 (2/3){a^2/(b+c) +b^2/(c+a) +c^2/(a+b)} ≧ (a^2 +b^2 +c^2)/(a+b+c) ≧ √{(a^2 +b^2 +c^2)/3} ≧ (a+b+c)/3 ≧ (abc)^(1/3).


2. (IMO 1995 Canada)
 Let a,b,c be positive real numbers such that abc=u. Prove that
 1/{a^3・(b+c)} + 1/{b^3・(c+a)} + 1/{c^3・(a+b)} ≧ (3/2)u^(-4/3).

(略証) 上式の a→1/a, b→1/b, c→1/c, u→1/u とおく。
93132人目の素数さん:2007/06/25(月) 00:14:52
並べ替えの不等式について質問です。

n!個の式のうち一番大きくなるは正順で、一番小さくなるのが逆順ですが
残りの中間の不等式で大小関係がはっきりつくグループは何個なんでしょうか?

n=3 のときは中間の3!−2=4個の式が2個のグループに分けられるみたいですが。
94132人目の素数さん:2007/07/01(日) 13:14:08
>>93
面白いね、これって論文1本書ける問題じゃないのかな
専門家のコメント希望
95132人目の素数さん:2007/07/01(日) 13:31:10
>>93
n=4 のときにやってみたらハッセ図?みたいのが出来たけど、、
96132人目の素数さん:2007/07/01(日) 13:56:25
(*゚∀゚)=3
97132人目の素数さん:2007/07/01(日) 14:54:06
〔問題〕
a>0 とする。
関数f(x)は上に凸な連続関数で、f(0)=a, f(a)=0 を満たすとする。
また、関数g(x)は、0≦g(x)≦a を満たす連続関数とする。
このとき次の不等式が成り立つことを示せ(下記不等式中にある積分は全て区間[0,a]の定積分とする)。
 ∫f(g(x))dx + ∫g(x)dx + a^2 ≦ 2∫f(x)dx.

* f(x)の微分可能性は保証されていません。

http://science6.2ch.net/test/read.cgi/math/1182629190/58-61
東大入試作問者スレ9
98132人目の素数さん:2007/07/01(日) 15:02:42
>97
 ∫f(g(x))dx + ∫g(x)dx ≦ 2∫f(x)dx.

(略解)
max_[0≦y≦a] {f(y)+y} = M とおくと
 (右辺) ≦ ∫_[0,a] M dx = Ma,
題意により、f(x)+x は上に凸な連続関数。よって、折れ線 (0,f(0))−(h,M)−(a,0) より上側にある。
 (左辺) = 2∫_[0,h] {f(x)+x}dx + 2∫_[h,a] {f(x)+x}dx - 2∫_[0,a] xdx ≧ {f(0)+M}h + (M+a)(a-h) -a^2 = Ma.
9998:2007/07/01(日) 15:13:30
>98の訂正, スマソ

>97
(略解)
max_[0≦y≦a] {f(y)+y} = M とおくと
 (左辺) ≦ ∫_[0,a] M dx = Ma,
題意により、f(x)+x は上に凸な連続関数。よって、y=f(x)+x のグラフは 折れ線 (0,f(0))−(h,M)−(a,a) より上側にある。
 (右辺) = 2∫_[0,h] {f(x)+x}dx + 2∫_[h,a] {f(x)+x}dx - 2∫_[0,a] xdx ≧ {f(0)+M}h + (M+a)(a-h) -a^2 = Ma.
100132人目の素数さん:2007/07/07(土) 00:26:28
http://messages.yahoo.co.jp/bbs?.mm=GN&action=m&board=1835554&tid=bdpbja1jiteybc0a1k&sid=1835554&mid=744 より。

n≧1, m≧2とするとき、
Σ{k=1,n}( (1/k)^((m-1)/m) ) < m n^(1/m)
101132人目の素数さん:2007/07/07(土) 04:27:54
>100

左辺に
 (1/k)^{(m-1)/m} < ∫[k-1,k] (1/x)^{(m-1)/m} dx
を代入するらしいお…
102132人目の素数さん:2007/07/12(木) 11:14:54
x,y,z is possible. Prove

{(xy^2+1)^(1/3)+(yz^2+1)^(1/3)+(zx^2+1)^(1/3)}^3≧xyz+1
103132人目の素数さん:2007/07/13(金) 03:23:30
104102:2007/07/13(金) 04:23:56
>>103
ありがとう!
105102:2007/07/13(金) 09:35:05
>>104 どちらさま?

>>103ありがとうございますっ
っってどうみても>>102には右辺の定数倍が欠けてるっっorz
右辺を3倍いやむしろ27倍してもたぶん成立するという事実
102自体も問題としてはなりたっているが…

103様、もしよろしければ解き直して、wikiのほうも追加してもらえませんか?
申し訳ない
106132人目の素数さん:2007/07/13(金) 10:45:35
|  |             
| ‖           ノノノノ -__  勘違いするなよ!
|>>102         (゚∈゚ )  ─_____ ___
|∧ 从ノ      (ミ_ (⌒\ヽ _ ___
( (≡ ̄ ̄ ̄ ̄三\⌒ノ ノ )
|(つWつ  ̄ ̄\  ⌒彡)   ノ  =_
| \つ つ    \,___,ノノ
|  |  )        / / ≡=
|  |          / ノ      __________
|  |         /ノ _─ (´⌒(´
|  |       ミ/= (´⌒(´⌒;;
| ''''""'''"'''"""''"""'''''"'"''''""''"''''"""''"'''""''"''"'''"''
107132人目の素数さん:2007/07/14(土) 05:35:41
>102

 (xy^2)^(1/3) =Z, (yz^2)^(1/3) =X, (zx^2)^(1/3) =Y とおくと
 (X+Y+Z)/3 ≧ XYZ = xyz,

 g(t) = (t^n +1)^(1/n) とおくと
 g'(t) = t^(n-1) /(t^n +1)^(1 -1/n) >0,    (単調増加)
 g"(t) = (n-1)t^(n-2) / (t^n +1)^(2 -1/n) >0,  (下に凸)

 (左辺)^3 = {g(X) + g(Y) + g(Z)}/3 ≧ g((X+Y+Z)/3) ≧ g((XYZ)^(1/3)) = g((xyz)^(1/3)) = (右辺)^3,
108107:2007/07/14(土) 08:01:35
>102 いつもの事だが訂正

 (X+Y+Z)/3 ≧ (XYZ)^(1/3) = (xyz)^(1/3),
 n>1
 (左辺)^(1/3) = …… = (右辺)^(1/3).
109132人目の素数さん:2007/07/14(土) 12:41:49
102は、f(e^x)がxについて凸な関数のときに、(x>0)
Jensen不等式の相乗平均verが成り立つことを
問題にしたかっただけなんだ。
迷惑かけて申し訳ない。お詫びとして


a,b,cは正の実数。このとき常に次の式が成り立つような最大のαを求めよ

a^b+b^c+c^a>α
110132人目の素数さん:2007/07/14(土) 23:23:13
>109

c=a^(1/a) のとき、
 (左辺) = a^b + b^c + a,
a→0 のとき c⇒0 なので,
 lim[a→0] (左辺) = 0^b + b^0 + 0 = 1,
 α = 1.
111132人目の素数さん:2007/07/16(月) 13:40:04
>110

Q. ほんとに1以下にならない??

A.
(1) a,b,c の1つでも1以上なら おk,
(2) 0<a,b,c<1 のとき
 f(x) = (1/a)^x は下に凸だから、
 (1/a)^b < (1-b) + b/a = (a+b-ab)/a  … ベルヌーイの不等式
 a^b > a/(a+b-ab) > a/(a+b+c),
辺々たす。

http://www.nikonet.or.jp/spring/zettaiti/zettaiti.htm
112132人目の素数さん:2007/07/21(土) 08:13:14
( ゚∀゚)つ>>87の改良版
a,b,c>0 のとき,
 (2/3)(a^2/(b+c)+b^2/(c+a)+c^2/(a+b)) ≧ ((a^3+b^3+c^3)/3)^(1/3)
が成立することを示せ。
113132人目の素数さん:2007/08/02(木) 11:33:35
Polyaの不等式のH.Alzerによる拡張

f,g,h は [a,b] 上の実数値関数で,f は単調増加,g,h はC^1級で,
g(a)=h(a),g(b)=h(b) を満たすものとするとき,
(∫_[a,b] f(x)g'(x)dx) (∫_[a,b] f(x)h'(x)dx)≦(∫_[a,b] f(x)√[(g(x)h(x))']dx)^2
114132人目の素数さん:2007/08/16(木) 02:18:39
〔問題〕
 x+y+z=1, x,y,z ≧0 のとき f(x,y,z) = (x-y)(y-z)(z-x) ≦ 1/(6√3) を示せ。

http://science6.2ch.net/test/read.cgi/math/1186908806/56
分かスレ279
115132人目の素数さん:2007/08/16(木) 02:32:44
>114

f(x,y,z)>0 となるのは 0≦x<y<z またはその cyclic の場合。
そこで、x<y<z の場合を考える。(他の場合も同様)
 f(x,y,z) = (y-x)(z-y)(z-x)
は zについて単調増加、xについて単調減少。
 f(x,y,z) ≦ f(0,y,z+x) = f(0,y,1-y) = y(1-y)(1-2y)
 = 1/(6√3) - 2{y -(1/2) +(1/2√3)}^2・{y +(1/2) +(1/√3)} ≦ 1/(6√3),
等号成立は x=0, y=(1/2)-1/(2√3), z=(1/2)+1/(2√3) のとき。
116132人目の素数さん:2007/08/16(木) 08:23:31
数蝉の最新号に、不等式が載っていたなはぁはぁ…せdfrtgyふじこlp
117132人目の素数さん:2007/08/16(木) 22:37:32
★東大入試作問者になったつもりのスレ★ 第九問
http://science6.2ch.net/test/read.cgi/math/1182629190/634-

【問題】
3辺の長さがa,b,cである三角形の内接円の半径をrとする.
このとき,不等式
  (a + b + c)/r ≧6√3
が成り立つことを示せ.
118132人目の素数さん:2007/08/16(木) 23:54:59
>117
 a/r = cot(B/2) + cot(C/2), …, …
を左辺に代入し、cotθは下に凸, A+B+C=π を使う。
119132人目の素数さん:2007/08/17(金) 22:37:24
【問題】
3辺の長さがa,b,cである三角形の外接円の半径をRとする.
このとき,不等式
   (a + b + c)/R ≦ 3√3
が成り立つことを示せ.
120132人目の素数さん:2007/08/17(金) 22:43:31
>119
だから
 a/R = 2sin(A), …, …
を左辺に代入し、sinθ は上に凸, A+B+C=π を使うだお。

〔系〕R ≧ 2r.

http://science6.2ch.net/test/read.cgi/math/1182629190/638-639
121132人目の素数さん:2007/08/17(金) 23:10:15
〔系〕R ≧ 2r.

これは、球殻不等式というんだお。 (・3・)
122132人目の素数さん:2007/08/18(土) 18:00:49
>121
dクス.

△の3辺の中点を通る円の半径 = R/2. この円は△の3辺を切るから、半径 ≧r. (清水多門氏)

[前スレ.496-499,660,974] 文献[3] p.8 (絶版)
123132人目の素数さん:2007/08/18(土) 20:10:08
【問題】
3辺の長さがa,b,cである三角形の内接円の半径をr, 外接円の半径をR とする.
このとき, 不等式
 (a + b + c - 4R) /r ≦ 6√3 -8 = 2.39230484…,
が成り立つことを示せ.

等号は正3角形のとき,
直角3角形のとき 左辺は2.

http://science6.2ch.net/test/read.cgi/math/1182629190/674
東大入試作問者スレ9
124132人目の素数さん:2007/08/21(火) 00:06:01
>123
このスレの解答は↓になるだろうな。ちっともエレガントぢゃねぇが…

a,b,cが3角形の辺をなすとき、次の附帯条件(3角不等式)がある。
 s-a >0, s-b >0, s-c >0, s=(a+b+c)/2
そこで s-a, s-b, s-c を独立変数と見れば、附帯条件は無くなる。基本対称式を
 (s-a) + (s-b) + (s-c) = s,
 (s-a)(s-b) + (s-b)(s-c) + (s-c)(s-a) = t,
 (s-a)(s-b)(s-c) = u,
とおくと abc = st-u,
  = √{s(s-a)(s-b)(s-c)} = √(su),
 r = /s = √(u/s),
 R = abc/(4) = (st-u) / {4√(su)},
 (左辺) = {2s - (st-u)/√(su)} / √(u/s) = 2(s^1.5)/√u - (st/u) +1,
示すべき式は
 {(st/u) +(右辺-1)}^2 - 4(s^3)/u = (1/t^2)H(s,t,u) ≧0,
 H(s,t,u) = {st + 2(右辺-1)u}sF_(-2) + 27(7-4√3)uF_(-1) + 3(16√3 -27)sG ,
ここに F_n はSchurの不等式のF_nで,
 F_(-2) = (t^3 -4stu +9u^2)/(u^2) ≧0,
 F_(-1) = (t^2 -3su)/u ≧0,
 F_0 = s^2 -3t ≧0,
 G(s,t,u) = st-9u ≧0,
これより、
 H(s,t,u) ≧0,

ぬるぽ
125124:2007/08/21(火) 00:22:21
(補足)

3角形の面積を凾ニおくと、
  = √{s(s-a)(s-b)(s-c)} = √(su),   …… ヘロンの公式
126132人目の素数さん:2007/08/21(火) 11:18:25
グッジョブ! (*゚∀゚)
127132人目の素数さん:2007/08/23(木) 05:27:52
【類題】
3辺の長さがa,b,cである鈍角*三角形の内接円の半径をr, 外接円の半径をR とする.
このとき, 不等式
 (a + b + c - 4R) /r ≦ 2,
が成り立つことを示せ。 (*直角3角形も含める)

等号は直角3角形のとき.
128132人目の素数さん:2007/08/25(土) 10:43:13
>127

 r=/s, R=abc/(4) より,
 (4R+r)r = {(竸2)/s + abc}/s = s^2 + (ab+bc+ca) = (2ab+2bc+2ca-a^2-b^2-c^2)/4 (=t),
 (4R+2r)^2 - (a+b+c)^2 = 16R^2 +4(4R+r)r - (a+b+c)^2
  = 16R^2 + (2ab+2bc+2ca-a^2-b^2-c^2) - (a+b+c)^2
  = 2(8R^2 -a^2 -b^2 -c^2),
  = -16R^2 cos(A)cos(B)cos(C),        (← 補題)

〔補題〕
 (a^2 +b^2 +c^2) -8R^2 = (a^2 +b^2 -c^2) - 2(4R^2 -c^2)
 = 2abcos(C) - 2(4R^2 -c^2)        (← 第2余弦定理)
 = 8R^2 {sin(A)sin(B)-cos(C)}cos(C)    (← 正弦定理)
 = 8R^2 {sin(A)sin(B)+cos(A+B)}cos(C)   (← A+B+C=π)
 = 8R^2 cos(A)cos(B)cos(C),
これは、鋭角・直角・鈍角に従って 正・0・負。(終)
(数セミ, 2007/09)

ぬるぽ
129132人目の素数さん:2007/08/26(日) 13:25:45
〔問題〕
a,b,c は abc=G^3 を満たす正の実数である. 0≦p≦q のとき次の不等式が成り立つことを示せ.
 {a^p + b^p + c^p}*G^(q-p) ≦ a^q + b^q + c^q.

http://science6.2ch.net/test/read.cgi/math/1182629190/753-754 を改作
東大入試作問者スレ9
130132人目の素数さん:2007/08/26(日) 14:54:55
131132人目の素数さん:2007/08/26(日) 15:37:41
>129 相加・相乗平均 と 乱順序積≦同順序積 より 左辺 ≦ (a^p+b^p+c^p)*{a^(q-p)+b^(q-p)+c^(q-p)}/3 ≦ 右辺.

http://science6.2ch.net/test/read.cgi/math/1182629190/766,771
132132人目の素数さん:2007/08/26(日) 16:54:33
>129 は q≦p≦0 のときも成立.

q-p = d とおくと >131 より
 (左辺) ≦ (a^p + b^p + c^p)(a^d + b^d + c^d)/3 = (右辺) + {(a^p -b^p)(a^d -b^d) + (b^p -c^p)(b^d -c^d) + (c^p -a^p)(c^d -a^d)}/3 ≧ (右辺).
133132人目の素数さん:2007/08/27(月) 11:14:33
>>132
確かに>>130の証明も,q≦p≦0のときにも成り立っていますね。
>>130の証明を追記しておきました。
134132人目の素数さん:2007/09/10(月) 22:34:38
IMO longlisted problem 1987
θ[1],θ[2],θ[3]・・・,θ[n]を実数とし、sinθ[1]+sinθ[2]+・・・sinθ[n]=0とするとき次の不等式を示せ。

|sinθ[1]+2sinθ[2]+・・・+nsinθ[n]|≦[n^2/4 ]


The IMO compendium P209 より
この本って問題は豊富なんだけど解答がその半分もないんですね
135132人目の素数さん:2007/09/11(火) 00:01:21
あっさりオイラー使えよ
136132人目の素数さん:2007/09/11(火) 06:43:58
nt-t+(n-1)t-2t...=tn(n+1)/2-2t(n/2)(n-2+1)/2=
137132人目の素数さん:2007/09/11(火) 06:48:20
tn(n+1)/2-2t(n/2)(n/2+1)/2= f

df/dt=n(n+1)/2-n(n+2)/4=0
nn/4=0
t=1->n^2/4
138132人目の素数さん:2007/09/11(火) 06:55:08
>134

a[k] = sinθ[k+1] + sinθ[k+2] + …… + sinθ[n],
とおく。題意より
 a[0] = a[n] = 0,
また
 |a[k-1] - a[k]| = |sinθ[k]| ≦ 1,
よって
 |a[k]| ≦ k   (k=0,1,2,…,[n/2])
 |a[k]| ≦ n-k  (k=[n/2]+1,・・・,n-1,n)
与式 = | Σ[k=1,n-1] a[k] | ≦ Σ[k=1,n-1] |a[k]| ≦ ・・・

あっさり。
139132人目の素数さん:2007/09/12(水) 16:23:24
>>138 あっさりでしたか。
問題仕入れてきました。1988年/大学への数学「宿題」らしいです。

実数x[1],,x[2],・・・,x[n]が
x[1]+x[2]+・・・+x[n]=0
(x[1])^2+(x[2])^2+・・+(x[n])^2=1
を満たしながら動くとき次の不等式を示せ。ただしnは3以上の整数とする。

(x[1])^3+(x[2])^3+・・・+(x[n])^3≦(n-2)/√(n^2-n)

140132人目の素数さん:2007/09/12(水) 20:02:15
xk=-xn-k+1=t
nt^2=1
t^3=n^-3/2
nt^3=n^-1/2
141132人目の素数さん:2007/09/14(金) 12:07:31
>>139
[略解]
ラグランジュ乗数法で停留点条件を調べると,
x[1],x[2],……,x[n] たちは2種類の値のみをとることが必要と分かる。
そこで x[1] から x[n] のうちで p 個が a という値をとり,(n-p)個が b という値をとるとする。
ただし x[1]=……=x[n] とはなりえないので a<b,1≦p≦n-1 としてよい。
2本の束縛条件の式に代入して解くと, a, b を p の式で表せる。
すると (x[1])^3 + …… + (x[n])^3 が p の関数として表せる。
この関数は p について単調増加なので,p=n-1 のときが最大値。
その最大値は (n-2)/√(n^2-n) となる。
142132人目の素数さん:2007/09/19(水) 12:38:04
★東大入試作問者になったつもりのスレ★ 第十問
http://science6.2ch.net/test/read.cgi/math/1188545067/622-
より転載。

622 :132人目の素数さん:2007/09/19(水) 11:28:28
0<x<eのとき,
 (e+x)^(e-x)>(e-x)^(e+x)
が成り立つことを示せ。ただし e は自然対数の底である。


ちなみにこれに続く>>624の解答は間違い。
143132人目の素数さん:2007/09/23(日) 08:22:51
>142
f(x) = (e-x)log(e+x) - (e+x)log(e-x) とおく。
 f(0) =0,
 f '(x) = (e-x)/(e+x) + (e+x)/(e-x) -log(e+x) -log(e-x)
    = 4(x^2)/(e^2 -x^2) +2 -log(e^2 -x^2)
    = 4(x^2)/(e^2 -x^2) - log{1-(x/e)^2} >0,
∴ 0<x<e ⇒ f(x) >0.
144132人目の素数さん:2007/10/03(水) 21:44:32
〔問題〕
nは自然数、x>0として、(1+x)(1-x)x^n の最大値を、
「微分積分も 相加相乗平均も コーシーの不等式も 因数定理も 判別式も 平方完成も 使わずに」求めよ。

http://science6.2ch.net/test/read.cgi/math/1190854032/272, 293
東大入試作問者スレ11
145132人目の素数さん:2007/10/03(水) 21:52:36
>144

最大値は M = {2/(n+2)}{n/(n+2)}^(n/2) の辺りなので、差をとってみよう。
x・√{(n+2)/n} ≡ y とおくと、
 M - (1+x)(1-x)x^n = M - {1-(n/(n+2))y^2}・{n/(n+2)}^(n/2)・y^n
 = M{1 -(1/2)(n+2)y^n +(n/2)y^(n+2)}
 = M(1-y){1 +y +y^2 +・・・+y^(n-1) -(n/2)(1+y)y^n}
 = M(1-y)^2・{1 +2y +3y^2 +・・・+ny^(n-1) +(n/2)y^n} ≧ 0,
等号成立は y=1 のとき。

もっとも、x≧1 のときは (左辺)≦0 から明らかだが・・・
146132人目の素数さん:2007/10/03(水) 22:23:52
さすがに後付けにもほどがあるな
147132人目の素数さん:2007/10/03(水) 22:51:55
これは、「最大値を求めた」のではなく、最大値を取る付近で適当な変数を取って関数を展開しただけのこと。
微分方程式等、動きが判らない関数の性質を調べるときなど、よく使われる手法。お疲れ様
148132人目の素数さん:2007/10/06(土) 01:13:14
任意の三角形の三辺a,b,cに対して常に
a^(2n)+b^(2n)+c^(2n)<2(a^nb^n+b^nc^n+c^na^n)
が成り立つような正の整数nを全て求めよ。
http://science6.2ch.net/test/read.cgi/math/1190854032/352

まず必要条件を求めるため,xを0<x<1なる実数として,
a=2,b=1,c=1+x という三角形を考える。
このとき,題意を満たす n が存在するとすると,
  2^(n+1) + 2(1+x)^n + 2^(n+1)(1+x)^n > 2^(2n) + 1 + (1+x)^(2n)
が成り立つ。
これが0<x<1なる任意のxに対して成立するので,両辺 x→+0 として,
  2^(n+2) ≧ 2^(2n)
  ∴ 4≧2^n
  ∴ n≦2
よって n≦2 が必要。

n=1のとき,
 2(ab+bc+ca)-(a^2+b^2+c^2)=(b+c-a)(c+a-b)+(c+a-b)(a+b-c)+(a+b-c)(b+c-a) > 0
より成立。

n=2のとき,
 2(a^2b^2+b^2c^2+c^2a^2)-(a^4+b^4+c^4)
 =(a+b-c)^2(b+c-a)(c+a-b)+(a+b-c)(b+c-a)^2(c+a-b)+(a+b-c)(b+c-a)(c+a-b)^2 > 0
より成立。

以上より n=1,2
149132人目の素数さん:2007/10/13(土) 22:17:13
〔問題〕
n を自然数として定積分 I(n) を
  I(n) = ∫[0,π/2] {x・sin(x)}^n dx
で定める。このとき、すべての自然数n に対して I(n+1) > I(n) が成り立つことを示せ。

http://science6.2ch.net/test/read.cgi/math/1190854032/174
東大入試作問者スレ11
150132人目の素数さん:2007/10/13(土) 22:21:20
>149

(略解)
・n = 1 のとき
 I(1) = ∫[0,π/2] x・sin(x) dx = [ sin(x) - x・cos(x) ](x=0,π/2) = 1,
 I(2) = ∫[0,π/2] {x・sin(x)}^2 dx = π/8 + (π^3)/48 = 1.038663・・・,
ゆえ n=1 のとき成立。
・n> 1 のとき
u = ∫[0,x] x'・sin(x') dx' = sin(x) - x・cos(x) はxについて狭義の単調増加。
xの替わりにuを独立変数と考え、x・sin(x) = s(u) とおく。x・sin(x)dx = du から
 I(n) ≡ ∫[0,π/2] {x・sin(x)}^n dx = ∫[0,1] s(u)^(n-1) du,
ここで ヘルダーの不等式 により
 {∫[0,1] s(u)^n du}^((n-1)/n)・{∫[0,1] 1^n du}^(1/n) ≧ ∫[0,1] s(u)^(n-1) du,
 I(n+1)^(1/n) > I(n)^(1/(n-1)),
 I(n) > I(2)^(n-1) > 1,
から
 I(n+1) > I(n),
n> 1 のときも成立。

http://science6.2ch.net/test/read.cgi/math/1190854032/637
東大入試作問者スレ11
151132人目の素数さん:2007/10/15(月) 15:39:52
〔問題〕
α、β、γ は 0 < α,β,γ< π/2 、(sinα)^3 + (sinβ)^3 +(sinγ)^3 =1 を満たす。このとき、以下の不等式が成り立つことを証明せよ。
  (tanα)^2 + (tanβ)^2 + (tanγ)^2 ≧ (3√3)/2

〔略解〕
(sinθ)^3 / (tanθ)^2 = (sinθ)(cosθ)^2 = (sinθ){1-(sinθ)^2}
= 2/(3√3) - {(2/√3) + sinθ}{(1/√3) - sinθ}^2 ≦ 2/(3√3),

∴(tanθ)^2 ≧ {(3√3)/2}(sinθ)^3

θ=α,β,γを代入して辺々足せば得られる。

http://science6.2ch.net/test/read.cgi/math/1190854032/438-462
東大入試作問者スレ11
152132人目の素数さん:2007/10/15(月) 15:43:15
任意の実数 x[1],……,x[n] に対して
 納k=1,n](x[k])^2・cosπ/n ≧ 納k=1,n-1]x[k]x[k+1]-x[n]x[1]
が成り立つことを示せ。

http://science6.2ch.net/test/read.cgi/math/1190854032/656
東大作問者スレ11
153132人目の素数さん:2007/10/18(木) 03:27:58
>152
 2次形式なので行列で表す。半正値であることを使う。

http://science6.2ch.net/test/read.cgi/math/1181970000/196-202
線形代数/線型代数4
154132人目の素数さん:2007/10/26(金) 21:18:01
[問題]
f:[0,1] → R は f(0)=f(1)=0を満たす滑らかな関数とするとき、次を示せ.
∫^1_0 |f'(x) x|^2 dx < 2 ∫^1_0 |f(x)|^2 dx
155132人目の素数さん:2007/10/27(土) 07:39:25
>>154
f(x)=sin(2πx) のとき,f(0)=f(1)=0 で,
2∫_[0,1] |f(x)|^2 dx = 1
∫_[0,1] |x f'(x)|^2 dx = (2π^2)/3 + 1/4 = 6.82……
よって不成立。
156132人目の素数さん:2007/10/27(土) 09:27:36
>>154
f(x)=sin(nπx)のとき,
∫_[0,1] |f(x)|^2 dx = 1/2
∫_[0,1] |x f'(x)|^2 dx = (n^2π^2)/6 + 1/4
なので,>>154の命題は係数2をいかに大きくしても不成立。
157132人目の素数さん:2007/10/28(日) 11:31:45
>>154
成り立たないのですか!
[3] 不等式への招待,大関信雄・大関清太,近代科学社,1987年(絶版)
の本の最後のページにこの手の不等式があって、幾つか自分でやったんですけど、
これだけはどうしても出来なかったが、間違っていたとは思わなかった orz

お騒がせしました。
しかし、直ぐに成り立たないと反例を挙げるその才能に驚きました。
158132人目の素数さん:2007/10/28(日) 18:57:34
〔問題〕
こんな問題が流れてきた。カッコ良く解いて呉れってよ。

x+y+z =s, x≧0, y≧0, z≧0 のとき、
 w(x,y,z) = (y^2+yz+z^2)(z^2+zx+x^2)(x^2+xy+y^2) の最大値は?

http://science6.2ch.net/test/read.cgi/math/1193029141/59
分かスレ280
159132人目の素数さん:2007/10/28(日) 19:03:47
>158
いつものように 基本対称式を x+y+z =s, yz+zx+xy =t, xyz =u とおく。
 y^2 +yz +z^2 = s^2 -t -sx,
 z^2 +zx +x^2 = s^2 -t -sy,
 x^2 +xy +y^2 = s^2 -t -sz,
よって
 w(x,y,z) = (s^2 -t -sx)(s^2 -t -sy)(s^2 -t -sz)
 = (s^2 -t)^3 -(s^2 -t)^2・s^2 +(s^2 -t)ts^2 -us^3
 = (s^2 -t)t^2 -us^3
 ≦ (s^2 -t)t^2 + min{0, -(s^3)(4st-s^3)/9}, (← s^3 -4st ≧ -9u)
ここで t/s^2 =τ, w/s^6 =ω とおくと 0≦τ≦1/3,
 ω ≦ τ^2 - τ^3 + min{0, -(4τ-1)/9},
ω(τ) の増減表から、ωは 0≦τ<1/4 で増加し、1/4<τ≦1/3 では減少する。
ゆえに τ=1/4 で最大値 3/64 をとる。
等号成立は τ =t/s^2 =1/4, u=0 のとき、すなわち
 (x,y,z) = (a,a,0), (0,b,b), (c,0,c).
ぬるぽ
160132人目の素数さん:2007/10/28(日) 20:46:19
問題を投下した者です。ちょっとこのスレ的ではない解ですが…

ω=exp(2πi/3) を用いて問題の関数は
w(x,y,z)=|(yω-z)(zω-x)(xω-y)|^2
と表せる。そこで p=x+yω+zω^2 という変数を考えるとpは複素平面上で
1,ω,ω^2 を頂点とする三角形の内部または周上 (Tとする) を動く。ここで

p-1=(x+yω+zω^2)-(x+y+z)=(1-ω^2)(yω-z)
p-ω=(x+yω+zω^2)-(x+y+z)ω=(ω-1)(zω-x)
p-ω^2=(x+yω+zω^2)-(x+y+z)ω^2=(ω^2-ω)(xω-y)

であるから

w=(1/27)|p^3-1|^2

である。この式の形とTの形状から、pの動く範囲はTのうちの
2π/3≦arg(p)≦4π/3 に制限してもよいことがわかる。このとき
p^3の動く範囲(Dとする)を描いてみればわかるように、max(w)
を与えるpはTの周上のどこかになる。そこで
p = (-1+it√3)/2 (-1≦t≦1)と置いてwを計算してみると

p^3-1 = (3√3/8)(t^2-1)(√3+it)
|p^3-1|^2 = (27/64)(t^2-1)^2(3+t^2)
w = (1/64)(t^2-1)^2(3+t^2)

あとは u=t^2 (0≦u≦1) の3次関数の問題で、u=0で最大となる
ことがわかり、max(w)=3/64 である。最大を与えるpは p=-1/2
のときと p^3の位置が同じp、すなわち p=-1/2,-ω/2,-ω^2/2 である。
161132人目の素数さん:2007/10/28(日) 22:11:57
>>160>>158 の解でございます。
162132人目の素数さん:2007/10/28(日) 22:17:41
おっと >>158 は x+y+z=s になってますね。元の問題は x+y+z=1 です。
163132人目の素数さん:2007/10/29(月) 00:47:04
イパーン化してしまうのが不等式ヲタのSA・GA
164156:2007/10/29(月) 13:38:52
>>157
>>157
確かにその本にはそう書いてありますね。
しかし,前後の文脈を読むと,おそらく著者が言いたかったのは

「f:[0,1] → R は f(0)=f(1)=0 を満たす C^1級の関数で,かつ恒等的に0でないものとする。
このとき,
∫_[0,1] |x f'(x)|^2 dx > (1/4) ∫_[0,1] |f(x)|^2 dx
が成立する。」

ではないかと思われます。

おそらく著者は,
http://links.jstor.org/sici?sici=0002-9890%28197108%2F09%2978%3A7%3C705%3AIIIAFA%3E2.0.CO%3B2-2&size=LARGE&origin=JSTOR-enlargePage
から引用したものだと思われますが,引用元のこの論文において既に同じミスをしています。

この修正版の不等式は,次のようにして示せます。

g(x) = √(x) f(x)$ とおくと,g(0)=g(1)=0 を満たし,かつg'(x)は[0,1]上で恒等的に0ではない。

また,f(x)=x^(-1/2)g(x)なので
{xf'(x)}^2=(1/4)x^(-1){g(x)}^2 - g'(x)g(x) + x{g'(x)}^2 = (1/4){f(x)}^2 - g'(x)g(x) + x{g'(x)}^2

よって,
∫_[0,1] {xf'(x)}^2 dx
=(1/4)∫_[0,1] \{f(x)\}^2dx - (1/2)[{g(x)}^2]_0^1 + ∫_[0,1] x\{g'(x)\}^2 dx
> (1/4)∫_[0,1] {f(x)}^2dx (∵g(0)=g(1)=0, x\{g'(x)\}^2≧0で恒等的に0でない)


この係数1/4の最良性は言えそうで言えない……
165132人目の素数さん:2007/10/29(月) 14:40:59
>>164
ご丁寧な解答ありがとうございます。
逆向きの不等号ならば、本に書いてある方法でできますね。
ちなみに、積分区間は [0,1] となっていますが、これは任意の区間 [a,b]
(ただし,0<a, b< ∞)でも大丈夫ですね。

私も少し調べたのですが、大体関数 f の積分を f やそれらの微分を
使って上から押さえるタイプのが多いようです。
しかし、逆タイプ、つまり、f の微分を f で押さえるというタイプの
式が見つからなかったので、案の定、間違っていたのですね。

そもそも、一般にこの逆向きの不等式は無理なのでしょうかね?
166156:2007/10/29(月) 15:55:37
>>165
おっと,
http://links.jstor.org/sici?sici=0002-9890%28197108%2F09%2978%3A7%3C705%3AIIIAFA%3E2.0.CO%3B2-2&size=LARGE&origin=JSTOR-enlargePage
をよく読むと,
 ∫_[0,1] |x f'(x)|^2 dx < 2∫_[0,1] |f(x)|^2 dx
については,「そういうf(x)が存在する」と主張しているだけでした。
存在を示すだけなら折れ線だけで大丈夫です。

つまり,この論文は間違っておらず,この論文を「不等式への招待」に転載したときに
著者が「存在」を「任意」だと取り違えてしまった,というのが実情でしょう。

>そもそも、一般にこの逆向きの不等式は無理なのでしょうかね?

難しいと思いますね。
直観的に言うと,|f(x)|がいかに小さく抑えられていたとしても,
その小さな幅の中で激しく振動しまくれば,|f'(x)|はいくらでも大きくすることができてしまいます。
逆に,|f'(x)|がある程度小さく抑えられていれば,f(x)の変動が小さいわけですから,
|f(x)|もある程度の幅しか動けなくなります。

また,[0,1]上の関数f(x)を周期1の周期関数と見てexp(2πinx)によって
フーリエ級数展開したときのフーリエ係数をc_nとすると,パーセバルの等式から
 ∫_[0,1] |f(x)|^2 dx = Σ_[n=-∞,∞] |c_n|^2
 ∫_[0,1] |f'(x)|^2 dx = (2π)^2Σ_[n=-∞,∞] n^2|c_n|^2
です。Σ|c_n|^2 と Σn^2|c_n|^2 の収束性の善し悪しを比較しても,
|f'(x)|を|f(x)|で評価することの困難さが分かると思います。
167132人目の素数さん:2007/10/30(火) 22:30:48
>>166
>∫_[0,1] |x f'(x)|^2 dx < 2∫_[0,1] |f(x)|^2 dx
>については,「そういうf(x)が存在する」と主張しているだけでした。
>存在を示すだけなら折れ線だけで大丈夫です。

ご丁寧にありがとうございます。それなら、納得です。
(ところで、JSTORってフリーじゃないですね。)

このタイプの不等式、つまり、微分を評価するのは、偏微分方程式の解の
評価とかで非常に重要で、また、いろいろと応用が多いのですが、
さすがにこれだけの条件では無理ですね。

ただ、不等式の形が特殊なのでいけるのかな?と思ったんですけど、おっしゃる
ように関数が激しく振動してしまうと無理ですよね。

積分型の不等式で何か良い本がございましたら、教えてください。
(洋書でも構いません)
168156:2007/10/31(水) 01:14:28
>>167
今手元にあるわけではなく,以前図書館でパラパラ見たときの記憶ですが,
http://amazon.com/o/ASIN/0444517952
には積分型の不等式が大量に載っていたように思います。
お探しのタイプの不等式が載っているかどうかは分かりませんが。
169132人目の素数さん:2007/11/12(月) 06:25:55
多変数が良いな。L^p, p\neq 2 に関する不等式はないか?
小平-Spencer-NirenbergのL^4 ぐらいで。
170132人目の素数さん:2007/11/12(月) 18:01:36
>>169
お前の負けだな。
171132人目の素数さん:2007/11/12(月) 18:04:38
リクエスト

「二次式」だけで、ごっつい不等式
172132人目の素数さん:2007/11/12(月) 21:02:13
>>171
[問題(激難)]
実数 a_i > 0 (i=1,,,n) のとき、

a_1/(a_2 + a_3) + a_2/(a_3 + a_4) + … + a_{n-1}/(a_{n} + a_1) + a_n /(a_1 + a_2) >= n/2

が成り立つような、n の範囲を求めよ。
173132人目の素数さん:2007/11/12(月) 22:45:09
>172

n≦13 および n(奇数)≦23 については成り立つらしいお。
 http://mathworld.wolfram.com/ShapirosCyclicSumConstant.html
 大関, 「不等式への招待」 近代科学社 (1987) 絶版

n=3〜6 については
 過去スレのミイラ置場の 不等式スレ2.html の >889

n≦13 については
 H.S.Shapiro: "Problem 4603." Amer. Math. Monthly, 61, p.571 (1954).

〔余談〕
(左辺) > n/3 ならば、3以上の自然数について成り立つらしい。
 過去スレのミイラ置場の 不等式スレ1.html の >501
174132人目の素数さん:2007/11/12(月) 23:25:12
>>173
>>171
激難というか、未解決問題じゃねえかよ!
Shapiro の巡回不等式だな。

まあ、答えが直ぐに出る問題もいいが、こんな不等式でも未解決である
ということは不思議だよな。(n によって真偽が異なるし)

これを解いたら、かなりいい雑誌に論文として載るだろうから、挑戦
する価値は十分にあるだろう。
175132人目の素数さん:2007/11/12(月) 23:40:49
不等式に未解決問題があるとは驚いた。
176171:2007/11/12(月) 23:50:29
>>172 どうもです。
>>173 なるほど。
干からびるにはもっていこいということですね。
177132人目の素数さん:2007/11/13(火) 03:22:28
Shapiro's Cyclic Inequality (google)
http://www.google.co.jp/search?hl=ja&q=Shapiro+Cyclic+Inequality&lr=

J. Ineq. Appl.
Shapiro’s cyclic inequality for even n (by P. J. Bushell and J. B. Mcleod)
http://www.hindawi.com/GetArticle.aspx?doi=10.1155/S1025583402000164

In 1954 H. S. Shapiro proposed an inequality for a cyclic sum in n
variables. All the numerical evidence indicates that the inequality
is true for even n≤12 and for odd n≤23.
We give an analytic proof for the case n=12, which implies the former
result. The remaining case n=23 remains an open problem.

2002年の時点ではまだ未解決。
178132人目の素数さん:2007/11/14(水) 00:46:05
>177
グッジョブ!
Full-text PDF もDLして読んでまつ・・・・
179132人目の素数さん:2007/11/14(水) 03:11:28
>>178
他にも Shapiroの巡回不等式関係の論文は山ほどあるから、最新のを探して
から読んだほうがいいよ。
漏れも今どこまで分かっているのか知らないから、もし分かったら教えてちょ。
しかし、Journal of Inequality なんて雑誌があるんだ。

不等式は奥が深いぞ!
やべ〜、はまりそうだ



180132人目の素数さん:2007/11/14(水) 04:48:46
『古田の不等式』は既出?
181132人目の素数さん:2007/11/14(水) 07:30:27
182132人目の素数さん:2007/11/14(水) 08:55:34
古田って前に新聞に出ていたけど、この不等式がよほどいい仕事だと
勘違いしているようだねw

かなり痛い男だw
183132人目の素数さん:2007/11/14(水) 08:57:15


95 :132人目の素数さん:03/02/19 11:18
古田の不等式。
作った本人に聞けばそれが載ってる数学辞典やら他
様々な文献を見せ付けられることでしょう。


96 :132人目の素数さん:03/02/19 11:28
ワロタ


184132人目の素数さん:2007/11/14(水) 20:47:18
問題 次の不等式を証明せよ。ただし0<=x<=1とする。
    1/2 <= 1/1+√x <= 1/1+x二乗
どうしても分かりません;;;誰か解いてください!!;;;;
185132人目の素数さん:2007/11/14(水) 20:51:44
858 名前:132人目の素数さん :2007/11/14(水) 20:41:47
藤川英華っておばはん顔じゃんw
ブスだな


186132人目の素数さん:2007/11/14(水) 21:11:15
>>182
いや、ホンマに凄いことなんやで!

世界的数学者

古田の不等式
http://www.zaikai21.co.jp/zaifuku/back/0310.html
187132人目の素数さん:2007/11/14(水) 21:14:59
国際的に権威のある「数学百科全書」に名前が掲載されている日本人数学者はわずかしか存在しない。

数学百科全書ってなに?
Springerからでている Encyclopaedia of Mathematical Science?
188132人目の素数さん:2007/11/14(水) 21:29:28
ディドロ&ダランベール
189132人目の素数さん:2007/11/15(木) 00:23:04
>>183
すごいな
四年も粘着してるのか
どんな私怨があるんだろ?
190132人目の素数さん:2007/11/15(木) 00:41:41
>>189
どこにいる?
191132人目の素数さん:2007/11/15(木) 10:00:55
おまえのこと?
192132人目の素数さん:2007/11/15(木) 11:14:35
Shapiroの巡回不等式って、
本当に>>172のような単純な形をしているのか?
元の原型はもっと複雑な形をしているんじゃないのか?
不等式の未解決問題にしては妙に単純な形だ。
「不等式への招待」に現れる不等式の中には
何らかの理論の中に現れるものが結構あるのだが。
193132人目の素数さん:2007/11/17(土) 10:15:46
【問題】
f: [0,1] ---> R を C^2 級関数で f(0)=f(1)=0 をみたせば,
次の不等式が常に成立することを示せ:
max_{x ∈ [0,1]} |f(x)| ≦ 1/8 max_{x ∈ [0,1]} |f^{(2)}(x)|.
194132人目の素数さん:2007/11/17(土) 15:15:42
>193

(左辺) = |f(ξ)| とする。(ξ∈[a,b])
X>ξ でも X<ξ でも f(X)≦f(ξ) だから f'(ξ)≧0 かつ f'(ξ)≦0,
∴ f'(ξ) = 0,
 |f'(X)−f'(ξ)| = |(X-ξ)f"(η)| ≦ |X-ξ|・max{|f"(x)|;x∈[a,b]|},
 |f (X)−f (ξ)| ≦ (1/2)(X−ξ)^2・max{|f"(x)|;x∈[a,b]},
題意より f(a)=f(b)=0 だから,
 (左辺) = |f(ξ)| ≦ (1/2){min(ξ-a,b-ξ)}^2・max{|f"|} ≦ (1/8)(b-a)^2・max{|f"|} = (右辺),

注) ξ∈[a,b] ⇒ min{ξ-a,b-ξ} ≦ |b-a|/2 を使った。
195132人目の素数さん:2007/11/18(日) 03:42:56
>180-189

「ある作用素不等式のやさしい証明」
数学(岩波), Vol.40, p.354 (1988)
http://wwwsoc.nii.ac.jp/msj6/sugaku/s-index.html

「それはスペルミスの手紙から始まった/フルタの不等式の成立をめぐって」
数セミ, Vol.32, No.10, 通巻385, p.68-71 (1993.10)
http://www.nippyo.co.jp/maga_susemi/ss9310.htm
196132人目の素数さん:2007/11/18(日) 14:41:48
>>192
そうだよ。
どうしてこの不等式が出てきたのかは知らないが、まだ未解決らしい。

もっとも、問題が簡単な形をしているから易しい、というのは完全な誤解。
それは、フェルマー予想やポアンカレ予想のことを思えば納得行くだろう。

しかし、Shapiro の巡回不等式の場合、n=14, 20 で反例があることは分かっている。
16≦n の場合を大型計算機ででチェックぐらいはすれば、ある程度は分かると思う。

なお、n が十分大きければ、不成立であることも分かっている。
197132人目の素数さん:2007/11/19(月) 20:00:37
Shapiro の巡回不等式は本当にあの形をしているのか!
むしろ、今まで多くの人に知られずにいたのが不思議なくらいだ。
フェルマー予想のように有名な問題であってよかった筈だが。
198132人目の素数さん:2007/11/23(金) 04:38:52
92 名前:MASUDA ◆5cS5qOgH3M [] 投稿日:2007/11/22(木) 21:22:50
a,bは正の実数,tは正の実数とする.このとき,いかなるa,bに対しても以下の不等式が成り立つようなtの最小値を求めよ.
log√(ab)≦{(a+b)/2}^t

93 名前:132人目の素数さん[sage] 投稿日:2007/11/22(木) 22:15:42
>>92
a=b=e^eとすれば、(左辺)=e,(右辺)=e^(et) となるので、与不等式が成立するためには
et≧1、すなわちt≧1/eでなければならない。
次に、t=1/eで不等式が成立することを示す。
y=logx上の点(e,1)での接線がy=x/eであり、y=logxのグラフが上に凸なので
x/e≧logxが言え、この式からx≧log(x^e)が導かれる。
このときx^e=zとすることでz^(1/e)≧logzとなる……@
また相加相乗平均の不等式から{(a+b)/2}^(1/e)≧(√ab)^(1/e)となる……A
@でz=√abとしてAと組み合わせることで{(a+b)/2}^(1/e)≧log√abが示される。
以上から、求めるtの最小値は1/e。

問題文の左辺をloga+logbとした方が解きづらい問題になりそう。

94 名前:132人目の素数さん[sage] 投稿日:2007/11/22(木) 22:46:51
>92
相加相乗平均より、(左辺) ≦ log((a+b)/2) = log(A),
 >84 の式で y=(1/e)A^t とおく。
 t*log(A) ≦ (1/e)A^t,
与式成立条件は、t≧1/e,
199132人目の素数さん:2007/11/28(水) 05:10:35
高校質問スレで質問したところ、こちらを勧められたので質問させて頂きます。

|a|,|b|,|c|<1のとき、(1)ab+1>a+b(2)abc+2>a+b+cを証明せよ。
という問題があり、この2つはゴリ押しで何とか解けたのですが
4文字以上の場合に繋がるような証明法がどうしても思いつきません。

|a1|,|a2|,・・・,|an|<1のとき、a1・a2・・・an+(n-1)>a1+a2+・・・+an
成り立つかどうかもわからないのですが、わかる方いましたらよろしくお願いします。
200132人目の素数さん:2007/11/28(水) 10:00:00
a(bc)+2>a+bc+1>a+b+c.
201132人目の素数さん:2007/11/28(水) 23:07:42
ありがとうございました。
202132人目の素数さん:2007/12/31(月) 21:24:01
あげ
203132人目の素数さん:2007/12/31(月) 21:40:16
>177
Shapiro’s cyclic inequality for even n
を保存しようとした時、何か変なメッセージ
(Acrobat 8 がどうのこうのと言う)
が出た。そんな物持ってないのに普通に保存出来たが、問題あったかな?
204 【吉】 【371円】 :2008/01/01(火) 17:31:46
今年こそは Shapiroの巡回不等式予想を解くぞ!

って、まだ本当に未解決なのか?
205132人目の素数さん:2008/01/01(火) 18:16:28
ふふ…
206132人目の素数さん:2008/01/03(木) 17:26:12
ただの 乱順序積 ≦ 同順序積 (積分版)だが…

〔FKG不等式〕
 f(x),g(x) を[a,b]上の単調増加(減少)な関数とすると
 ∫[a,b] f(x)dx・∫[a,b] g(y)dy ≦ (b-a)∫[a,b] f(x)g(x)dx,

FKG は C.Fortuin, P.Kasteleyn, J.Ginibre の頭文字らしい…

http://elis.sigmath.es.osaka-u.ac.jp/~aida/lecture/18/double-integral2.pdf
207132人目の素数さん:2008/01/05(土) 19:20:50
(∬_D f (x, y) dxdy)*(∬_D h (x, y) dxdy) ≦ ∬_D f (x, y)*g (x, y) dxdy
208132人目の素数さん:2008/01/05(土) 20:13:19
官軍の同志諸君、ならびに賊軍のあほんだれwに告ぐ:−

御大は、無事、日本に帰られた。飛行機を使われなかったことは確かだ。
ハイテク筏かどうかは不明!

決戦の場は、sci.logic や sci.math だ!!!!

語学力(英語で充分)を磨こう!

目標は、7万語の語彙だ。

"Word Power Made Easy"http://www.amazon.co.jp/s/ref=nb_ss_fb?__mk_ja_JP=%83J%83%5E%83J%83i&url=search-alias%3Denglish-books&field-keywords=Word%81%40Power%81%40Made%81%40Easy&Go.x=17&Go.y=15&Go=Go などを読んでおけ!

尚、同書がきつい者(読みこなせない)者は「試験に出る英単語」を
もう一度とりだして、「完全に」マスターすることから始めよ!

某スレで恩大は、こうおっしゃっているので引用しる:−

Yo(余) ni dekita koto ga soch-tachi ni dekinai wake ga
arouka?!!!!

Onaji mana kutte doko tsugau(違う)!!!!
209132人目の素数さん:2008/01/06(日) 10:32:49
任意の実数x,y,z,nに対して不等式

(x-y)(x-z)x^n + (y-z)(y-x)y^n + (z-x)(z-y)z^n ≧ 0

を証明せよ


これがわかりません
210132人目の素数さん:2008/01/06(日) 10:50:21
あきらか
211132人目の素数さん:2008/01/06(日) 14:07:38
>>209
問題設定おかしくね?
212132人目の素数さん:2008/01/06(日) 18:44:40
>>206 ただの 乱順序積 ≦ 同順序積 (積分版)だが…

それを、普通はチェビシェフの不等式と言う。
積分版も同じ。
FKGかなんかしらんが、チェビシェフの不等式のパクリ。

213132人目の素数さん:2008/01/06(日) 19:11:46
数学の世界で「パクリ」という言葉を初めて聞いた気がする
214132人目の素数さん:2008/01/06(日) 19:40:39
或る人が書いた数学本の中には、
不等式の本といってよいものが存在する。
215132人目の素数さん:2008/01/06(日) 19:49:52
どこの存在定理ですか?
216132人目の素数さん:2008/01/06(日) 20:01:48
>>213
でも定理の系や簡単な応用なのに名前をつけるのは、どうかと思う。

最初にやった人の功績は重要だが、それを統一化された現在では、
変てこな名前を言われるより、「チェビシェフの不等式」と言って
くれた方が十分通じるし、理解も早い。
217132人目の素数さん:2008/01/06(日) 21:47:55
同時期に独立に出したのなら、パクリではないが、
最近は論文数を増やす為のパクリも多い。
218132人目の素数さん:2008/01/16(水) 16:41:27
Shapiro's Cyclic Sum Constant
http://mathworld.wolfram.com/ShapirosCyclicSumConstant.html

Shapiro's Cyclic Inequality is ture for all even n ≦ 12 and
odd n ≦ 23 (Mitrinovic et al. 1993).
219132人目の素数さん:2008/01/16(水) 16:46:14
Journal of Inequalities and Applications
http://www.hindawi.com/journals/jia/contents.html
220132人目の素数さん:2008/01/16(水) 16:53:26
P. J. Bushell and J. B. Mcleod
"Shapiro’s cyclic inequality for even n",
Journal of Inequalities and Applications Volume 7 (2002),
Issue 3, Pages 331-348

http://www.hindawi.com/GetArticle.aspx?doi=10.1155/S1025583402000164

Abstract
In 1954 H. S. Shapiro proposed an inequality for a cyclic sum in n variables.
All the numerical evidence indicates that the inequality is true for even n≤12
and for odd n≤23. We give an analytic proof for the case n=12, which implies
the former result. The remaining case n=23 remains an open problem.
221132人目の素数さん:2008/01/26(土) 23:22:02
〔問題〕
a,b,c は 0≦a,b,c<1 をみたす実数とする.また,
 S = 3(a+b+c+abc)/(1+ab+bc+ca),
 A = (3+a^2)a/(1+3a^2),
 B = (3+b^2)b/(1+3b^2),
 C = (3+c^2)c/(1+3c^2),
と定める。このとき,
 A+B+C ≦ S < 3,
を示せ.(MASDA)

http://science6.2ch.net/test/read.cgi/math/1199706844/155 ,168
東大入試作問者スレ13
222132人目の素数さん:2008/01/26(土) 23:27:05
>221

右側は
 1 - S/3 = 3(1-a)(1-b)(1-c)/(1+ab+bc+ca) >0 より。

左側は
 a = tanhα, b = tanhβ, c = tanhγ とおくと、tanhの加法公式より
 S = 3tanh(α+β+γ),
 A = tanh(3α),
 B = tanh(3β),
 C = tanh(3γ),
∴ tanhθy は θ≧0で上に凸だから、 A+B+C ≦ S.

ここに tanhθ = {e^θ -e^(-θ)}/{e^θ +e^(-θ)},
223132人目の素数さん:2008/01/27(日) 03:00:31
(*゚∀゚)=3 ハァハァ…
224132人目の素数さん:2008/01/28(月) 09:32:39
A(x) = ( a(x)_{ij} ) をn次対称行列で、各成分 a_{ij}(x) は [0,1] 上の連続関数とする。
このとき、次をしめせ。
 det { ∫_[0→1] A(x) dx }^{-1} ≦ ∫_[0→1] { det A(x) }^{-1} dx.

ただし,∫_[0→1] A(x) dx = ( ∫_[0→1] a_{ij}(x) dx ) であり、det は行列式を表す。
225132人目の素数さん:2008/01/28(月) 09:43:25
>>224
訂正:A(x) はn次の「正定値」実対称行列です。
「正定値」がぬけていました。
226132人目の素数さん:2008/01/28(月) 22:54:23
x > 2、y > 2、1/x + 1/y ≦ 1/2 のとき、2x+yの最小値を求めよ。

簡単だからエレガントに頼むぜ、ブラザー!
227132人目の素数さん:2008/01/29(火) 01:19:46
>>226
レポート問題を人にやらせるなよw
228132人目の素数さん:2008/01/29(火) 11:43:38
レポートって…(笑)
高校の問題をレポートに出す大学って、教育学部?
229132人目の素数さん:2008/01/30(水) 14:23:51
>>228
私立の文系(受験科目に数学なし)の選択必修など沢山ある。
文系は高校の微積分も知らないし、私大だと中学の数学(今はゆとり教育で、
以前は中学の数学が今は高校でやるようになった)が怪しい奴が大勢いる。

不等式の両辺に負の数を掛けると、不等号の向きが変るのが分からない奴が
いるから。ゆとり教育はマジでやばい。
230132人目の素数さん:2008/01/30(水) 14:55:30
>>229
分数の計算すらまともにできない大学生が蔓延っている今の日本
不等号の向きがナンチャラカンチャラなんぞ知らない人がいることなんて
別に驚くにあたらないし、今に始まったことでもない

これが今の日本の現状
事実だ!これが現状だ!
目を背けるな!

そして、じゃあはたして僕らはどうしたらいいのだろうと・・・
日々自問自答を繰り返している
231132人目の素数さん:2008/01/30(水) 16:02:14
そんな大学生が居ない大学に行けばいいだけの話だろ
2321stVirtue ◆.NHnubyYck :2008/01/30(水) 16:47:14
そこで 1stVirtue 王国の創設だ。
233132人目の素数さん:2008/01/31(木) 01:23:20
じゃぁ数ヲタ達はエリートだな
234132人目の素数さん:2008/01/31(木) 02:39:38
>>231
私立大の文系ではほとんど入試科目に数学が無いから(最近は推薦やAO入試があるから
理系でもやばいけど)、一部の学生を除いて全然数学を勉強してきていない。
そういう奴らに数学を教えると、まあ易しいのをやれば大丈夫なのだが、そこで必ず
単位を落とすような奴が出てくる。よくよく問い詰めるとそういう奴は>>229>>230
のように中学レベルの数学で落ちこぼれているんだから、救いようが無い。
私立のトップといわれるW大やKOでもそういう奴がいるそうだから、きついよ。
それにこれからもっとゆとり世代が入ってくるから、ガクガクブルブル。
手っ取り早い改善策は、とにかく入試問題に数学を課すことだ。
センターの数学でもいいからさ。
235132人目の素数さん:2008/01/31(木) 08:53:16

nを自然数とするとき
e-(1+1/n)^n<e/(2n+1)
が成り立つことを示せ。
236132人目の素数さん:2008/01/31(木) 11:44:55
平成の時代に不平等は許されません
よって与式は成り立たない
2371stVirtue ◆.NHnubyYck :2008/01/31(木) 18:07:50
不平等を許さないという奴が平成の時代にも居たのか。
238132人目の素数さん:2008/01/31(木) 18:49:06
>>235
見かけによらず意外に難しい…
2391stVirtue ◆.NHnubyYck :2008/01/31(木) 19:03:27
不平等を許さないというやつは、すべての悪人に対しても平等を強いておけ。

Reply:>>235 e*(2*n)/(2*n+1)<(1+1/n)^n.
2401stVirtue ◆.NHnubyYck :2008/01/31(木) 19:17:06
なんとなくレスをつけてみたが、e<(1+1/n)^n*(1+1/(2*n)) をどうやって証明しよう。
241132人目の素数さん:2008/01/31(木) 21:01:57
>>239
人の脳を読む能力を悪用する奴でも?
242132人目の素数さん:2008/01/31(木) 21:52:20
自作問題。

nとMは自然数で、1≦n<Mを満たすとする。Q(n)を次のように定義する。
Q(n)=Π[k=0〜n−1](1−k/M)=(1−0/M)*(1−1/M)*(1−2/M)*…*(1−(n−1)/M)
また、非負の実数cに対して、
a={−(2c−1)+√{(2c−1)^2+8Mc}}/2 , b={1+√{1+8Mc}}/2
とおく。

(1)次を示せ。
・n≧bならばQ(n)≦e^(−c)である
・n≦aならばQ(n)≧e^(−c)である
・0≦b−a≦2cである

(2)nは自然数で、1≦n<365とする。n人の人間のうち、誕生日が一致する
2人がいる確率をP(n)とおく。次を示せ。
・n≧42ならばPn≧1−e^(−2.3) (≒0.9)
・n≦39ならばPn≦1−e^(−2.3)
2431stVirtue ◆.NHnubyYck :2008/02/02(土) 08:21:34
Reply:>>241 思考盗聴で個人の生活に介入する奴を排除するかすべての人が思考盗聴できるようにならないと、平等にはならない。
244132人目の素数さん:2008/02/03(日) 05:05:28
>>235
0 ≦ d < 1 とする。
log(1-d) = -d -(1/2)d^2 -(1/3)d^3 -(1/4)d^4 - ……
     ≦ -d -(1/2)d^2 -(1/4)d^3 -(1/8)d^4 - …… (等比級数)
    = -2d/(2-d),
これに d = 1/(n+1) を代入すると
 -log(1 +1/n) = log(n/(n+1)) ≦ -2/(2n+1),
 n・log(1 +1/n) ≧ 2n/(2n+1) = 1 - 1/(2n+1),
あとは exp( ) するだけ。
 (1 +1/n)^n ≧ e・exp(-1/(2n+1)) ≧ e{1 - 1/(2n+1)},
245132人目の素数さん:2008/02/04(月) 12:15:20
〔235の類題〕
nを自然数とするとき
 e・exp(-1/(2n+2)) > (1+1/n)^n > e・exp(-1/(2n+1)),
が成り立つことを示せ。
246132人目の素数さん:2008/02/04(月) 12:22:50
>245
右側は >244
左側も同様に
log(1-d) = -d -(1/2)d^2 -(1/3)d^3 -(1/4)d^4 - ……
   ≧ -d -(1/2)d^2 -(1/2)d^3 -(1/2)d^4 - …… (等比級数)
   = -d -(d^2)/(2(1-d))
   = -d(2-d)/(2(1-d)),
これに d = 1/(n+1) を代入すると
 -log(1 +1/n) = log(n/(n+1)) > -(2n+1)/(n(2n+2)),
 n・log(1 +1/n) < (2n+1)/(2n+2) = 1 -1/(2n+2),
あとは exp( ) するだけ。
247132人目の素数さん:2008/02/05(火) 11:50:54
〔235の拡張〕
nを自然数とするとき
 e/(2n+2) < e - (1+1/n)^n < e/(2n+1),
が成り立つことを示せ。
248132人目の素数さん:2008/02/05(火) 12:20:13
>247
右側は >244
左側も同様に
log(1-d) = -d -(1/2)d^2 -(1/3)d^3 -(1/4)d^4 -(1/5)d^5 - ……
   > -d -(1/2)d^2 -(1/3)(d^3 +d^4 +d^5 + …… )   (等比級数)
   = -d -(1/2)d^2 -(d^3)/(3(1-d)),
これに d = 1/(n+1) を代入すると
 -log(1 +1/n) = log(1 -1/(n+1))
  > -1/(n+1) -1/(2(n+1)^2) -1/(3n(n+1)^2),
  = -(1/n){1 -1/(2(n+1)) -1/(6(n+1)^2)},
 n・log(1 +1/n) < 1 -1/(2n+2) -2/(3(2n+2)^2)
  ≦ 1 -1/(2n+2) -1/(2(2n+2)(2n+1))      (← 3(2n+2) ≦ 4(2n+1) )
  < 1 + log(1 -1/(2n+2)),           ( >246 の公式に d=1/(2n+2) を代入)
あとは exp( ) するだけ。
 (1 +1/n)^n < e{1 -1/(2n+2)},
249132人目の素数さん:2008/02/05(火) 18:15:11
>>245>>247
同じ問題はいいよ。つまんねえ
250132人目の素数さん:2008/02/06(水) 03:28:54
つまんなくて申し訳ねぇ…

nが大きいとき、マクローリン展開して
 n*log(1 +1/n) = n*{1/n -1/(2n^2) +1/(3n^3) -1/(4n^4) + …}
  = 1 -1/(2n) +1/(3n^2) -1/(4n^3) +1/(5n^4) - …,

 (1 +1/n)^n = e{1 -1/(2n) +11/(24n^2) -21/(48n^3) +2447/(5760n^4) - …},

 {e - (1 +1/n)^n}/e = 1/(2n) -11/(24n^2) +21/(48n^3) -2447/(5760n^4) + …,

 e/{e - (1+1/n)^n} -2n = 2n/{1 -11/(12n) + 21/(24n^2) -2447/(2880n^3) + …} -2n
   = 2n{1 +11/(12n) -5/(144n^2) +17/(1080n^3) -… } -2n
   = 11/6 -5/(72n) + 17/(540n^2) - … → 11/6, (n→∞)

 e - (1+1/n)^n ≒ e/(2n +11/6).   (n>> 1)
251132人目の素数さん:2008/02/06(水) 09:31:57
数学科の微積分での証明だったら、これらは全部不合格だよ。

まず、e の存在を証明して(有界単調数列は収束する)から物事が始まる。
指数関数の定義やその逆関数として log x を定め、さらに、それらが解析的
であること、つまりTaylor展開できることという順番だからな。

ここの「証明」は全部循環論法。
e の不等式の証明にTaylor展開を使うのは、数学科だとアウト。
252132人目の素数さん:2008/02/06(水) 12:27:04
最初からTaylor展開でe^xを定義する事だってよくあるけど。
君が知らないだけで。定義も書かないで251みたいな事を書くのはナンセンス。

まあ>>249には大体同意。
253132人目の素数さん:2008/02/06(水) 23:24:48
>>251
それ、数学科の微積分じゃなくて、高校の微積分って言った方が正しいと思うよw
254132人目の素数さん:2008/02/08(金) 17:59:08
【問題】
f を開区間 (a,b) の C^2 級関数とするとき,次の不等式を示せ.
∫^b_a |f ' (x)|^2 dx ≦ 54 [ 1/(b-a)^2 ∫^b_a |f(x)|^2 dx + (b-a)^2 ∫^b_a |f ' ' (x)|^2 dx ]
255132人目の素数さん:2008/02/08(金) 20:12:14
>251
小平の解析入門なんかでは
無限級数の極限で定義してる。
いろんな定義が可能なことを知らないなんて
数学科ではないなw
256251:2008/02/09(土) 11:19:43
数学科の微積分をナメるなよ!!!!!!!!11111
257132人目の素数さん:2008/02/09(土) 11:40:03
ってか、king氏にも分からないことがあるんだと
かつ、このスレの優秀さを改めて見直した
258132人目の素数さん:2008/02/09(土) 11:42:33

                       _,.-‐"':" ̄~゙'ヽ、       __
      _,---‐" ̄\         /          ``ー‐-、   ノ   \
    /        ヽ      ;"                ) /      \
   /   ぐ わ   |      /                |ノ/        \
  /    ら か     |     |                 )/.|   ・  オ   |
  |    .い ら    |     |          ,;';;,,    /ノ |   ・   レ   |
  |     ・  な    |    |::::.................:::::::::;;,'^;、::::::'''..,,_;、丿 |   ・   に   |
  |     ・  い    |    /:::::::::::::::::::::::::::;"゙, /゙~゙`''::;'゙;     |  ・   だ.  |
  |    あ  こ    |    `、;;::::::::::::::::;/ ),;'   :.'.,、   |  ・   っ  |
  |    る  と    |  ,へノ   `'''''"´   .:;     .:::_ヽ  |  ・   て   |
  |    ・        Y   \       .::;     ::::ゝ    .|  ・         |
  |    ・       ∧    \     ::::::、   .:;`     |         |
  |    ・       |ヽ丶    \;;  :::;;;;::..,,、. ::i       |          |
  |    ・       | `       \;;;;/    `゙"       \
259132人目の素数さん:2008/02/09(土) 18:24:21
>>255
ふ〜ん
じゃあその e^x をTaylor展開で定義する方法で、指数法則
e^{x+y} = e^x e^y
や、三角関数の加法定理
sin (x+y) = sin x cos y + cos x sin y
を証明してみせてよ。
数学科なんだからこのくらいは出来るよね。
260132人目の素数さん:2008/02/09(土) 18:32:33
>>259
どの定義からも他の定義のものが得られることが知られている
その証明はいい練習になるだろうが、本質的でない
本質でないことに拘ることの意味が分からないのですが
261132人目の素数さん:2008/02/09(土) 18:38:52
>>260
へ〜、どの定義から始めるかは大事なことだと思うけどね。
それは個人のスタイルだから、義務ではないけど、その時々に都合良く定義
を変えることは、何も証明をしていないことだね。

どの定義から始めても同等であることの事実は非常に重要なことですけど。
それは、実数の完備性をどの公理を採用するかの問題と似ていますね。
262132人目の素数さん:2008/02/09(土) 18:39:29
>>255
ふ〜ん
じゃあその e^x をTaylor展開で定義する方法で、指数法則
e^{x+y} = e^x e^y
や、三角関数の加法定理
sin (x+y) = sin x cos y + cos x sin y
を証明してみせてよ。
数学科なんだからこのくらいは出来るよね。
263132人目の素数さん:2008/02/09(土) 18:42:47
e^xをどの定義で始めるかは一長一短がある。
お前はそれを知っていて、>>259のようなことを書きやがったな。
その通りだよ、この証明はベキ級数の収束の議論が入るから、非常に面倒だよ。
微分を使ってもいいけど、それには項別微分の可能性を示さなくてはならない。
264132人目の素数さん:2008/02/09(土) 18:51:59
指数法則はe^xが絶対収束することと二項定理から得られる
加法定理はcosとsinがそれぞれe^xを用いて表現できることから得られる

で、こんなの常識でしかないのだが
265132人目の素数さん:2008/02/09(土) 19:16:06
> 加法定理はcosとsinがそれぞれe^xを用いて表現できることから得られる

それは複素変数の場合
一応大学1年のレベルなんだから、複素数を使わないで証明してもらいたいね
266132人目の素数さん:2008/02/09(土) 23:45:00
何言ってるの? 基地外?
267132人目の素数さん:2008/02/10(日) 00:21:54
exp(x)と同じようにやればいいだけなのもわからんのか
268132人目の素数さん:2008/02/10(日) 01:07:41
弧長とか使って厳密にsinとかcosとか定義するのも
それはそれで面倒だと思うけどな。

e^{x+y} = e^x e^y は実際にTaylor展開で証明してる本が
結構あると思うけど。三角函数もちょっと面倒になるだけで基本的には同じ。
(本質的には複素変数にしただけなんだから当然といえば当然)
269132人目の素数さん:2008/02/10(日) 08:46:25
>>268
>>三角函数

か・・・漢字が読めねぇ・・・orz
270132人目の素数さん:2008/02/10(日) 10:12:25
歴史的名著は大抵函数表記だった気がするが
271132人目の素数さん:2008/02/10(日) 14:04:52
>>269
ゆとり世代乙

>>270
収束のことを収斂とか書いていたが、流石に今は直されているだろう
だけど、函数は時々見かける。
あと、個人的には線形代数という表記が気に入らない。
線型代数だろうよと,,,
272132人目の素数さん:2008/02/10(日) 15:07:04
函はハコと読みます。

サンカクハコカズです。
273132人目の素数さん:2008/02/10(日) 15:30:28
>>271
>>収斂

俺も読めない・・・orz
274132人目の素数さん:2008/02/10(日) 15:32:59
>>269
北海道の函館(はこだて)って知らないのか?
275132人目の素数さん:2008/02/10(日) 15:34:01
ゆとり・・・
276132人目の素数さん:2008/02/10(日) 21:42:47
>>273
釣りかも知れんが「しゅうれん」だ。
覚えておけ!
277132人目の素数さん:2008/02/11(月) 15:28:17
〔問題〕
絶対値が1より小さい任意の実数の組(x,y,z)に対して (a,b,c) を
 a = x+y+z
 b = x^2 +y^2 +z^2
 c = x^3 +y^3 +z^3
と定める。下記の不等式が成り立つことを示せ。(MASUDA)
 |a^3 +6a -3ab +2c| < 3|a^2 -b+2|,

http://science6.2ch.net/test/read.cgi/math/1199706844/350 ,359
東大入試作問者スレ13
278132人目の素数さん:2008/02/11(月) 15:43:06
>277
示すべき不等式を整理すると
 | (xyz + x+y+z)/(xy+yz+zx + 1) | < 1,
を示せばよいことがわかる。
問題文に (x,y,z) の絶対値は1より小さい, とある。そこで
>>222 に習って x=tanhξ, y=tanhη, z=tanhζ とおこう。tanh の加法公式より
 (xyz + x+y+z)/(xy+yz+zx + 1) = tanh(ξ+η+ζ),
 | tanh(……) | < 1,
よって、問題の不等式も示される。
279132人目の素数さん:2008/02/11(月) 17:24:24
>278 の補足
 (coshθ)^2 - (sinhθ)^2 = {[e^θ + e^(-θ)]/2}^2 - {[e^θ - e^(-θ)]/2}^2 = 1,
より
 1 - (tanhθ)^2 = 1/(coshθ)^2 >0,
よって
 |tanhθ| < 1,
280132人目の素数さん:2008/02/11(月) 22:53:50
>>269-276
読めない漢字@数学板

三角函数(さんかく・かんすう)→三角関数

収斂(しゅうれん)→数学の用語で収束のこと

帰謬法(きびゅうほう)→背理法ともいう
281132人目の素数さん:2008/02/11(月) 23:01:39
数学板、誤変換

○確率
×確立

○置換
×痴漢

○偏微分
×変微分

○整式
×正式

○小数
×少数

○対数
×大数
(ただし『大学への数学』または"大数の法則"の意の場合も・・・)

○シミュレーション
×シュミレーション
(日本語にない発音のため。ただし方言には近い発音があるらしい)

○キチ(既知)
×ガイチ
(またちなみに、既出(きしゅつ)と読む。"がいしゅつ"ではない。)

既知の既の字に「木」へんが付くと
高木貞治の『解析概論』"かいせき・がいろん"の概の字になる。

       、____,, -―――- 、ヽ 、
       _>           ヽ} )
      /  /   ' /        ⌒ヽ
    ∠(  /  ^メ、 //    }         ',
       ヽ/   { / {{   ハ  } ヽ.  |
.       /    ,ノx=ミ从  / |⌒/   V |
    ∠ -ァフ ,イ〃うハハ/ _ | ∧    { リ
      厶‐'´! } V辷j   ≠弌 〉、   ∨
       V{. ヽゝ    '__    /  \ \
          \个 .  V _) _厶 人ノ ̄
          ^ j人>rー/^}_ ,イノ´   ニホンゴのカンジってムズカシイネ
          xr<了  (`ヽ{ /`ヽ
           / {.  {YY´ ̄ }7   }
            /〃}   } 人_,   j    /
        / {{ {   {{  ヽ. \ /
2831stVirtue ◆.NHnubyYck :2008/02/11(月) 23:33:13
1stVirtue教では応用数学の習得もする。
284132人目の素数さん:2008/02/12(火) 00:49:18
>>283
お前誰だ? 馬鹿じゃねーの?
2851stVirtue ◆.NHnubyYck :2008/02/12(火) 07:26:02
Reply:>>284 日本人の心を持つことをお前様はわかるのだろうか。
286132人目の素数さん:2008/02/12(火) 16:30:11
>>285 は気違いだから相手にするな。


「1stVirtue教」だとさwww
2871stVirtue ◆.NHnubyYck :2008/02/12(火) 16:56:38
Reply:>>286 不心得者は早く日本から去りてくださいませ。
2881stVirtue:2008/02/12(火) 19:01:31
>>287
お前が出て行け!偽者。
2891stVirtue ◆.NHnubyYck :2008/02/12(火) 19:32:57
Reply:>>288 誰が本物であるかの議論をしなくてはならぬのか。
290132人目の素数さん:2008/02/13(水) 21:21:30
>>289
当たり前だろ
それより俺の心を読むのをやめてくれないか
2911stVirtue ◆.NHnubyYck :2008/02/13(水) 23:38:37
Reply:>>290 どうしろという。
292132人目の素数さん:2008/02/14(木) 01:19:01
数学の命題にはそれ自体真か偽かが証明不可能な命題が存在する(ゲーデル)
293132人目の素数さん:2008/02/14(木) 11:34:35
1stVirtue ◆.NHnubyYck

お前邪魔やからさっさと消えろや!
2941stVirtue ◆.NHnubyYck :2008/02/14(木) 11:48:43
Reply:>>293 自分または自分の親戚がよそ者かどうか考えてみよ。
295132人目の素数さん:2008/02/16(土) 22:15:57
>>277
示すべき不等式を整理すると
 | N | < D,
を示せばよいことがわかる。ここに N = xyz + (x+y+z), D = (xy+yz+zx) +1,
問題文に (x,y,z) の絶対値は1より小さい, とある。よって
 D + N = (1+x)(1+y)(1+z) >0,
 D - N = (1-x)(1-y)(1-z) >0,
辺々掛けて
 D^2 - N^2 = (1-x^2)(1-y^2)(1-z^2) >0,
 | N | < D,
296KBumDUXdQj:2008/02/28(木) 11:50:58
297132人目の素数さん:2008/03/08(土) 20:45:38
自然数 n に対し、 a[n] = (1 + 1/n)^n とする。
a[n+1] - a[n] < a[n] / {2 * (n+1)^2}
を示せ。

★東大入試作問者になったつもりのスレ★ 第十四問
http://science6.2ch.net/test/read.cgi/math/1204606214/69

(補注)
n=1 だと左辺=右辺だから n≧2 の誤りだと思われる。
298132人目の素数さん:2008/03/12(水) 00:29:56
同スレからもう一題。

82 :69:2008/03/09(日) 18:11:30
【補題】
x,y>0 のとき
 x^(n+1) - (n+1)x・y^n + n・y^(n+1) ≧0,
 等号成立は x=y のとき。

(略証)
 (左辺) = (x-y)^2・Σ[k=0,n-1] (k+1)・x^(n-k-1)・y^k, より明らか。
 {S_n = 1 + 2r + 3r^2 + … + n・r^(n-1) を求める頻出問題より}

http://science6.2ch.net/test/read.cgi/math/1204606214/82

-------------------------------------------------------

(別証)
 (左辺) = (x-y)S,
ここに
 S = x^n + x^(n-1)・y + …… + x・y^(n-1) - n・y^n = Σ[k=0,n-1] {x^(n-k) - y^(n-k)}・y^k,
とおいた。
x>y>0 のとき S >0,
 (左辺) = (x-y)S >0.
y>x>0 のとき S <0,
 (左辺) = (x-y)S >0. (終)
299132人目の素数さん:2008/03/12(水) 04:31:35
入試レベルの不等式キボンヌ
300132人目の素数さん:2008/03/12(水) 04:34:48
ヘルダーの不等式を証明汁
301132人目の素数さん:2008/03/12(水) 08:50:42
302132人目の素数さん:2008/03/16(日) 22:15:52
同スレからもう一題。

【問題】(改作)
n≧2 とし、n次元Euclid空間を考える。
半径rの超球面(中心は原点にある)と座標軸の交点は2n個ある。
半径r'の超球の内部(超球面を含む)にある点Pから2n個の交点までの距離の積の最大値をもとめよ。

http://science6.2ch.net/test/read.cgi/math/1204606214/162, 165
303132人目の素数さん:2008/03/16(日) 22:23:59
>302
(略解)
各点の座標を
 O = (0,0,…,0),  原点
 A_1 = ( r,0,…,0), A_2 = (0, r,0,…,0), ……, A_n = (0,…,0, r),
 B_1 = (-r,0,…,0), B_2 = (0,-r,0,…,0), ……, B_n = (0,…,0,-r),
 P = (x_1,x_2,…,x_n)
とおく。題意より
 OP = √{(x_1)^2 + (x_2)^2 + … + (x_n)^2} ≦ r'.
Π[i=1,n] A_iP・B_iP の最大値をもとめる。
 (A_iP・B_iP)^2 = {(x_1)^2 + … + (x_(i-1))^2 +(x_i -r)^2 + (x_(i+1))^2 + … + (x_n)^2}
         * {(x_1)^2 + … + (x_(i-1))^2 +(x_i +r)^2 + (x_(i+1))^2 + … + (x_n)^2}
         = (OP^2 + r^2 - 2r・x_i)(OP^2 + r^2 + 2r・x_i)
         = (OP^2 + r^2)^2 - (2r・x_i)^2,
i=1,2,…,n について相加・相乗平均をとる。
 (Π[i=1,n] A_iP・B_iP)^(2/n) ≦ (OP^2 + r^2)^2 - (1/n)(2r・OP)^2 = OP^4 + (2- 4/n)(r・OP)^2 + r^4,
等号成立は |x_1| = |x_2| = … = |x_n| = OP/√n のとき。
題意より OP≦r', n≧2 だから、
 Π[i=1,n] A_iP・B_iP ≦ {(r')^4 + (2- 4/n)(r・r')^2 + r^4}^(n/2),
とくに r'=r のとき
 Π[i=1,n] A_iP・B_iP ≦ {4(1 -1/n)}^(n/2) r^(2n),

(例)
 n=2, r'=r のとき 2r^4,
 n=3, r'=r のとき (8/3)^(3/2) r^6.
304132人目の素数さん:2008/03/16(日) 23:25:16
半径1として考える。超球体の点x=(x1,...,x_n)を
単位ベクトルt=(t1,..,tn)を使ってx=rt (0≦r≦1)と書く。
(距離の積の2乗)
=Π{(r^2+1)^2-4r^2(t_i)^2}
≦{(1/n)Σ((r^2+1)^2-4r^2(t_i)^2)}^n (相加相乗)
={(r^2+1)^2-(4/n)r^2}^n
={(r^2+(1-2/n))^2+1-(1-2/n)^2}^n
よって右辺はr=1で最大となるから
距離の積はr=1, |t1|=...=|tn|(=1/√n)
のとき最大値(4-4/n)^(n/2)
305132人目の素数さん:2008/03/16(日) 23:26:15
リロードしてなかったorz
306132人目の素数さん:2008/03/28(金) 01:59:51
同スレからもう一題…

〔問題244〕(改作)
三角形の三辺をa,b,cとし、外接円の半径をRとおく。このとき次を示せ。
 R ≧ {√(a^2 +b^2 +c^2)}/3, 等号成立は R√3 =a=b=c のとき。

http://science6.2ch.net/test/read.cgi/math/1204606214/244
307132人目の素数さん:2008/03/29(土) 00:24:53
>306
(略解)
 a,b,cに対する頂角をA,B,C とする。
 a^2 + b^2 + c^2 = (-a^2+b^2+c^2) + (a^2-b^2+c^2) + (a^2+b^2-c^2)
   = 2bc・cosA) + 2ca・cos(B) + 2ab・cos(C)    (←第2余弦定理)
   = (8R^2){cos(A)sin(B)sin(C) + sin(A)cos(B)sin(C) + sin(A)sin(B)cos(C)}
   = (8R^2){ -cos(A+B+C) + cos(A)cos(B)cos(C)}        (*)
   = (8R^2){ 1 + cos(A)cos(B)cos(C)}         (← A+B+C=180゚)
   ≦(9R^2).

(*) exp(iA)exp(iB)exp(iC) = exp(i(A+B+C)) の実部をとる。

〔補題〕
 三角形の三頂角をA,B,Cとするとき、cos(A)cos(B)cos(C)≦ 1/8, 等号成立は A=B=C=60゚ のとき。
(略証)
 ・鈍角3角形のときは、残りの2角は90゚未満だから cos(A)cos(B)cos(C) <0,
 ・鋭角3角形のときは、子s(A),cos(B),cos(C)≧0,
  相乗・相加平均と cos(x) が上に凸であることから{または log(cos(x))が上に凸であることから}
  cos(A)cos(B)cos(C) ≦ {[cos(A)+cos(B)+cos(C)]/3}^(1/3) ≦ cos((A+B+C)/3)^3
    = (1/2)^3 = 1/8,           (← A+B+C=180゚)   (終)
308307:2008/03/29(土) 03:10:13
訂正。スマソ。
  cos(A)cos(B)cos(C) ≦ {[cos(A)+cos(B)+cos(C)]/3}^3 ≦ cos((A+B+C)/3)^3
309132人目の素数さん:2008/03/30(日) 16:20:16
相加相乗の不等式をできるだけ多くの方法で証明せよ
310132人目の素数さん:2008/03/30(日) 23:25:44
>>309
君がしたまえ!
311132人目の素数さん:2008/03/31(月) 23:11:01
>>306
[同スレ262]

既に解かれているが別解。

a ≦ b ≦ c として考えてよい。
R = abc/4S  (S は三角形の面積)
 = abc/√{(a+b+c)(a+b-c)(a-b+c)(-a+b+c)}  (∵ヘロンの公式)
 = {√(a^2 + b^2 + c^2)}/3
∴ 9 a^2 b^2 c^2 = (a^2 + b^2 + c^2)(a+b+c)(a+b-c)(a-b+c)(-a+b+c)

0 = 左辺 - 右辺
 = a^6 + b^6 + c^6 + 3 a^2 b^2 c^2 - a^4 b^2 - a^4 c^2 - b^4 a^2 - b^4 c^2 - c^4 a^2 - c^4 b^2
 = a^2 (b^2 - a^2) (c^2 - a^2) + (c^2 - b^2)^2 (c^2 + b^2 - a^2)
 ≧ 0  (∵ a ≦ b ≦ c)
だから等号は成り立っていなければならない。
等号の成立条件は {a = b または a = c} かつ b = c すなわち a = b = c。
このとき R = a/√3。
312132人目の素数さん:2008/05/05(月) 23:07:28
801
313132人目の素数さん:2008/05/06(火) 00:59:34
age
314132人目の素数さん:2008/05/06(火) 17:50:09
>>309

n個の正の数 {a,b,c,…} の相乗平均をGとする。
すべての要素がGに等しい場合を除いて、 a < G < b となるような要素a,bがある。ここで
 a' = G, b' = a・b/G,
と変更しても相乗平均はGのまま。一方、相加平均は
 (G + a・b/G)/2 - (a+b)/2= -(G-a)(b-G)/G <0
より減少する。
この変更操作を繰り返すと、(n-1)回以内にすべての要素がGに等しくなり、相加平均もGになる。
しかし相加平均は減り続けた筈だから、元々の相加平均Aは Gより大きかった。(終)

参考文献[3] の p.71-72
315132人目の素数さん:2008/05/07(水) 00:27:35
0≦x≦1,0≦y≦1,0≦z≦1の範囲で
{(x+y+z)/3}+√{x(1-x)+y(1-y)+z(1-z)}
のとりえる値の最大値を求めよ。
316132人目の素数さん:2008/05/08(木) 09:06:26
半径rの球面上を4点A,B,C,Dが動く.このとき,
  AB↑・AC↑+AC↑・AD↑+AD↑・AB↑
の最小値をrで表せ.
317132人目の素数さん:2008/05/08(木) 11:22:51
>>316
0
318132人目の素数さん:2008/05/10(土) 19:26:12
>>315
(x+y+z)/3 =A の断面で考える。 Σ逆順序積 ≦ Σ乱順序積 より
 x(1-x) + y(1-y) + z(1-z) ≦ (x+y+z)(3-x-y-z)/3 = 3A(1-A),
よって
 (与式) ≦ A + √[3A(1-A)]
 = (3/2) - {(3/2 -A) - √[3A(1-A)] }
 = (3/2) - 4(A -3/4)^2/{(3/2 -A) + √[3A(1-A)]} ≦ 3/2,
等号成立は A=3/4, x=y=z=3/4 のとき。

>>316
球の中心をOとし、OA↑=a↑, OB↑=b↑, OC↑=c↑, OD↑=d↑ とおく。
 (与式) = (b-a)(c-a) + (c-a)(d-a) + (d-a)(b-a)
  = b・c + c・d + d・b -2(a・(b+c+d)) + 3(a・a)
  = (S^2 -b^2 -c^2 -d^2)/2 -2(a・S) + 3(a・a)   (← S=b+c+d)
  = (1/2)(S-2a)^2 + a^2 - (1/2)(b^2 +c^2 +d^2)   (← 平方完成)
  ≧ a^2 - (1/2)(b^2+c^2+d^2)
  = -(1/2)r^2,
等号成立は S = b+c+d = 2a のとき。
 (例えば、 △BCDが正3角形、その重心の方向にAがあり、∠AOB=∠AOC=∠AOD=arccos(2/3).)
319132人目の素数さん:2008/05/14(水) 03:01:17
(゚Д゚)≡゚д゚)、カァー ペッ!!
320132人目の素数さん:2008/05/14(水) 03:01:34
(゚Д゚)≡゚д゚)、カァー ペッ!!
321132人目の素数さん:2008/05/14(水) 03:01:49
(゚Д゚)≡゚д゚)、カァー ペッ!!
322132人目の素数さん:2008/05/14(水) 03:02:03
(゚Д゚)≡゚д゚)、カァー ペッ!!
323132人目の素数さん:2008/05/14(水) 03:46:36
ん?タン虫は4連で終わり?
つまらん!

1000までやりゃいいのに
324132人目の素数さん:2008/05/14(水) 21:44:03
宿題ですが。。解き方が、わかりませんので、教えてください。

不等式2a-1/3<xを満たすxの最小の整数値が4であるとき、整数aの値をすべて求めなさい。
っていう問題です。

3≦2a-1/3<4 を満たす a 
を求めればよい。となっていますが、
5/3≦a<13/6 となり
a=2 と答はなりますが。。

解き方として

2a-1/3<4 は、xに4を代入(最小の整数値は4のため)分かりますが
2a-1/3≧3 がどうして3がでてくるのか分かりません。

機械的に、不等式で最小の整数値と出てきた問題は
   整数値をBとした場合
    B-1≦式<B  と機械式に覚えるのでしょうか。

また、不等式で最大の整数値と出てきた問題は
    整数値をCとした場合
    C<式≦C+1  と機械式に覚えるのでしょうか。

    

325132人目の素数さん:2008/05/14(水) 22:20:06
>>324
2a-1/3<4 は成り立つが
2a-1/3<3 は成立たない。(← 4は最小値)
326132人目の素数さん:2008/05/20(火) 20:20:39
a[1],・・・,a[n]>0 に対し, 不等式
 (a[1]/a[2])+(a[2]/a[3])+・・・+(a[n]/a[1])
 ≧{(a[1]+a[2])/(a[2]+a[3])}+{(a[2]+a[3])/(a[3]+a[4])}
  +・・・+{(a[n]+a[1])/(a[1]+a[2])}
が成立することを証明せよ.
(出典;数学セミナー)
327132人目の素数さん:2008/05/21(水) 00:48:10
ベクトルで…と思ったが、分けわかめ ('A`)
328132人目の素数さん:2008/05/23(金) 15:21:27
>>327
低脳は書き込まないように。
329132人目の素数さん:2008/05/24(土) 00:15:18
330132人目の素数さん:2008/05/24(土) 14:15:32
331132人目の素数さん:2008/05/24(土) 14:16:56
332132人目の素数さん:2008/05/24(土) 21:01:37
>>314
蛇足だが…

その変更操作によって調和平均は
 2ab/(G + ab/G) = 2ab/(a+b-) > 2ab/(a+b),
により増加する。
 ……
しかし調和平均は増え続けた筈だから、元々の調和平均HはGより小さかった。(終)
333132人目の素数さん:2008/05/28(水) 17:29:13
http://science6.2ch.net/test/read.cgi/math/1209732803/79
から転載

nPk < (k! 2^n)/√n が成り立つことを示せ
ただしnPkは順列の個数を意味する
334132人目の素数さん:2008/05/31(土) 20:35:13
>>333

nPk /k! = C(n,k) とおくと、問題の式は C(n,k) < (2^n)/√n,
  C(n,k) = C(n,k-1)*((n+1-k)/k),
より、左辺は k=[n/2] に向かって単調に増加する。
∴ C(n,k) ≦ C(n,[n/2]),

〔補題〕
 C(n,[n/2]) ≦ (2^n)/√(n+2),
 等号成立は n=2 のとき。
(略証)
nについての帰納法による。
n=1,2 のとき成立。
nが偶数のとき、n=2m,
 C(2m,m) = 4{(m -1/2)/m}・C(2m-2,m-1) < 4√{2m/(2m+2)}・C(2m-2,m-1),
 C(2m,m){1/[2^(2m)]}√(2m+2) は単調減少。
 C(2m,m) ≦ {2^(2m)}/√(2m+2),
nが奇数のとき、
 C(2m-1,m) = (1/2)C(2m,m) < {2^(2m-1)}/√(2m+2),   (終)

※ 分母を √(n+2) にすると、間単に出る所がミソ。
335132人目の素数さん:2008/06/04(水) 02:51:52
>>334 の補足

 C(n,k) = n!/{k!(n-k)!} = n!/{(k-1)!(n+1-k)!}*((n+1-k)/k) = C(n,k-1)*((n+1-k)/k),

 (m -1/2) /m < √{2m/(2m+2)},
(略証)
 2m(m^2) - (2m+2)(m -1/2)^2 = 2m^3 -2(m+1)(m^2 -m +1/4) = (3m-1)/2 >0,
336132人目の素数さん:2008/06/08(日) 22:07:04
〔問題83〕(改作)
a,b,c>0 とする.
 a^4 + b^4 + c^4 ≧ (ca)^2 + (ab)^2 + (bc)^2 ≧ abc(a+b+c) ≧ abc{√(bc) + √(ca) + √(ab)} ≧ 3(abc)^(4/3),
を示せ。

東大入試作問者スレ15
http://science6.2ch.net/test/read.cgi/math/1212563635/83

---------------------------------------------------------

(略証)
左端
 (1/2)(a^4 + b^4) ≧ (ab)^2,
 巡回的にたす。
中央左
 (1/2){(ca)^2 + (ab)^2} = (1/2)(a^2)(c^2 + b^2) ≧ (a^2)bc,
 巡回的にたす。
中央右
 (1/2)(b+c) ≧ √(bc),
 巡回的にたす。
右端
 √(bc) + √(ca) + √(ab) ≧ 3{√(bc)√(ca)√(ab)}^(1/3) = 3(abc)^(1/3),
337132人目の素数さん:2008/06/08(日) 23:13:45
ハァハァ…
338132人目の素数さん:2008/06/14(土) 19:02:42
>>326

a[k]/a[k+1] = b[k] とおく。
 (右辺) = Σ_{k=1,n} (a[k] + a[k+1]) / (a[k+1] + a[k+2])
  = Σ_{k=1,n} (b[k] +1) / (1 + 1/b[k+1])
  = Σ_{k=1,n} (b[k] +1) * b[k+1]/(b[k+1] +1)
ここで x/(x+1) = 1 - 1/(x+1) は単調増加ゆえ、チェビシェフ和の不等式から
  ≦Σ_{k=1,n} (b[k] +1) * b[k]/(b[k] +1)
  = Σ_{k=1,n} b[k]
  = (左辺).
ただし、a[n+1]=a[1], a[n+2]=a[2] 等とした。
ぬるぽ

http://mathworld.wolfram.com/ChebyshevSumInequality.html
339132人目の素数さん:2008/06/23(月) 23:58:44
a,b,c を実数,nを自然数としたとき,次の不等式を示せ.

|a+b+c|^{2n/n+1} ≦ 3^{2n/n+1} { |a|^{2n/n+1} + |b|^{2n/n+1} + |c|^{2n/n+1} }
340132人目の素数さん:2008/06/24(火) 01:01:32
>>339 |a+b+c|≦3*max{|a|, |b|, |c|} から明らか。
341132人目の素数さん:2008/06/26(木) 05:30:00
342132人目の素数さん:2008/06/26(木) 05:31:02
343132人目の素数さん:2008/06/28(土) 14:55:09
【問題148】(改作)
 sin(cosθ)、cos(sinθ) の大小を比較せよ。

http://science6.2ch.net/test/read.cgi/math/1212563635/148

---------------------------------------------------
(略解)
・|θ| < π/2 のとき, |sin(x)| ≦ |x| より
 |sin(cosθ)| < |cosθ| = cosθ ≦ cos(sinθ),
・cosθ ≦0 のとき
 -1 ≦ cosθ ≦0,
 sin(cosθ) ≦ 0 < cos(1) ≦ cos(sinθ),
344132人目の素数さん:2008/06/28(土) 21:48:06
>>341
A.435ムズイな…
345132人目の素数さん:2008/06/28(土) 21:58:18
>>341
やさしいのは・・・

B.4019.
  1/(2k+1)^2 < 1/{4k(k+1)} = 1/(4k) - 1/(4(k+1)),
 より
  (左辺) < 1/4 - 1/(4(n+1)) < 1/4.
 なお、真の極限値は (3/4)ζ(2) -1 = (3/4)(π^2)/6 -1 = (π^2)/8 -1 = 0.23370055013617・・・

B.4035. 積和公式
 2cos(kx)sin(x/2) = sin((k+1/2)x) - sin((k-1/2)x),
を使うと
 (左辺) = sin((11/2)x) / sin(x/2),
 x=(2/11)nπ,   (nは整数, 但し11の倍数を除く.)

B.4043.
 (a,b,c,d) = (1,3,5,11) (1,2,8,17)

B.4046.
 (a,b) = (169/9, 196/9)  順不同
 |a-b|=3,
346345:2008/06/28(土) 22:07:34
〔問題〕は↓でつ・・・

B.4019. Prove that
   1/(3^2) + 1/(5^2) + ・・・ + 1/(2n+1)^2 < 1/4,
 for every positive integer n.

B.4035. Solve the following equation:
   2cos(5x) + 2cos(4x) + 2cos(3x) + 2cos(2x) + 2cos(x) + 1 = 0.

B.4043. For what pairwise-different positive integers is the value of
   a/(a+1) + b/(b+1) + c/(c+1) + d/(d+1) = 3, or
   1/(a+1) + 1/(b+1) + 1/(c+1) + 1/(d+1) = 1 ?

B.4046. Solve the following simultaneous equations:
   a√a + b√b = 183,
   a√b + b√a = 182,
347132人目の素数さん:2008/06/29(日) 00:50:11
私のコレクションの中にも無いなぁ…
348132人目の素数さん:2008/06/30(月) 22:51:51
A436とか微妙におもしろいね。不等式ならでは。解析ならでは。
349132人目の素数さん:2008/07/02(水) 01:21:20
中心がOで半径1の円に内接する△ABCがある。
↑OA=↑a、↑OB=↑b、↑OC=↑c
とするとき

↑a・↑b+↑b・↑c+↑c・↑a

の取りうる値の範囲を求めよ。
350132人目の素数さん:2008/07/02(水) 20:57:26
>>341

B.4040.
 a=tan(A/2), b=tan(B/2), c=tan(C/2)    (0<A,B,C<π)
とおく。附帯条件から
 cot((A+B+C)/2) = (1-ab-bc-ca)/(a+b+c-abc) = 0,
 A+B+C = π,
 ABCは三角形をなす。

(1) 鋭角三角形(or直角三角形)のとき
 (左辺) = cos(A) + cos(B) + cos(C) ≦ 3cos((A+B+C)/3)  (← 上に凸)
    = 3cos(π/3) = 3/2.
(2) 鈍角三角形のとき、0<A,B<π/2<C とする。
   (左辺) = cos(A) + cos(B) + cos(C) ≦ 2cos((A+B)/2) + cos(C) (← 上に凸)
    = 2sin(C/2) + cos(C) = 1 +2sin(C/2) -2sin(C/2)^2
    = √2 - 2{sin(C/2) -(1/√2)}{sin(C/2) -1 +(1/√2)} < √2  (← sin(C/2) > 1/√2)
351350:2008/07/02(水) 21:18:08
〔問題〕は↓でつ・・・

B.4040.
 a,b,c are positive real numbers, and ab+bc+ca=1. Prove that
 (1-a^2)/(1+a^2) + (1-b^2)/(1+b^2) + (1-c^2)/(1+c^2) ≦ 3/2.
352132人目の素数さん:2008/07/03(木) 01:31:20
>>351
ab+bc+ca=1 をみたす正の数 a、b、c に対して、
  1/(1+a^2) + 1/(1+b^2) + 1/(1+c^2) ≦ 9/4
を示せばよい。

s = a+b+c、t = ab+bc+ca、u = abc とおくと、 t=1、u > 0 より、
  (右辺)-(左辺) = (s-9u)(s-u)/{4(1+a^2)(1+b^2)(1+c^2)} ≧ 0
等号成立条件は a=b=c.

なぜならばっ! なぜならばっ!
  s-u > s-9u = st-9u ≧ 0 (←相加相乗平均の関係)

蛇足、t=1 より
  s-u = st-u = (a+b)(b+c)(c+a) > 0
  s-9u = st-9u = c(a-b)^2 + a(b-c)^2 + b(c-a)^2 ≧ 0

          ___  
    |┃三 ./  ≧ \
    |┃   |::::  \ ./ | 久々の出番だね!
    |┃ ≡|::::: (● (● |  不等式と聞ゐちゃぁ
____.|ミ\_ヽ::::... .ワ......ノ     黙っちゃゐられねゑ…
    |┃=__    \         ハ,ァハァ、ハァハァ、ハァハァ…
    |┃ ≡ )  人 \ ガラッ
353350-351:2008/07/03(木) 23:28:16
>>352
 成る程。 >>350 は牛刀だったか・・・orz.

>>349
 (与式) = ↑a・↑b + ↑b・↑c + ↑c・↑a = {|↑a + ↑b + ↑c|^2 - |a↑|^2 - |b↑|^2 - |c↑|^2 }/2
 = {|↑a + ↑b + ↑c|^2 -3}/2,
ここに
 0 ≦ |↑a + ↑b + ↑c| ≦ 3,
 -3/2 ≦ (与式) ≦ 3,
等号成立は (左側 :ABCが正三角形のとき, 右側 : A=B=C のとき)
ハァ ハァ

>>350
354132人目の素数さん:2008/07/03(木) 23:58:49
>>353
牛刀も大歓迎です! (*゚∀゚)=3 ハァハァ…
なぜならばっ! なぜならばっ!
不等式ヲタだからです!
別解が多いほど興奮するからです!
355132人目の素数さん:2008/07/04(金) 23:51:52
B.4101.
Assume xyz=8. Prove that
1/√(1+x^2) + 1/√(1+y^2) + 1/√(1+z^2) ≧ 1,

不等式スレッド 143-157

IMO-2001 (USA) Problem 2 の類題らしいよ。
http://imo.wolfram.com/problemset/index.html
356132人目の素数さん:2008/07/05(土) 04:38:10
>>355
解けたけど芋のもんだいよりは簡単だった。
357132人目の素数さん:2008/07/06(日) 10:42:24
>>341 , >>355 念のため・・・

B.4101.
 a=k/(x^(3/2)), b=k/(y^(3/2)), c=k/(z^(3/2)), (k>0) とおくと
x^2 = 4bc/a^2, y^2 = 4ca/b^2, z^2 = 4ab/c^2,
 (左辺) = a/√(a^2 + 4bc) + b/√(b^2 + 4ca) + c/√(c^2 + 4ab)
   ≧ a/√{a^2 +(b+c)^2} + b/√{b^2 +(c+a)^2} + c/√{c^2 +(a+b)^2} (← 相乗相加平均)
   > a/(a+b+c) + b/(a+b+c) + c/(a+b+c)
   = 1.
358357:2008/07/06(日) 10:49:06
>357 の訂正、スマソ

 a=k/(x^(2/3)), b=k/(y^(2/3)), c=k/(z^(2/3)), (k>0) とおくと
359132人目の素数さん:2008/07/09(水) 17:25:53
誰かA.435解いて〜
360132人目の素数さん:2008/07/09(水) 17:27:45
★東大入試作問者になったつもりのスレ★ 第十五問
http://science6.2ch.net/test/read.cgi/math/1212563635/231

nは自然数とする
{Σ[k=0→2n](C[2n,k])}/{Σ[k=0→n](C[n,k])^2}≦2√n
を示せ
361132人目の素数さん:2008/07/10(木) 00:11:21
バーゼル不等式を自力で見つけた俺は普通より上
362132人目の素数さん:2008/07/10(木) 21:42:46
A435
s1=a+b+c,s2=ab+bc+ca,s3=abc
S1*S2/S3≧6{a/(s1-a)+b/(s1-b)+c/(s1-c)}
363132人目の素数さん:2008/07/10(木) 21:45:10
a/(s1-a)+b/(s1-b)+c/(s1-c)={a(s1-b)(s1-c)+b(s1-c)(s1-a)+c(s1-a)(s1-b)}/{(s1-a)(s1-b)(s1-c)}
364132人目の素数さん:2008/07/10(木) 21:50:42
a(s1-b)(s1-c)+b(s1-c)(s1-a)+c(s1-a)(s1-b)
=(a+b+c)s1^2-(ab+ac+bc+ab+ca+bc)S1-3abc
=S1^3-S1*S2-3*S3
365132人目の素数さん:2008/07/10(木) 21:53:15
>>362-364
証明になっとらん
366132人目の素数さん:2008/07/10(木) 21:53:25
a(s1-b)(s1-c)+b(s1-c)(s1-a)+c(s1-a)(s1-b)
=(a+b+c)s1^2-(ab+ac+bc+ab+ca+bc)S1+3abc
=S1^3-2*S1*S2+3*S3

(s1-a)(s1-b)(s1-c)=S1^3-(a+b+c)S1^2+(ab+bc+ca)S1-abc
=S1*S2-S3
367132人目の素数さん:2008/07/10(木) 21:57:14
>>366
続き教えてください
368132人目の素数さん:2008/07/10(木) 21:58:19
S1*S2/S3≧6{S1^3-2*S1*S2+3*S3}/{S1*S2-S3}

S1*S2*{S1*S2-S3}-6*S3*{S1^3-2*S1*S2+3*S3}≧0
369132人目の素数さん:2008/07/10(木) 22:02:04
-6*S3*S1^3+S2^2*S1^2+13*S2*S3*S1-18*S3^2≧0
370132人目の素数さん:2008/07/10(木) 22:08:15
>>369
それが常に成り立つことの証明は?
371132人目の素数さん:2008/07/10(木) 23:05:37
>>360

 (分子) = Σ[k=0→2n] C[2n,k] = (1+1)^(2n) = 2^(2n),
 (分母) = Σ[k=0→n] (C[n,k])^2 = Σ[k=0→n] C[n,k]・C[n,n-k] = C[n+n,n],
より
 (左辺) = {2^(2n)}/C[2n,n] = b[n]
とおく。
 b[1] = 2 = √(2n),
 b[n]/b[n-1] = 4*(n*n)/[(2n)(2n-1)] = n/(n - 1/2)
  = √{n*n/(n - 1/2)(n - 1/2)}
  = √{n/(n-1)} * √{(n-1)n/[(n-1)n + (1/4)]}
  < √{n/(n-1)}.
∴ b[n]/√(2n) は単調減少。
なお、
 b[n]/√(2n) → (√π)/2,    (n→∞)

http://science6.2ch.net/test/read.cgi/math/1212563635/239
372371:2008/07/10(木) 23:08:51
 b[1] = 2 = 2√n,
 b[n]/b[n-1] = 4*(n*n)/[(2n)(2n-1)] = n/(n - 1/2)
  = √{n*n/(n - 1/2)(n - 1/2)}
  = √{n/(n-1)} * √{(n-1)n/[(n-1)n + (1/4)]}
  < √{n/(n-1)}.
∴ b[n]/(2√n) は単調減少。
なお、
 b[n]/(2√n) → √(π/2),    (n→∞)
373132人目の素数さん:2008/07/11(金) 10:14:37
a,b,cは0より大きく1/2より小さい実数でa+b+c=1を満たすとする。このとき
(7a-1)/(a-a^2)+(7b-1)/(b-b^2)+(7c-1)/(c-c^2)≦18
374132人目の素数さん:2008/07/11(金) 10:15:47
>>373
0<a,b,c≦1/2 で考えてください。m(_ _)m
375132人目の素数さん:2008/07/13(日) 11:09:47
∫[-1,1]|x^2+ax+b|dx≧1/2
を示せ
376132人目の素数さん:2008/07/13(日) 18:12:00
今までで一番綺麗だと思った不等式は何ですか
377132人目の素数さん:2008/07/13(日) 20:49:27
>376
シュワルツの不等式
378132人目の素数さん:2008/07/14(月) 21:51:14
>>375
(ア) [-1,1] 内に x^2 +ax+b =0 の2根がない場合は
 a ,bを適当に動かすことによって [-1,1]の全域にわたり |x^2 +ax +b| を減少させることが可能(証略)。

(イ) [-1,1] 内に x^2 +ax +b =0 の2根がある場合 -1≦α≦β≦1 と置いて積分を実行!
 (左辺) = ∫_[-1,α] (α-x)(β-x)dx + ∫_[α,β] (x-α)(β-x)dx + ∫_[β,1] (x-α)(x-β)dx
   = {(1/6)(3β-α)α^2 + b - (1/2)a + (1/3)} + (1/6)(β-α)^3 + {(1/6)(β-3α)β^2 + b + (1/2)a + (1/3)}
   = (1/3)(β-α)^3 + 2b + 2/3            (αβ≧0 のときは 明らかに ≧2/3)
   = (1/3)(β-α)^3 -(1/2)(β-α)^2 + 1/6 + (1/2)a^2 + 1/2 (← 以下、α≦0≦β とした.)
   = (1/3)(β-α +1/2)(β-α-1)^2 + (1/2)a^2 + 1/2
   ≧ 1/2.
等号成立は β-α=1 かつ α+β= -a =0、すなわち α=-1/2, β=1/2 のとき. (終)

いくら何でもマンドクセ?
379132人目の素数さん:2008/07/14(月) 22:40:32
>>342
B.4097.
 (x,y) = (6,2), (50,10), (294,42).

>>377
 さようなら。
 http://science6.2ch.net/test/read.cgi/math/1110615777/

>>378    (← 注釈無用)
380379:2008/07/14(月) 22:44:37
〔問題〕は↓でつ・・・

B.4097.
Solve the following equation on the set of integers:
   2^((x-y)/y) − (3/2)y = 1.
381132人目の素数さん:2008/07/14(月) 23:29:34
>>380
そういや、まだ考えてなかった…
382132人目の素数さん:2008/07/15(火) 13:27:24
Jensenの不等式で
f(t_1・x_1+…+t_n・x_n)<=t_1・f(x_1)+…+t_n・f(x_n)
が証明されて、特に
t_1=…=t_=1/n
とおけば
f((x_1+…+x_n)/n)<=(f(x_1)+…+f(x_n))/n
ですが、
t_1+…+t_n=1
の場合を示さないで、直接
f((x_1+…+x_n)/n)<=(f(x_1)+…+f(x_n))/n
を示すことは可能なんですかね?
383132人目の素数さん:2008/07/16(水) 23:32:04
入試問題でも貼ろうか?
384132人目の素数さん:2008/07/16(水) 23:36:21
不等式ならドンと来い
385132人目の素数さん:2008/07/16(水) 23:59:00
2S|x^2+ax+b|>2S|(1+a)x^2+b|>2S|x^2+b|>2S|x^2|>2/3|x|>2/3>1/2
386132人目の素数さん:2008/07/17(木) 01:05:11
>>380
x,yは整数でy≠0よりx≠0、さらに2^(x/y)は整数よりy|xかつx≧y≧1
あとはゴリ押しで、任意の正整数nに対して
x=(2/3)(2n+1)((2^n)-1)((2^n)+1)、y=(2/3)((2^n)-1)((2^n)+1)
が求める整数解となる

不等式とか関係ない気がするが
387132人目の素数さん:2008/07/17(木) 02:16:55
a≧0,b≧0,c≧0,a+b≧cのとき,次の不等式が成り立つことを証明せよ
{a/(1+a)}+{b/(1+b)}≧c/(1+c)
(53群馬大,59中部工大)


2数a,bがa>0,b>0,a+b=1を満たすとき,
{a+(1/a)}^2+{b+(1/b)}^2≧25/2
を証明せよ (52茨城大)


3つの正数x,y,zがx+y+z=1をみたすとき,不等式
{2+(1/x)}{2+(1/y)}{2+(1+z)}≧125
が成り立つことを示せ (58東京女大・数理)


nは25以上の定数,x,y,zは負でない整数で,x+y+z=25のとき
{1−(x/n)}{1−(y/n)}{1−(z+n)}
の最大値を求めよ (58東京理科大)

暇潰しにもならないと思うがどうぞ
388132人目の素数さん:2008/07/17(木) 02:20:48
訂正
nは25以上の定数,x,y,zは負でない整数で,x+y+z=25のとき
{1−(x/n)}{1−(y/n)}{1−(z/n)}
の最大値を求めよ (58東京理科大)
389132人目の素数さん:2008/07/17(木) 03:40:53
フハハハハ…、解ける、解けるぞ!
390132人目の素数さん:2008/07/17(木) 07:02:42
(1-25/3n)^3
2*2.5^2=2*5^2/2>25/2
2(c/2+2/c)^2=(4+c^2)^2/2c^2>c/(c+1)

 2^((x-y)/y) − (3/2)y = 1
2^(x/y-1)=1+(3/2)y=(2+3y)/2
2^(x/y)=2+3y
(x/y)log2=log(2+3y)
xlog2=ylog(2+3y)
x=ylog(2+3y)/log2

log(2+3y)=klog2
2+3y=2^k
y=(2^k-2)/3=2(2^(k-1)-1)/3
2^(k-1)-1=3m
k-1=log(3m+1)/log2
k=log(3m+1)/log2+1
y=(2^k-2)/3=2(2e^log(3m+1)-2)/3=2(2(3m+1)-2)/3=4m
x=ylog(2+3y)/log2=4mlog(2+12m)/log2

391132人目の素数さん:2008/07/17(木) 22:10:13
私にも解けますた…

>>387
(1) a/(1+a) + b/(1+b) ≧ a/(1+a+b) + b/(1+a+b) = (a+b)/(1+a+b) ≧ c/(1+c).   {← x/(1+x) は単調増加}
  ∵ (a+b)(1+c) - (1+a+b)c = (a+b) - c ≧ 0.

(2) a+b=s, b-a=d とおくと
 (左辺) = {a+(1/a)}^2 + {b+(1/b)}^2
  = (a^2 + b^2) + {(1/a)^2 + (1/b)^2} + 4
  = (a^2 + b^2){1 + (1/ab)^2} + 4
  = (1/2)(s^2 + d^2){1 + 16/(s^2 - d^2)^2} + 4
これは d^2 について単調増加。d=0 のとき最小値
 (1/2)(s^2){1 + (2/s)^4} + 4 = 2{(s/2)+(2/s)}^2.
 (別法) f(x) = {x + (1/x)}^2 = x^2 + 2 + 1/(x^2) は下に凸だから
  f(a) + f(b) ≧ 2f((a+b)/2) = 2f(s/2) = 2{(s/2)+(2/s)}^2.

(3) 基本対称式を x+y+z =s, xy+yz+zx =t, xyz =u とおく。
 (xy+yz+zx)/(xyz) = t/u ≧ 3*(3/s),
 (x+y+z)/(xyz) = s/u ≧ 3*(3/s)^2,
 1/(xyz) = 1/u ≧ (3/s)^3,
 (左辺) = {2+(1/x)}{2+(1/y)}{2+(1/z)} = {8xyz + 4(xy+yz+zx) + 2(x+y+z) + 1}/(xyz)
  = 8 + 4(xy+yz+zx)/(xyz) + 2(x+y+z)/(xyz) + 1/(xyz)
  ≧8 + 12*(3/s) + 6*(3/s)^2 + (3/s)^3
  = {2 + (3/s)}^3

>>388
(4) 相乗相加平均より
 (与式) ≦ {1 - (x+y+z)/(3n)}^3 = {1 - s/(3n)}^3.
392132人目の素数さん:2008/07/18(金) 19:46:07
とりあえずIMO

'08 2
(1)x,y,z∈R-{1}, xyz=1のとき, x^2/(x-1)^2+y^2/(y-1)^2+z^2/(z-1)^2≧1を示せ。
(2)(1)で、等号が成立する有理数の組(x,y,z)が無限に存在することを示せ。
393132人目の素数さん:2008/07/19(土) 00:09:16
>>392
(1) 基本対称式を x+y+z =s, xy+yz+zx =t, xyz =u とおく。
 {x(y-1)(z-1)}^2 + {(x-1)y(z-1)}^2 + {(x-1)(y-1)z}^2 - {(x-1)(y-1)(z-1)}^2
 = 3u^2 -4tu +2t^2 -2(st-3u) + (s^2 -2t) - (u-t+s-1)^2
= (t-3)^2 + 2(u-1)(u-t-s+5),
 = (t-3)^2           (← 題意より u=1)
 ≧ 0,
これを {(x-1)(y-1)(z-1)}^2 >0 で割る。

(2) 等号条件は t=3, u=1. なので・・・
394132人目の素数さん:2008/07/20(日) 08:47:38
とりあえず、>>373-374が解ければA.435が解けることが分かった。
395132人目の素数さん:2008/07/20(日) 15:21:54
そろそろ、3文字の対称式に対する不等式に対して一般的解法をここへ載せてもいい時期ではないか?
表立って現れて来ない条件は3文字が実数を現すって事だけで、後は問題の条件が付け加われば
もう、解法に使える条件式は本質的にはないのだから、後はその式が少々煩雑で
一般的にはそれほどよく目にしないってだけの話なんだから、、、。
そうして、文字の次数をあげれば、それはそれで一般的な理論への道が開けている。

そうして、事はそれほどには単純明快にはならないだろうが、、、。
ここらへんから先には幾何や代数もからんで来て数学を学んで視野や地平線を広げていくのには
格好の話題になると思う。

別に不等式だけからでも、数学の全分野に近い範囲を見渡す事だってできると思うし、
そいつは結構おもしろい旅路だと思うよ。
396132人目の素数さん:2008/07/21(月) 00:10:03
>>395
さあ!
397132人目の素数さん:2008/07/23(水) 01:01:13
a,b,c≧1のとき、次の不等式を証明せよ。
4(a+b+c)≧(1+a)(1+b)(1+c)
・・・なんか間違ってるような気がするんだが、
どのようなものと間違えたか心当たりある人いる?
398132人目の素数さん:2008/07/23(水) 01:38:49
>>397
a = b = c = 2 のとき左辺 = 24、右辺 = 27 にて不成立
399132人目の素数さん:2008/07/24(木) 23:12:44
頭悪いので次の式が成り立つのが分かりません。
C>1、nが正の整数であるとき、C<=(1+((C-1)/n))^n
教えてください。引き算と割り算のどちらからも
数学的帰納法をうまく使えませんでした。
400132人目の素数さん:2008/07/24(木) 23:21:20
数式部分を書き直すと、
C>1で、nが正の整数であるとき、
C≦(1+((C-1)/n))^n
401132人目の素数さん:2008/07/24(木) 23:31:57
>>399
x ≧ 0 について (1 + x)^n = 1 + n x + ... ≧ 1 + n x なので
x = (C - 1)/n を代入するだけ。
402399:2008/07/25(金) 11:47:10
早速のご回答ありがとうございます。リロードするのを忘れて
レスが付いたのに気づくのが遅れました。2項定理でしたか。
なるほど。ありがとう。
403132人目の素数さん:2008/07/26(土) 18:28:48
図書館に
[2] 不等式,大関信雄・青木雅計,槇書店,1967年(絶版)
あったから借りてきた
受験参考書っぽくてよさげ
404132人目の素数さん:2008/07/26(土) 23:38:37
>>403
すばらしい!
くれ!
405132人目の素数さん:2008/07/26(土) 23:42:40
どこの大学にもあるんじゃね?
うちの大学にもあるが数学科が借りてるらしい
406132人目の素数さん:2008/07/27(日) 18:03:41
うちの大学って言われても……
407132人目の素数さん:2008/07/28(月) 03:43:15
復刊希望ンヌ!
408132人目の素数さん:2008/07/30(水) 23:03:59
他スレから1題・・・

〔問題396〕実数 a,b,c が条件
 (a-b)^3 + (b-c)^3 + (c-a)^3 = 60,
を満たすとき、
 S = |a-b| + |b-c| + |c-a| の最大値と最小値を求めよ。

http://science6.2ch.net/test/read.cgi/math/1212563635/396 ,442
409132人目の素数さん:2008/07/30(水) 23:07:48
>>408

(略解)
 b-a=x, c-b=y, a-c=z とおく。x+y+z =0,
∴ ≧0 のものと ≦0 のものがある。
題意より (a-b)^3 + (b-c)^3 + (c-a)^3 = 3(a-b)(b-c)(c-a) = -3xyz > 0,
{x,y,z} の2つは正、1つが負である。
x,y>0>z としてもよい。(x,y)-平面の第一象限で考える。
 (a-b)^3 + (b-c)^3 + (c-a)^3 = -3xyz = 3xy(x+y),

(最小値)
軸を45゚回して
 S/(√8) = (x+y)/√2, 
 d = (x-y)/√2,
とおくと、
 3xy(x+y) = (3/√2){(1/8)S^2 - d^2}S,
題意より、
 0 ≦ d^2 = (1/8)S^2 - 80/S = F(S),   (F は単調増加函数)
 S ≧ 4・10^(1/3),
 等号成立は x = y = -z/2 = 10^(1/3), またはその rotation のとき。

(最大値) なし
 x→∞ のとき、0 < y < 20/x^2 →0, S=2(x+y) →∞.
410132人目の素数さん:2008/08/02(土) 12:58:58
π^4+π^5<e^6を示せ
411132人目の素数さん:2008/08/02(土) 13:39:34
グーグルで計算したら殆ど同じだった。
でも少しだけe^6が大きかった。
これはただの偶然か?
412132人目の素数さん:2008/08/02(土) 14:24:28
これだけ近い値だと近似して示すのは無理ぽいな
なんかうまい方法があるのかな
413132人目の素数さん:2008/08/02(土) 23:05:28
π^4 + π^5 ≒ e^6 は有名な近似式でつ ( ゚∀゚) テヘッ
414132人目の素数さん:2008/08/03(日) 02:29:53
スゲーΣ(0д0`ノ)ノ
誰がこんな近似思いついたんだ!
415132人目の素数さん:2008/08/03(日) 02:49:01
持ってる本で一つ、>>410関連のことをごく短く書いてあるのがあったな

「数のエッセイ」(一松信、ちくま学芸文庫)
のP.236(ただし、俺の持ってるのは第二刷発行のものなので、それ以外はわからん)で、
文庫本編のエッセイに対する補足説明の部分の一文なのだが、


――――もっとおもしろいのは
π^4 + π^5 = 403.4267758… ≒ e^6 = 403.4267934…
であろう。
最後の式(↑の式のこと)はカナダでT^3(電卓研究会)が開かれたとき(2003年)、
現地の年配の女性教員から教えられた。


と書いてあった
他にも何かの本で見たような気がするが、ちょっと思い出せないな
416132人目の素数さん:2008/08/03(日) 04:36:00
別にすごくないでしょ。
eやπでの近似を考えて掛けたり割ったりしていれば
e^6をπで割っていってe^6/π^4を見れば気づく。
417132人目の素数さん:2008/08/03(日) 05:33:41
>>416
じゃあ、eとπだけを使って
π^4+π^5≒e^6
みたいに小さい数で見た目のキレイな誤差0.0001以下の近似を作ってみな
どうせ作れないから
418132人目の素数さん:2008/08/03(日) 06:39:02
>>417
 20 + π - e^π ≒ 0.0009000208 1052423273 3557015330 9555・・・
 e^6 - π^5 - π^4 ≒ 0.0000176734 5123210920 5537811247 561872・・・
419132人目の素数さん:2008/08/03(日) 12:07:27
>>418
「e π 整数」
とかで検索すると上の式の載ってるHPがいくつか出てくる。
それと、下の式をそんな自信満々に貼られても……移項しただけじゃん。


そろそろ不等式に戻ろうか。
420132人目の素数さん:2008/08/04(月) 00:32:46
近似式がすごいのか、思いつくのがすごいのか、どっちだ?
421132人目の素数さん:2008/08/04(月) 00:39:08
よく見つけるなーとは思う
422132人目の素数さん:2008/08/04(月) 00:53:21
係数と定数を無視すれば、e1項とπ2項では50乗くらいまでの中で一番良い近似っぽいね
これにて終了
423132人目の素数さん:2008/08/04(月) 01:34:21
結局
π^4+π^5<e^6
を示すのは電卓使わないと無理でOK?
424132人目の素数さん:2008/08/04(月) 07:08:35
>>423
ゼータ関数ζ(3)を求める事くらい難しいと思うからOK
425132人目の素数さん:2008/08/04(月) 07:26:18
>>423
その不等式を数値計算しないで示せというのが数検であったからOKじゃない
426132人目の素数さん:2008/08/04(月) 19:46:28
>>425 kwsk
427132人目の素数さん:2008/08/05(火) 19:33:24
3段の問題だな
428132人目の素数さん:2008/08/06(水) 00:32:34
>>427
もっとくわしく! (;´ρ`) ハァハァ
429132人目の素数さん:2008/08/06(水) 07:51:39
>>415
情報グッジョブ!
430132人目の素数さん:2008/08/06(水) 10:09:16
431132人目の素数さん:2008/08/06(水) 16:29:09
>回答締切 平成20年9月12日(当日消印有効)

ここに解答書いたらまずいってことか。
ここにいる猛者なら誰かは出来たであろう
432132人目の素数さん:2008/08/06(水) 19:03:52
x>1に対しd/dx (1 - √(ln(x)/x))^(√(xln(x))<0を示せ.
433132人目の素数さん:2008/08/06(水) 20:48:07
(√2)+(√3)-π>0
であることをなるべく数値計算をせずに示せ

一応、答は用意してある
434132人目の素数さん:2008/08/09(土) 08:07:33
東大スレに不等式がらみのが沢山あるね
435132人目の素数さん:2008/08/09(土) 18:35:34
分かスレにもある・・・

〔問題823〕
曲面Q: (x/a)^r + (y/b)^r + (z/c)^r = 1 (a,b,c,r>0)と
平面P: ax + by + cy = 0 がある。

平面Pに平行で曲面Qに接する平面P'の式と接点の座標を a,b,c,r で表わせ。

http://science6.2ch.net/test/read.cgi/math/1217024032/823 874
436132人目の素数さん:2008/08/09(土) 23:11:58
面白スレに三角比の(;´ρ`) ハァハァ問題があるね
437132人目の素数さん:2008/08/18(月) 16:38:21
相加相乗平均に新証明法 高校教諭、運転中にひらめく
http://www.asahi.com/science/update/0816/OSK200808160004.html

高校の数学で習う定理の新しい証明法を県立倉敷古城池高校教諭の内田康晴さん(49)が見つけ、オーストラリアの数学専門誌に論文が掲載された。
「高校の教育現場から論文投稿はもっと増えていい。励みになるだろう」と数学者からも喝采の声が上がっている。

証明したのは「相加相乗平均の定理」。高校1年で習うことが多い。

内田さんは、ある定理の証明で描いていた図形が、相加相乗平均の定理の証明に使えることに気づいた。さらに簡単な証明法がないかと連日、考えていたところ、
出勤途中の運転中にひらめいた。高校入学後すぐに扱う簡単な公式を使うだけの方法だった。
438132人目の素数さん:2008/08/18(月) 16:40:34
>「この証明方法に気づいた人はこれまでにもいたはず。
>簡単すぎるので発表済みと思ったのかもしれない」と謙遜(けんそん)する
分かってる先生だな。

>>437の証明法ってどういう証明かご存知の方いらっしゃいますか?
439132人目の素数さん:2008/08/18(月) 16:42:33
440132人目の素数さん:2008/08/18(月) 17:11:56
萌え死にそうでつ
(*゚∀゚)=3 ハァハァ…
441132人目の素数さん:2008/08/18(月) 17:20:19
ありがとう。
442132人目の素数さん:2008/08/18(月) 17:32:14
443132人目の素数さん:2008/08/18(月) 22:13:17
>>439
あんまり簡単だとは思えないな。
444132人目の素数さん:2008/08/18(月) 22:22:40
まぁあれだ!
不等式があれば、あと10年は戦える!
445132人目の素数さん:2008/08/18(月) 22:28:24
実数a,b,cに対してf(x)=ax^2+bx+cとする.このとき

∫[-1,1](1-x^2){f'(x)}^2dx≦6∫[-1,1]{f(x)}^2dx

であることを示せ. (08京大文系)
446132人目の素数さん:2008/08/18(月) 23:27:08
>>445
x∈[-1,1] のとき 1-x^2 < 6 だから自明?
447132人目の素数さん:2008/08/18(月) 23:33:48
∫[-1,1](1-x^2){f’(x)}^2dx≦6∫[-1,1]{f(x)}^2dx
448446:2008/08/18(月) 23:41:22
>>447
thanks
' が見えなかった
449132人目の素数さん:2008/08/19(火) 01:04:57
>>439
昔すう折りの本で見た子とあるぞ
450132人目の素数さん:2008/08/19(火) 08:32:28
>>437
相加相乗平均に新証明法 県立高校教諭、運転中にひらめく
http://namidame.2ch.net/test/read.cgi/news/1219015315/
451132人目の素数さん:2008/08/21(木) 00:38:27
>>433
用意しといた解答書いておく

sin(π/12) > (π/12) - (1/6)(π/12)^3
tan(π/12) > (π/12) + (1/3)(π/12)^3
より
8sin(π/12) + 4tan(π/12) > π  …(*)

一方
sin(π/12) = (√6 - √2)/4
tan(π/12) = 2 - √3
より
√2 + √3 - (8sin(π/12) + 4tan(π/12))
= √2 + √3 - (2√6 - 2√2 + 8 - 4√3)
= (√2-1)^2 (2-√3)^2 (√3-√2)
これは明らかに正なので
√2 + √3 > 8sin(π/12) + 4tan(π/12)  …(**)

(*)(**) より
√2 + √3 > π
452132人目の素数さん:2008/08/21(木) 00:43:24
>>451
神過ぎる!
453132人目の素数さん:2008/08/21(木) 01:53:34
>>452
thx
今、こんなの見つけた
ttp://www2.ocn.ne.jp/~mizuryu/toukou2/toukou56.html
簡単な証明あったら、カッコ悪いと思ったけど、大丈夫だった
454132人目の素数さん:2008/08/21(木) 03:13:45
>>451

sin(π/12) > (π/12) - (1/6)(π/12)^3
tan(π/12) > (π/12) + (1/3)(π/12)^3

これはどこから沸いてきた。
455132人目の素数さん:2008/08/21(木) 04:03:37
3 次のTaylor展開を用いた不等式でしょ。
456132人目の素数さん:2008/08/21(木) 18:36:37
k∈N、t≠0 のとき、

|(d^k/dt^k) exp{-1/(t^2)}|≦ {(2^k・k! / (|t|^k)} exp{-4/(81t^2)}

を示せ。
457132人目の素数さん:2008/08/21(木) 22:17:40
>>445
 f(x) = ax^2 +bx +c,
 f '(x) = 2ax +b,
を与式の両辺に代入する。
奇数次の項は積分すれば0である(計算するには及ばない)。
(左辺) = ∫[-1,1](1-x^2){(2ax)^2 + b^2}dx = (16/15)a^2 + (4/3)b^2,
(右辺) = 6∫[-1,1]{(a^2)x^4 +(2ac+b^2)x^2 +c^2}dx = (12/5)a^2 + 4(2ac+b^2) + 12c^2
   = (16/15)a^2 + 4b^2 + (4/3)(a+3c)^2
   ≧ (16/15)a^2 + 4b^2,
458132人目の素数さん:2008/08/21(木) 22:39:41
0<θ<π/4のとき不等式
(cosθ)^(cosθ)>(sinθ)^(sinθ)
を示せ。
459132人目の素数さん:2008/08/21(木) 23:30:30
>>458
この範囲において
cosθ>sinθ
よって不等式は明らか
460132人目の素数さん:2008/08/22(金) 01:40:53
x1,x2,x3,a1,a2,a3は実数。
x1≧0,x2≧0,x3≧0、
a1+a2≧0,a2+a3≧0,a1+a3≧0とする。
x1+x2+x3=1のとき、
a1x1+a2x2+a3x3≧a1(x1)^2+a2(x2)^2+a3(x3)^2を示せ。
(2)正の実数x,yに対し
√x+√y≦k√(2x+y)が成り立つような実数kの最小値を点と直線の距離公式を用いて求めよ。
461132人目の素数さん:2008/08/22(金) 07:16:47
>>460
学校の宿題は自分で考えましょうね
462132人目の素数さん:2008/08/22(金) 07:30:01
>>459
x^xは単調増加ではない。
463132人目の素数さん:2008/08/22(金) 11:48:30
>>462
ですよね
464132人目の素数さん:2008/08/22(金) 17:44:47
cosxと(sinx)^tanxのグラフを書いて・・・
力技過ぎるか
465132人目の素数さん:2008/08/22(金) 18:17:06
対数とって考えてみるか・・・
466132人目の素数さん:2008/08/22(金) 21:43:06
f(θ) =log (cosθ)^(cosθ) -log (sinθ)^(sinθ)
を微分したら単調性は自明。
多分 0 < θ < π/4 の条件はもっと弱くできると思う。
467466:2008/08/22(金) 21:45:36
最後の1行は勘違い。
468466:2008/08/22(金) 21:49:54
全部勘違いだ〜
469132人目の素数さん:2008/08/23(土) 00:41:31
>>460(1)解説よろ
470132人目の素数さん:2008/08/23(土) 07:00:51
>>458

0≦t≦1 とする。
 f(t) = (1/2)log(1+t^2) + log(1/√2)・t,
とおくと
 f(0) = f(1) = 0,
 f "(t) = (1-t^2)/(1+t^2)^2 ≧0,
 f(t) ≦ 0            (0≦t≦1)
t=tanθ とおいて
 log(cosθ) ≧ tanθ・log(1/√2),
cos(x) >0 を掛けて
 cosθ・log(cosθ) ≧ sinθ・log(1/√2) ≧ sinθ・log(sinθ), (0<θ≦π/4)
471132人目の素数さん:2008/08/23(土) 07:36:55
>>460(1) ,>>469
(左辺) - (右辺) = a1・x1(1-x1) + a2・x2(1-x2) + a3・x3(1-x3)
  = a1・x1(x2+x3) + a2・x2(x3+x1) + a3・x3(x1+x2)
  = (a1+a2)x1・x2 + (a2+a3)x2・x3 +(a3+a1)x3・x1 ≧0,

>>460(2)
 2x=u^2, y=v^2 とおく。
 (k^2)(u^2 + v^2) - (u/√2 + v)^2 = (k^2 -1/2)u^2 -(√2)uv + (k^2 -1)v^2,
が常に≧0である条件は相異なる2実根をもたないこと。
判別式 D' ≦0,
 D' = 1/2 - (k^2 -1/2)(k^2-1) = (k^2)(3/2 - k^2),
 k ≧ √(3/2),
472132人目の素数さん:2008/08/23(土) 09:16:09
>>470
見事な攻撃だ、たけちゃんまん。
f(t)≧g(t) を示すより、f(t)≧h(t) かつ h(t)≧g(t) を示す方が簡単な h(t) を作ってくる眼力には脱毛だぁ
473132人目の素数さん:2008/08/23(土) 21:54:54
>>454

 f(x) = sin(x) - x + (1/6)x^3,
とおくと
 f '"(x) = -cos(x) +1 ≧ 0,
これと f "(0) =0 から,
 x・f "(x) = x{-sin(x) +x} > 0,
これと f '(0) =0 から,
 f '(x) = cos(x) -1 +(1/2)x^2 > 0,
これと f(0) =0 から,
 x・f(x) > 0,

 {tan(x) - x} ' = 1/cos(x)^2 -1 = tan(x)^2 > 0,
と tan(0) -0 =0 から
 x・{tan(x)-x} > 0,  (|x|<π/2)

 g(x) = tan(x) -x -(1/3)x^3
とおくと
 g '(x) = 1/cos(x)^2 -1 -x^2 = tan(x)^2 - x^2 >0  (|x|<π/2)
これと g(0) =0 から
 x・g(x) >0,
474132人目の素数さん:2008/08/23(土) 23:55:48
>>473
マクローリンの 3次+剰余項 で自明でないの。
475132人目の素数さん:2008/08/24(日) 00:10:03
>>474
ええじゃん。
476132人目の素数さん:2008/08/24(日) 00:14:25
誰か>>456をお願いします
477132人目の素数さん:2008/08/24(日) 03:29:33
通約可能って意味を教えてくんなませ
はーでぃの本でいきなりつまったwww
478132人目の素数さん:2008/08/25(月) 01:52:02
>>477
ネタ?
何ページ?
479132人目の素数さん:2008/08/26(火) 22:46:19
>>456って右辺は {(2^k・k! / (|t|^k)} exp{-4/(9t^2)} にならないかな?
勿論 exp{-4/(9t^2)} ≦ exp{-4/(81t^2)} なんだけど。
おらの勘違い?
480132人目の素数さん:2008/08/26(火) 23:05:21
>>477
通約可能ってのは例えば 4/6 が = 2/3 と直せたり
(x^2 - 1)/(x^2 + 2x - 3) が
= [(x + 1)(x - 1)]/[(x - 1)(x + 3)] = (x + 1)/(x + 3) と直せたりするように
分母分子に共通の因子があることだけど。

そういう意味で言ってるの?
もしかして不等式の専門書では通約可能という言葉を
難しい意味で使うのかな、とも思ってレス控えてたけど。

Hardyの本って「不等式」のこと?もしかして「数論入門」のほう?
481132人目の素数さん:2008/08/27(水) 07:48:32
★東大入試作問者になったつもりのスレ★ 第十五問
http://science6.2ch.net/test/read.cgi/math/1212563635/937-945 より。


937 :132人目の素数さん:2008/08/26(火) 19:55:34
a,b,cをabc=1を満たす正の実数とする。次の不等式を示せ。
 (a-1+1/b)(b-1+1/c)(c-1+1/a)≦1

945 :132人目の素数さん:2008/08/26(火) 22:40:11
 abc=1 とあるから当然、 a=y/z, b=z/x, c=x/y と置くんだろうな。
(左辺) = {(y-z+x)/z}{(z-x+y)/x}{(x-y+z)/y}

題意より、-x+y+z, x-y+z, x+y-z のいづれか2つの和は正だから、
 正でないのは高々1つだけ。
・1つが正でない場合、明らかに成り立つ。
・3つとも正の場合、 相乗・相加平均より
 √{(x-y+z)(y-z+x)} = √{x^2 - (y-z)^2} ≦ x,
 √{(y-z+x)(z-x+y)} = √{y^2 - (z-x)^2} ≦ y,
 √{(z-x+y)(x-y+z)} = √{z^2 - (x-y)^2} ≦ z,
辺々掛けて
 (x-y+z)(y-z+x)(z-x+y) ≦ xyz,
482132人目の素数さん:2008/08/30(土) 00:54:44
>>478, 480さん
レスありがとうございます。
「不等式」のほうです
例えば、p15に「通約可能なウエイト付き平均」と「通約可能でないウエイト付き平均」の
話が出てきます

「また、通約可能でないウエイト付き平均は、通常の平均の中のある種の極限とみなす」
などと書いてあります。


それと、話は違うのですが、これは益田塾のなかの問題ですが、
abc=1(a,b,cは正)で、abcについての対称式である不等式を
a=x/y, b=y/z, c=z/xとおいて一般性を失わず、として書き換えて解く論法っていうのは
一般的な方法なんですか?
483132人目の素数さん:2008/08/30(土) 00:56:27
うわwww 偶然だげと481とかぶった。481で引用されているのが益田塾サイトにある
問題とその解答ですね。
484132人目の素数さん:2008/08/30(土) 01:12:12
ちょっと長めに引用しておきます

「通常の平均はすでに述べたようにウエイト付き平均の特別な場合である。
一方、通約可能なウエイトつき平均は, 通常の平均に帰着することができる。
実際、同次性によりウエイトはすべて整数であるとしてよく、さらに(ry」
これ読むと、ウエイトが有理数って意味なんかなと(深く考えずに)思っていたんですが
ググってもよくわからずで。
485132人目の素数さん:2008/09/06(土) 06:57:59
>>456,476

f(t) = exp{-1/(t^2)} とおく。
f^(k)(t) = {(2/t^3)^k - 3・2^(k-2)・k(k-1)/t^(3k-2) + ・・・・・ + (-1)^k (k-1)(k+6)(k+1)!/[12t^(k+4))] + (-1)^(k-1) (k+1)!/t^(k+2)} exp{-1/(t^2)}
  = {(2/t^2)^k - (3/2)k(k-1) (2/t^2)^(k-1) + ・・・・・・ + (-1)^k (k-1)(k+6)(k+1)!/(12t^4) + (-1)^(k-1) (k+1)!/t^2} (1/t^k) exp{-1/(t^2)}
  = P_k(1/t^2) (1/t^k) exp{-1/(t^2)},
ここに P_k はk次の多項式で
 P_k(x) = (2x)^k -(3/2)k(k-1) (2x)^(k-1) + ・・・・・・・・ + (-1)^k [(k-1)(k+6)(k+1)!/12] x^2 + (-1)^(k-1) (k+1)! x,
ところで、
f^(k)(t) (t^k) exp{a/(t^2)} = P_k(1/t^2) exp{-(1-a)/(t^2)} = P_k(x) exp{-(1-a)x},    (a=4/81 or 4/9)
これの絶対値が (2^k)(k!) 以下であることを示す。

〔補題〕 a<1, j>0 ならば
 (x^j)exp{-(1-a)x} ≦ {j/[(1-a)e]}^j,    等号成立は x=j/(1-a) のとき。
486132人目の素数さん:2008/09/06(土) 15:45:53
>>485
流石不等式スレ、恐れ入ります。
実は元ネタがあって、コーシーの積分公式より

(d^k/dt^k) exp{-1/(t^2)}=(k!/2πi)∫[exp{-1/(z^2)}/(z-t)^(k+1)] dz
487132人目の素数さん:2008/09/06(土) 18:52:13
1<cosA+cosB+cosC≦3/2
を示す巧い方法ありますかね?
488132人目の素数さん:2008/09/06(土) 18:56:09
>>487
成り立たないだろ
489132人目の素数さん:2008/09/06(土) 20:16:27
記号から考えて、A≧0,B≧0,C≧0,A+B+C=πが仮定されているのではなかろうか。
凸不等式とか使えばなんとかなるんじゃね。
490132人目の素数さん:2008/09/06(土) 20:47:20
(0,π)でcos xは凸関数でも凹関数でもないからなあ。
とりあえず、A≧B≧Cを仮定して、
f(B,C)=cosB+cosC-cos(B+C)を、0<C≦π/3、C≦B≦(π-C)/2の範囲で
偏微分でゴリゴリやれば、示せるが。
491132人目の素数さん:2008/09/06(土) 21:03:58
それはウマい方法じゃないだろw
492132人目の素数さん:2008/09/07(日) 01:12:43
A+B+C=π π>A≧B≧C>0 として
cos(A)+cos(B)+cos(C)-1
=cos(A)+cos(B)-cos(B+C)-1
=2*cos((A+B)/2)*cos((A-B)/2)-2*cos^2((A+B)/2)
=2*cos((A+B)/2)*{cos((A-B)/2)-cos((A+B)/2)}
=2*sin((A+B)/2)*{(-2)*sin(A/2)*sin(-B/2)}
=4*sin(A/2)*sin(B/2)*sin(C/2)>0

log(cos(A)+cos(B)+cos(C)-1)
=log(4)+log(sin(A/2))+log(sin(B/2))+log(sin(C/2))
≦log(4)+3*log( sin( (A/2+B/2+C/2)/3 ) ) (∵log(sin(x)) は0<x<π/2で上に凸)
=log(4)+3*log(sin(π/6))=log(1/2)

log(x)の単調増加性から cos(A)+cos(B)+cos(C)-1≦1/2
以上から 1<cos(A)+cos(B)+cos(C)≦3/2
493132人目の素数さん:2008/09/07(日) 01:15:39
>>487
cosA+cosB+cosC=4sin(A/2)sin(B/2)sin(C/2)+1 が成り立つ[*]ので,
示すべき不等式は 0<sin(A/2)sin(B/2)sin(C/2) ≦ 1/8 と同値。
sin(A/2)>0などより,左側の不等号は明らか。

右側は,まずは相加相乗平均により
sin(A/2)sin(B/2)sin(C/2) ≦ ( { sin(A/2)+sin(B/2)+sin(C/2) }/3 )^3
さらに,凸不等式より
{ sin(A/2)+sin(B/2)+sin(C/2) }/3 ≦ sin((A+B+C)/6) = 1/2
なので,sin(A/2)sin(B/2)sin(C/2) ≦ (1/2)^3 = 1/8 となり示せた。

[*]の証明は,C=π-(A+B)を左辺に代入して和積,倍角公式で変形するだけ。
494132人目の素数さん:2008/09/07(日) 13:37:35
半径1の円の周上に3点A,B,Cをとる
↑AB・↑ACの最大値,最小値を求めよ
495132人目の素数さん:2008/09/07(日) 13:46:11
>>487
三角形ABCの内接円の半径をr、外接円の半径をとすればR
cosA+cosB+cosC=1+r/R
で、R≧2rはすぐ示せるから与不等式も示される
496132人目の素数さん:2008/09/08(月) 01:06:12
>>494
それ、ハイ理にあったような
497132人目の素数さん:2008/09/08(月) 01:15:31
>>496
ハイ理とは何ぞや?

>>495
cosA+cosB+cosC=1+r/R はどうやってだすの?
498132人目の素数さん:2008/09/08(月) 02:04:14
>>497
ハイレベル理系数学という大学受験参考書の一つでつ
ところで、R≧2rを一番簡単に示す方法はなんでしょ?
499132人目の素数さん:2008/09/08(月) 02:31:50
>>498
( ゚∀゚)テヘッ、呼んだ?
私のコレクションには4通りの解法が汚い字でメモってあるけど、
久しぶりなので、自分の走り書きが理解できない秘密 ('A;;;,,...

----------------------------------------------------------
(1) ヘロンの公式に …(←走り書きなので読み取れない)を用いた後、
  S = abc/(4R) を用いる

(2) 示すべき不等式を基本対称式を用いて表してから頑張る!
  R = u/(4S)、r = 2S/s、16S^2 = s(-s^3+4st-8u)

(3) チャップル・オイラーの定理を用いる

(4) 示すべき不等式を sinA、sinB、sinC で表してから頑張る!
----------------------------------------------------------

私は初代不等式スレで自作自演していた一人です。
主に収拾&出題担当でしたが…

その頃には、書き込んだ不等式を片っ端から証明する不等式神がいますた
それらの証明は、その神に託します

不等式にハァハァしたいのに、雑事が多すぎて時間が取れない我が糞人生… ('A;;;,,...
500132人目の素数さん:2008/09/08(月) 03:12:31
>>495の式が一番明瞭の気がするけど、r/R示すのにまたワンステップ踏まないといけないのか

>>499
なんか各辺の中点を通る円でのキカ的な証明があった気がする
501132人目の素数さん:2008/09/08(月) 07:55:04
>>498
外心をO、内心をIとするとに
OI=√(R^2-2Rr)
となることを幾何学的に示す
502132人目の素数さん:2008/09/08(月) 18:39:01
>>501
それ、チャップル・オイラー
503132人目の素数さん:2008/09/10(水) 02:33:32
ヴィルティンガーの不等式
504132人目の素数さん:2008/09/10(水) 02:38:27
有名どころでヘルダーの不等式。
505132人目の素数さん:2008/09/10(水) 02:53:30
>>503
聞いたことないのは不勉強?
506132人目の素数さん:2008/09/10(水) 03:09:02
ディルレヴァンガーの方程式

>>505
ヴィルティンガーつったらspellはwirtingerだろうからぐぐろうぜ
507132人目の素数さん:2008/09/11(木) 17:32:26
ぐぐったら聞いたことがあることになる?
508132人目の素数さん:2008/09/11(木) 21:01:53
聴覚を使わない限り聞いたことにはならないと思う。
ぐぐった結果、動画ファイルなどを見つけて聞いたのならOK
509132人目の素数さん:2008/09/11(木) 21:12:16
>>508
何その詭弁
空気読めないねって良く言われるだろ
510132人目の素数さん:2008/09/11(木) 21:54:23
>>509
死ねw
511132人目の素数さん:2008/09/11(木) 21:57:09
独学した内容は聞いたことないってのも珍しくないがな
聞いたことはないが知ってるってやつ
512132人目の素数さん:2008/09/11(木) 21:58:30
>>508
KY1級
513132人目の素数さん:2008/09/13(土) 16:23:44
不等式ではないですが・・・
θ=(360/11)°の時(1/cosθ)+(1/cos2θ)+(1/cos3θ)+(1/cos4θ)+(1/cos5θ)の値を求めよ
お願いします
514132人目の素数さん:2008/09/13(土) 16:49:42
なぜスレチとわかってて...
515132人目の素数さん:2008/09/13(土) 17:38:47
マルチと見た
516132人目の素数さん:2008/09/13(土) 18:31:11
>>513
ヒント:2倍して1を足せ
517132人目の素数さん:2008/09/14(日) 07:01:32
>>513

次の恒等式を考える。  (11倍角公式)
 cos(11t) = T_11(cos(t)),
ここに T_11(x) = 1024x^11 -2816x^9 +2816x^7 -1232x^5 +220x^3 -11x,
T_11(x) -1 = (x-1)(32x^5 +16x^4 -32x^3 -12x^2 +6x+1)^2 = (x-1)p(x)^2,
∴ cos(θ), cos(2θ), cos(3θ), cos(4θ), cos(5θ) は T_11(x)-1=0, x≠1 の根、すなわち p(x)=0 の根。
∴ 1/cos(θ), 1/cos(2θ), 1/cos(3θ), 1/cos(4θ), 1/cos(5θ) は p(1/t)=0 の根。
 (t^5)p(1/t) = t^5 +6t^4 -12t^3 -32t^2 +16t +32,
根と係数の関係より、
 (与式) = -6.
518132人目の素数さん:2008/09/15(月) 11:17:09
>498

>>121-122
519132人目の素数さん:2008/09/15(月) 12:15:13
>>497
左辺に 第二余弦定理 cos(A) = (b^2 +c^2 -a^2)/(2bc), etc. を代入してゴリゴリ計算する.

 (左辺) = 1 + (-a+b+c)(a-b+c)(a+b-c)/(2abc) = 1 + 4(s-a)(s-b)(s-c)/(abc) = 1 + 4(S^2)/(abcs) = 1 + (r/R) = (右辺),

ここで、s=(a+b+c)/2, Sは△ABCの面積, r=S/s, R=abc/(4S) を使った。
520132人目の素数さん:2008/09/15(月) 12:43:00
>>487
中辺に 第二余弦定理 cos(A) = (b^2 +c^2 -a^2)/(2bc), etc. を代入して計算すると
 (中辺) = 1 + (-a+b+c)(a-b+c)(a+b-c)/(2abc),
ここで、
 √{( a-b+c)(a+b-c)} = √{a^2 -(b-c)^2} ≦ a,
 √{(-a+b+c)(a+b-c)} = √{b^2 -(c-a)^2} ≦ b,
 √{(-a+b+c)(a-b+c)} = √{c^2 -(a-b)^2} ≦ c,
辺々掛けて
 (-a+b+c)(a-b+c)(a+b-c) ≦ abc,
521519:2008/09/15(月) 21:24:35
>>497
 ヘロンの公式も使った。
 s = (a+b+c)/2 とおくと、S = √{s(s-a)(s-b)(s-c)},

http://ja.wikipedia.org/wiki/ヘロンの公式
http://mathworld.wolfram.com/HeronsFormula.html
遠山 啓, 数学セミナー, √{(三辺の和の半)×(同−第一辺)(同−第二辺)(同−第三辺)} (1977)
宮沢賢治, 和賀郡二子村・花巻農学校 齋藤貞一あて 封書 (1927)
522132人目の素数さん:2008/09/15(月) 22:19:32
∫[0→1]dx/(1+x^2*e^x)>1/(e-1)


x+y+z=1,x>0,y>0,z>0のとき
(x^x)(y^y)(z^z)≧1/3


△ABCの内部に点Pをとり,△ABCの面積をSとおけば
PA+PB+PC≧2(3S^2)^(1/4)
523132人目の素数さん:2008/09/15(月) 22:49:30
>>521
>>ヘロンの公式
どっかの馬鹿が勝手につけた名前は重要ではない
524132人目の素数さん:2008/09/16(火) 15:30:46
For distinct real numbers $a,\ b,\ c,\ d,\ e,\ f$ such that $a < b < c < d < e < f$ and $abcdef = - 6$, let
\[
M = (a^{12} + 58a^8 + 193a^4 + 36)(b^{12} + 58b^8 + 193b^4 + 36)(c^{12} + 58c^8 + 193c^4 + 36)(d^{12} + 58d^8 + 193d^4 + 36)(e^{12} + 58e^8 + 193e^4 + 36)(f^{12} + 58f^8 + 193f^4 + 36)
\]

\[
N = 64(a^8 + 12a^4 + 11)(b^8 + 12b^4 + 11)(c^8 + 12c^4 + 11)(d^8 + 12d^4 + 1)(e^8 + 12e^4 + 11)(f^8 + 12f^4 + 11)
\]
.

Prove the following inequality.
\[
\sqrt [8]{\frac {M}{N}}\geq 6.
\]
525132人目の素数さん:2008/09/16(火) 21:58:34
>>523
そりゃ定理名や式の名前は重要ではないけど、
名前が付いてなきゃ呼びづらいだろ。
526132人目の素数さん:2008/09/17(水) 00:56:06
>>522

(上) コーシーの不等式より
 ∫[0→1] {1+(x^2)(e^x)}dx * ∫[0→1] 1/{1+(x^2)(e^x)} dx ≧ {∫[0→1] dx}^2 = 1,
 ∫[0→1] {1+(x^2)(e^x)}dx = [ x + (x^2 -2x+2)(e^x) ](0→1) = e-1,

(中)
 f(x) = x・log(x) とおく。
 f "(x) = 1/x >0 だから、fは下に凸。
 f(x) + f(y) + f(z) ≧ 3f((x+y+z)/3) = 3f(1/3) = log(1/3).
この真数をとる。
527132人目の素数さん:2008/09/17(水) 22:00:51
f’(x)≧0とする
∫[-1→1]{f(x)/√(1+x^2)}dx≧0
528132人目の素数さん:2008/09/17(水) 22:34:51
質問です
(0<a<b、X、Y、Zはいずれもa以上b以下であるー)「X+Y+Z=a+2b⇒XYZ≧ab^2」を示せ
529132人目の素数さん:2008/09/18(木) 07:08:59
>>522 (下)

(Toth の証明)
 Pの辺BC,CA,ABに関する対称点をA',B',C'とし、6辺形AC'BA'CB'を考える。
 周長L=2(AP+BP+CP), 面積F=2S,

 一方、等周問題から、n辺形については、L^2 ≧ {4n*tan(π/n)}F,
 n=6 のとき L^2 ≧ (8√3)F, これに代入。

文献[3] 例題9, p.17 (1987)
大関・青柳「不等式」p.162
530132人目の素数さん:2008/09/18(木) 09:25:58
http://www.thehcmr.org/issue2_1/problems.pdf

S08-4 が不等式の手強い問題。
既に応募締切は過ぎてるけど…。


531132人目の素数さん:2008/09/18(木) 22:03:27
というか寧ろ締切り過ぎてない問題は晒しちゃダメだろうw
532132人目の素数さん:2008/09/18(木) 22:04:08
>>522
(上) の別解
e^x ≦ e < 3 より
 (左辺) > ∫[0→1] 1/(1+3x^2) dx = (1/√3)∫[0→√3] 1/(1+y^2) dy = (1/√3)arctan(√3) = π/(3√3) > 3/5,
一方、e > 2 + 2/3 より
 (右辺) < 3/5.




>>528
 XY - b(X+Y-b) = (b-X)(b-Y) ≧ 0,  ・・・・ (1)
 Z(a+b-Z) - ab = (b-Z)(Z-a) ≧ 0,  ・・・・ (2)
 X+Y+Z -a -2b = 0,          ・・・・ (3)
(1)*Z + (2)*b + (3)*bZ より
 XYZ - ab^2 ≧ 0.
等号成立は (X,Y,Z)=(a,b,b) (b,a,b) (b,b,a) のとき。
533132人目の素数さん:2008/09/18(木) 22:29:42
そう言えば上の数検の3段の問題は締め切りすぎてそろそろ回答できたやつもいるのかな。
534132人目の素数さん:2008/09/18(木) 22:58:03
今気付いたけどこのスレのURL0がいっぱい並んでて綺麗
535132人目の素数さん:2008/09/19(金) 01:11:37
>>528 2008 千葉大の問題でした。回答速報の答えはへたくそ。
実際、誘導つきなんですがねえ。
536132人目の素数さん:2008/09/19(金) 02:50:44
n個の正の実数x[1],x[2],,,,x[n]がx[1]+x[2]+・・・+x[n]=1を満たすとき
不等式
{x[1]}^2+{x[2]}^2+・・・+{x[n]}^2<{-1+x[1]*x[2]+x[2]*x[3]+x[3]*x[4]+・・・+x[n-1]*x[n]+x[n]*x[1]}^2
を示せ。
537536:2008/09/19(金) 02:55:48
n> 2です
538132人目の素数さん:2008/09/19(金) 11:19:16
king ↔ うんち
(ab)^½ ≤ (a²+b²)/2
(a²+b²+c²)(x²+y²+z²) ≥ (ax+by+cz)²
539132人目の素数さん:2008/09/19(金) 11:20:41
荒らそうと必死ですね わかります
540132人目の素数さん:2008/09/19(金) 11:42:06
>>535-537
(-1+納j=1,n]x[j]x[j+1])^2 (x[n+1]=x[1])
=1-2納j=1,n]x[j]x[j+1]+(納j=1,n]x[j]x[j+1])^2
=(納j=1,n]x[j])^2-2納j=1,n]x[j]x[j+1]+(納j=1,n]x[j]x[j+1])^2
=納j=1,n]x[j]^2+2納1≦i<j-1≦n]x[i]x[j]+(納j=1,n]x[j]x[j+1])^2
>納j=1,n]x[j]^2
541132人目の素数さん:2008/09/19(金) 11:45:52
↑なんか凄い見難いし、安価ミスってるし、狽フ記法もいい加減だけど、なんかこんな感じだと思う。
542132人目の素数さん:2008/09/19(金) 11:46:41
なんか²
543KingMind ◆KWqQaULLTg :2008/09/19(金) 12:30:28
Reply:>>538 お前は何をたくらんでいる。
544132人目の素数さん:2008/09/19(金) 14:54:59
>>540
> =(納j=1,n]x[j])^2-2納j=1,n]x[j]x[j+1]+(納j=1,n]x[j]x[j+1])^2
> =納j=1,n]x[j]^2+2納1≦i<j-1≦n]x[i]x[j]+(納j=1,n]x[j]x[j+1])^2

これはどういう変形?
545132人目の素数さん:2008/09/19(金) 16:20:24
>>544
(納j=1,n]x[j])^2 を展開しただけ
546132人目の素数さん:2008/09/19(金) 17:52:03
king ≤ うんこ
547132人目の素数さん:2008/09/19(金) 18:29:25
荒らそうと必死ですね わかります
548KingMind ◆KWqQaULLTg :2008/09/19(金) 21:06:37
Reply:>>546 何をしている。
549132人目の素数さん:2008/09/20(土) 03:45:57
>>536
 x[1] + x[2] + ・・・ + x[n] = s,
 Σ[1≦i<j≦n] x[i]*x[j] = t,
とおくと
 s = 1,
 納j=1,n]x[j]*x[j+1] ≦ t,
だから
 (左辺) = s^2 -2t = 1-2t,
 (右辺) > (1-t)^2,
よって成立。
550132人目の素数さん:2008/09/20(土) 08:07:35
本質的に>>540=>>549
551551蓬莱:2008/09/20(土) 23:51:56
552132人目の素数さん:2008/09/21(日) 07:57:45
>>533
3段の方が4段より難しかったな。
553132人目の素数さん:2008/09/25(木) 20:27:12
〔問題620〕
全ての自然数nについて
 n*log(n) -(n-1) ≦ log(n!) ≦ (n+1)log(n) -(n-1),
が成り立つことを証明せよ。
http://science6.2ch.net/test/read.cgi/math/1209732803/620

(略証)
 左辺を a_n, 右辺を b_n とおく。
nについての帰納法による。

 log(1!)=0 より a_1 = log(1!) = b_1,
n>1 のとき
 a_n - a_(n-1) = n*log(n) -(n-1)log(n-1) -1
  = n*log(n) - (n-1){log(n) + log(1 -1/n)} -1
  = n*log(n) - (n-1){log(n) - log(1 + 1/(n-1))} -1
  < n*log(n) - (n-1){log(n) - 1/(n-1)} -1
  = log(n),
 b_n - b_(n-1) = (n+1)log(n) -n*log(n-1) -1
  = (n+1)log(n) -n*log(n-1) -1
  = (n+1)log(n) -n{log(n) + log(1 -1/n)} -1
  > (n+1)log(n) -n{log(n) - 1/n} -1
  = log(n),
よって成立。
554132人目の素数さん:2008/09/25(木) 23:57:11
不等式たん (;´д`) ハァハァ…
555132人目の素数さん:2008/10/03(金) 03:31:17
俺も>>410の証明知りたい
夏休みずっと考えたけどできんかった(´・ω・`)
556132人目の素数さん:2008/10/04(土) 01:08:48
>>555
みせてもらおうか!
その過程とやらを!
557132人目の素数さん:2008/10/04(土) 01:37:22
πを上から評価して e を下から評価するだけでしょ。
e の方はテーラー展開ですぐ出る。
π の方は単位円に外接する正 2^n 角形の面積を考えれば良い。
558132人目の素数さん:2008/10/04(土) 07:12:07
>>557
全然計算してないでしょ
それじゃあ、1日計算しても無理
559132人目の素数さん:2008/10/04(土) 09:38:47
>>557
なめんなよ!
560132人目の素数さん:2008/10/04(土) 14:07:15
>>415の小数第三位は 6 じゃ無くて 8 の間違いじゃない?
e^6 =403.42879 34927 35122 60838 71805.........
とかそんな感じになったんだけど。
561132人目の素数さん:2008/10/04(土) 20:39:17
〔問題096〕
連続函数f(x): R→R に対して、以下の2つの方程式(1)〜(4)を考える。
 f(x) = x  … (1)
 f(f(x)) = x  … (2)
 f(f(f(x))) = x  … (3)
 f(f(f(f(x)))) = x  … (4)
方程式(1)が実数解を持たないならば、方程式(2)〜(4)も実数解を持たないことを示せ。

http://science6.2ch.net/test/read.cgi/math/1219648297/096, 104, 118
京都大学入試作問者スレ@
562132人目の素数さん:2008/10/04(土) 20:41:28
>>561 スレ違いっぽいが・・・・・

(略証)
 f(x) - x = g(x) とおくと (1) は
 g(x) = 0,
題意により、g はすべての実数xで連続。もし
 g(a) ≦ 0 ≦ g(b),
なる a,b があったと仮定すれば、中間値の定理により、(1)が実数解をもつ。
これは 題意に反する。
∴ g(x) は定符号。

題意より f(0) ≠ 0,
f(0) < 0 のとき g(x) <0,
 x > f(x) > f(f(x)) > f(f(f(x))) > …
f(0) > 0 のとき g(x) >0,
 x < f(x) < f(f(x)) < f(f(f(x))) < …
563132人目の素数さん:2008/10/04(土) 23:49:07
>>557
評価がかなりシビアなんで、手計算だとその方法では無理。
564132人目の素数さん:2008/10/05(日) 00:26:53
>>562
関数方程式ヲタもいるから、okokよん! (*゚∀゚)=3 ハァハァ…
565132人目の素数さん:2008/10/05(日) 04:42:02
e のほうはTaylor展開の収束がやたら速いから
十数桁ならすぐ計算できる。(解析概論に例題として載ってる)

だからこの問題は要するに
π を 誤差 0.00 001/400・5 = 1/200,000,000 ・100% 程度で
上から評価せよという問題とほぼ同義。11桁くらい正しく出ればうまくいく。

2^n 角形による近似は誤差が約 1/2 倍で小さくなっていくだけだから
1 桁進むのに 3.3 回くらい掛かる。 30 回程度は計算しないといけないので
一日じゃ無理そう。じゃあ不可能なのかというとそうでもなくて
1579年にVièteが外接正 393216 角形の周長から π < 3.1415926537 を導出している。
1596-1610年にはLudolph van Ceulenが正 32212254720 ( = 60・2^29 ) 角形の周長から
32(35?) 桁まで正しく計算している。独逸では彼の業績を記念して円周率をLudolph数とも言う。
和算家の村松茂清が同じ方法で七桁正しく計算している。Archimedesから続く伝統的方法で
中国人は劉徽のalgorithmというらしい。
http://en.wikipedia.org/wiki/Liu_Hui%27s_%CF%80_algorithm
# 建部賢弘は正1024角形を用いて42桁まで求めたとか書いてあるけど
# これは42桁まで正しかったんだろうか?だとするとかなり工夫を凝らした方法のはずだが。

で、もっと早く求めたいなら色んな方法があるが、
θ < (2sin θ + tan θ)/3 を使うSnell(Ludolphの弟子)の方法(1621)ってのがあって、
Huygensはこれを改良して正六角形だけで π < 3.1415926538 まで出している。
これ系を使うのが一番賢いかな。
566132人目の素数さん:2008/10/05(日) 04:43:19
arctan のTaylor 展開(Gregory-Leibnitz級数)を使う方法もあり
これはかなり色んな亜種があってMachin-like formulaと呼ばれている。
π/4 = 4arctan(1/5) - arctan(1/239) が本家Machinだけど、これ以外にいろいろあって
π/4 = arctan(1/2) + arctan(1/7) とか π/4 = 5arctan(1/7) + arctan(3/79) はEulerによる。
Eulerは後者を使って一時間で20桁計算したらしい。ただしEulerは暗算の達人だったので
自分も出来るなどとはあまり思わないほうが良いかも。
ただarctanを使って上から評価はきちんと厳密にやると面倒。ほぼ等比級数のスピードで収束。

Ramanujanの9801公式とかChudnovsky兄弟の公式なんてのもあって
これはきちんと証明されたのはつい最近のこと。厳密な上からの評価には向かなさそう。
計算機で計算する場合は算術幾何平均を利用した
Gauss-Legendre algorithm(Brent-Salamin algorithm)
とかBorwein's algorithmとかいうのも使われる。

円周率の公式と計算法
http://www.kurims.kyoto-u.ac.jp/~ooura/pi04.pdf

イロモノとしてはBBP公式なんていう、16進数表記での n 桁目を
n-1 桁までを計算せず直接に計算できるような公式や、
Buffon's needleと言って針を等間隔の縞模様にたくさん
確率計算から近似的にπを求める、というのもある。
統計的に処理できれば、これでも科学的には実験で値を測定したことになる。
数学的には却下だが。

残りの参考サイト
円周率の公式集 暫定版
http://www.pluto.ai.kyutech.ac.jp/plt/matumoto/pi_small/
http://en.wikipedia.org/wiki/Category:Pi_algorithms
http://en.wikipedia.org/wiki/Pi 記事内のリンクも参照。
http://ja.wikipedia.org/wiki/%E5%86%86%E5%91%A8%E7%8E%87%E3%81%AE%E6%AD%B4%E5%8F%B2
567132人目の素数さん:2008/10/05(日) 15:38:02
全面的に数値計算するのは題意に反してるんだが。
と思ったが>>410には書いてないか。数検の方にはなるべく数値計算せずに、と書いてある
568132人目の素数さん:2008/10/05(日) 17:33:57
綺麗には出て来ないと思うけどなあ。
だから「なるべく」と数値計算が少ない解答を
高く評価するという表現に止めてあるわけで。

Snellの公式改良してsinやtanのTaylor展開使ったら
数値計算は少なくて済むと思う。
569132人目の素数さん:2008/10/05(日) 18:00:03
>>410は数値計算しないで示すことができるの?
もしできても普通思いつきもしないようなことするんだろうな(人´A`;)
570132人目の素数さん:2008/10/06(月) 20:05:15
>>410 のもとの問題文は↓(>>430)

円周率をπ、自然対数の底をeとするとき
  π^4+π^5≒e^6  (400余りの数値で小数点以下4けたまで同じ)
で、しかも右辺が僅かに大きいことがコンピュータによる数値計算で知られています。
 数値計算をせずに
  π^4+π^5<e^6
であることを理論的に証明しなさい。
571KingMind ◆KWqQaULLTg :2008/10/06(月) 20:54:56
Reply:>>570 数値計算もまた誤差の評価で成り立っている。
572132人目の素数さん:2008/10/06(月) 23:59:10
>>570
論理的になら証明できるんだけど、残念!
573132人目の素数さん:2008/10/07(火) 00:09:55
>>570
いかにもエレ解な問題だな
574132人目の素数さん:2008/10/07(火) 01:17:05
これなんで自然に出て来なさそうかというと、
π^4+π^5≒e^6
ってのは偶然近いだけで、別に
深い数学的事実の表れとかじゃないからなんだよなあ、
575132人目の素数さん:2008/10/08(水) 01:17:31
>>534
http://www3.tokai.or.jp/meta/gokudo-/omoshi-log/
最後の2つを参照、作成時刻にも注目
576132人目の素数さん:2008/10/09(木) 21:33:02
>373-374 , 394
http://science6.2ch.net/test/read.cgi/math/1219648297/121

f(x) = 6/(1-x) - 1/x とおくと、
 (左辺) = f(a) + f(b) + f(c),
(a,b,c)の変域は、平面a+b+c=1上の a=1/2, b=1/2, c=1/2 を辺とする正三角形(但し頂点は除く)

・境界上の極大
 6/x + 6/(0.5+x) = 16 + 2(1-4x)^2 /{(1-x)(1+2x)},
 1/x + 1/(0.5-x) = 8 + (1-4x)^2 /{x(1-2x)},
より、辺 c=1/2 では
 (左辺) = f(a) + f(1/2 - a) + f(1/2) = 18 - (1-4a)^2 {7+(1-4a)^2}/{4a(1-a)(1+2a)(1-2a)} ≦ 18,
等号成立は (a,b,c) = (1/4,1/4,1/2) のとき。

・ 内部の極大
生姜ないから、微分法を使おう。
束縛条件(a+b+c=1)があるので、ラグランジュの未定乗数λを使う。
 I(a,b,c;λ) = f(a) + f(b) + f(c) - λ(a+b+c-1),
 ∂I/∂a = ∂I/∂b = ∂I/∂c = 0 から
 f '(a) = f '(b) = f '(c) = λ,  f '(x) = 6/(1-x)^2 + 1/(x^2),
∴ (a,b,c; λ) = (1/3, 1/3, 1/3; 45/2) で 極大値 18 をとる。

なお、この極大と境界上の極大(1/4,1/4,1/2)の間の鞍点(峠点)↓も解ではあるが、これらは捨てる。
 (a,b,c; λ) = (0.279000307274921, 0.279000307274921, 0.441999385450158, 24.3886725897975)

しかし・・・・・後味わるいな。
577576:2008/10/09(木) 21:47:21
・境界上の極大
 6/(1-x) + 6/(0.5+x) = 16 + 2(1-4x)^2 /{(1-x)(1+2x)},
578132人目の素数さん:2008/10/10(金) 00:57:08
後味の悪さってのは、やはり、中高生でも分かる解法じゃないからだろうな
579132人目の素数さん:2008/10/11(土) 00:16:10
ド田舎に住んでいるんだけど、近所の大学にAMMが置かれなくなってネタがたりねぇ…
580576:2008/10/13(月) 04:25:00
>> f '(a) = f '(b) = f '(c) = λ,
を解くところを補足しとく。
 f '(x) = 6/(1-x)^2 + 1/(x^2),
 f "(x) = 12/(1-x)^3 - 2/(x^3),
∴ {x - 1/[1+6^(1/3)]}*f "(x) ≧ 0,
∴ 区間 (0,1/2] で、f '(x) が等しいxは高々2個しかない。
∴ 極値では、a,b,c のうちの2つは一致する。
a=b, c=1-2a としてよい。このとき
 (左辺) = 2f(a) + f(1-2a)
  = 2{6/(1-a) -1/a} + 6/(2a) - 1/(1-2a)
  = (1+8a-21a^2)/{a(1-a)(1-2a)}
  = 18 - (4a-1)(3a-1)^2 /{a(1-a)(1-2a)}
  ≦ 18,     (1/4 ≦ a ≦ 1/2).
等号成立は a=1/4 と 1/3 のみ。

なお、a=0.279000307274921・・・ には極小(鞍点、峠点)がある。
581132人目の素数さん:2008/10/18(土) 06:28:12
>>341
A.435. Prove
 (a+b+c)*(1/a + 1/b + 1/c) ≧ 6{a/(b+c) + b/(c+a) + c/(a+b)},
where 1≦a,b,c≦2.

(略解)               (>>394 を参照)
>>373-374 から,
 6/(b+c) - 1/a + 6/(c+a) - 1/b + 6/(a+b) - 1/c ≦ 18/(a+b+c),
両辺に a+b+c を掛けて,
 6{a/(b+c) + b/(c+a) + c/(a+b)} - (a+b+c)(1/a + 1/b + 1/c) ≦ 0,
582132人目の素数さん:2008/10/18(土) 08:19:15
>>341

B.4021
a_k = 1 + b_k, b_k≧0 とおくと
 (左辺) = (b_1 +2)(b_2 +2)(b_3 +2)・・・・・(b_n +2) ≧ (b のn〜2次の項) + 2^(n-1)・(b_1 + b_2 + ・・・・・ + b_n) + 2^n
   ≧ {2^(n-1)}{b_1 + b_2 + ・・・・・ + b_n +2) = (右辺).

A.433 A.436 A.439 A.447 は解答付き。
583132人目の素数さん:2008/10/19(日) 00:12:38
>>373-374 を何とか高校レベルで解けないか頑張ってみて
次の問題に帰着され所までいって挫折した。
より遠ざかった感もあり...

t に関する実係数3次方程式 t^3 - (r-2)t^2 + qt - r=0 が全て1以上の実数解を3個持てば、
r(r+1)≧6q が成り立つ。
584583:2008/10/19(日) 00:45:17
4 r^2 + 32 r - 4 q r - 11 r^2 - q r^2 + 2 r^3≦0、r≧5、q≧2 r-7 ならば
r (r+1)≧6 q が成り立てば良いか...駄目だ...
585584:2008/10/19(日) 00:46:41
× 4 r^2 + 32 r - 4 q r - 11 r^2 - q r^2 + 2 r^3≦0
○ 4 q^2 + 32 r - 4 q r - 11 r^2 - q r^2 + 2 r^3≦0
586132人目の素数さん:2008/10/19(日) 07:23:26
〆切過ぎたから今月の大数の宿題

a_1=2,a_(n+1)={1+(2+√3)a_n}/{(2+√3)-a_n}

a_n<5を示せ
587132人目の素数さん:2008/10/19(日) 11:54:01
>586
 a_(n+1) = (2-√3 + a_n)/{1 - (2-√3)a_n}
   = {tan(π/12) + a_n}/{1 - tan(π/12)a_n},
∴ a_n = tan(α + (n-1)π/12),
ここに α = arctan(a_1), a_n は周期12をもつ。
 a_(n+6) = -1/a_n.
∴ はじめの6項を求めれば分かる。
588583-585:2008/10/19(日) 15:08:21
多投スマ祖。

q≦2 r - 3 を忘れてた。
4 q^2 + 32 r - 4 q r - 11 r^2 - q r^2 + 2 r^3 ≦ 0 を q について解いて、
r-q 平面でグラフ書いて領域で責めたら何とかなった。
589132人目の素数さん:2008/10/20(月) 04:57:18
>>583
 t - {(r-2)/3} = T,
とおいて 2次の項を消すと、
 t^3 - (r-2)t^2 + qt - r = T^3 + QT - R,
ここに、Q = q -3{(r-2)/3}^2, R = r - q{(r-2)/3} + 2{(r-2)/3}^3,

・3つの実根をもつから
 Q <0, R^2 < 4(-Q/3)^3,

・解が t≧1 だから
 (t-1)^3 -(r-5)(t-1)^2 + (q-2r+7)(t-1) + (q-2r-3) =0,
の解がすべて t-1≧0.
根と係数の関係より
 r-5 ≧0, q-2r+7 ≧0, q-2r+3 ≦0,
590132人目の素数さん:2008/10/20(月) 05:37:48
>>341

A.436. Prove that |{n√2} - {n√3}| > 1/(7n^3),
 for every positive integer n.

(略証)
0 ≦ {x} <1 より |t| <1, また、k = [n√3] - [n√2] とおくと   (← ガウスの記号)
 t = {n√2} - {n√3}
  = n√2 - [n√2] - n√3 + [n√3]
  = k - (√3 -√2)n (k∈N)
  = ((k^2 -5n^2) + (2√6)n^2) / (k + (√3 -√2)n)
  = ((k^2 -5n^2)^2 -24n^4) / ((k - (√3 +√2)n)(k + (√3 -√2)n)(k + (√3 +√2)n))
  = (k^4 - 10(kn)^2 + n^4) / ((t -2√2・n)(t +2(√3 -√2)n)(t +2√3・n)),
 分母は0でない整数。

・ n≧20 のとき
 |t -2√2・n| < 2√2・n + 1 ≦ (2√2 + 1/20)n,
 t +2(√3 -√2)n < 2(√3 -√2)n + 1 ≦ (2(√3 -√2) + 1/20)n,
 t +2√3・n < 2√3・n +1 ≦ (2√3 + 1/20)n,
 辺々掛けて
 |分母| < 6.9356560324845688673761191952915・・・ * n^3,
 より成立。

・n≦20 のとき、
 (左辺) ≧ (√3 -√2)/n^3 > 1/(√10・n^3) > 1/(7n^3).
591132人目の素数さん:2008/10/20(月) 23:06:03
>590
 分子は0でない整数。


>>565
Snellの方法の略証
相加・相乗平均より
 { cos(x) + cos(x) + 1/cos(x)^2 }/3 > 1,
これをxで積分する。 [0<x<θ]
592132人目の素数さん:2008/10/27(月) 01:28:31
>>341

B.4049. a,b,c are positive real numbers, such that ab+bc+ca=t. Prove that
   a/(a^2 -bc+3t) + b/(b^2 -ca+3t) + c/(c^2 -ab+3t) ≧ 1/(a+b+c),

(略証)
 a+b+c =s, abc =u とおく。
 (左辺) - (右辺) = 2{us^3 + (s^2 -4t)t^2} / {(6s^2・t^2 + 8t^3 + us^3)s}  ・・・・(*)
  = 2{u(s^4 -9t^2) + (s^3 -4st +9u)t^2} / {(6s^2・t^2 + 8t^3 + us^3)s^2}
  = 2{u(s^2 +3t)F_0 + t^2・F_1} / {(6s^2・t^2 + 8t^3 + us^3)s^2}
  ≧ 0,
ここで Schur の不等式 F_0 = s^2 -3t ≧0, F_1 = s^3 -4st +9u ≧0 を使った。

(*) a^2 -bc =A, b^2 -ca =B, c^2 -ab =C とおくと
 S = A + B + C = s^2 -3t,
 T = AB + BC + CA = -t(s^2 -3t),
 U = ABC = us^3 - t^3,
 (左辺) = a/(A+3t) + b/(B+3t) + c/(C+3t)
  = (aBC + AbC + ABc +9st^2)) / (U +3tT +9t^2・S +27t^3)
  = 2{us^3 + (s^2 -4t)t^2}/{(6s^2・t^2 + 8t^3 + us^3)s}.
593132人目の素数さん:2008/10/27(月) 02:43:50
次の問題を現実的な方法でやってくれませんか?
30分以内に確実にやって下さいと要請されたらどうするかっていうこと。

[問題]
abc=2なる正の実数a,b,cの組に対して、次の式の最小値を求めよ
1/(a(b+1))+1/(b(c+1))+1/(c(a+1))

594132人目の素数さん:2008/10/27(月) 08:44:05
>>593
> 次の問題を現実的な方法でやってくれませんか?
> 次の問題を現実的な方法でやってくれませんか?
> 次の問題を現実的な方法でやってくれませんか?
> 次の問題を現実的な方法でやってくれませんか?
> 次の問題を現実的な方法でやってくれませんか?

新手の釣り師か?
595132人目の素数さん:2008/10/27(月) 09:34:46
計算機で解けば?
596132人目の素数さん:2008/10/27(月) 09:45:21
釣りじゃないですよ。
この問題の出典は数学検定1級の2次という変なところなのですが、
試験時間が短めで、時間制限を気にしないといけないのです。
もちろん値だけではだめで、論証しないといけません。
なので、『現実的な方法』という言葉を使いました。
そこで皆様の知恵をかりたいのですが、どうでしょうか。

597132人目の素数さん:2008/10/27(月) 09:55:56
それのどこが不等式?
598132人目の素数さん:2008/10/27(月) 16:59:40
>597さん
a=b=c=2^(1/3)で最小を取ることが予想できるので、
abc=2なる任意に正の実数a,b,cに対して、
次の不等式を示すことになるので、
そういう意味で不等式の問題とみなせると思いました。

1/(a(b+1))+1/(b(c+1))+1/(c(a+1)) ≧ 3/(2^(1/3)*(1+2^(1/3)))

599132人目の素数さん:2008/10/27(月) 23:10:30
>>596
お前な、順序が間違ってるだろ!
まず、>>596を書いてから、>>593で質問だろ!
情報を小出しにするなとママに教わらなかったのか?
600132人目の素数さん:2008/10/29(水) 03:55:59
Σ[k=1→n](1/k)>5
となる最小の整数nを求めよ

a,b,cが相異なる正の数で√a+√b+√c=1を満たすとき
{ab/(b−a)}log(b/a)+{bc/(c− b)}log(c/b)+{ca/(a−c)}log(a/c)≦1/3
を示せ
601132人目の素数さん:2008/10/29(水) 04:24:00
Σ[k=1→n](1/k)= log(n)+γ+O(1/n) に注意すると、
だいたいn=[e^(5-γ)]=83 とわかる。
答えはn=83
602132人目の素数さん:2008/10/29(水) 19:49:32
Σ[k=1→n](1/k)>4
となる最小の整数nを求めよ

これだと高校生でも何とかできるか
603132人目の素数さん:2008/10/29(水) 21:34:14
それの改良問題。
[Σ[k=1→n](1/k)] = [e^(5-γ)]
を満たさない正整数nは無限に存在するか。
ただし、γはオイラー定数とし、
[x]はxの整数部分を表すとする。

これだと愚直に計算機使うだけじゃ無理。

604132人目の素数さん:2008/10/29(水) 21:40:55
>603
問題ミス。
Σ[k=1→n](1/k)>m を満たす最小の整数nが、
n = [e^(m-γ)] とならない正整数mは無限に存在するか。

605132人目の素数さん:2008/10/29(水) 23:24:13
>>600
S_82 = 5 -  971061970808803141778039548955447 / D_5,
S_83 = 5 + 16703434187251287967291034353582814 / (D_5 * 83),
D_5 = 2^6 * 3^4 * 5^2 * 7^2 *11*13*17*19*23*29*31*37*41*43*47*53*59*61*67*71*73*79,

>>602
S_30 = 4 - 11675421053 / D_4,
S_31 = 4 + 1967151510157 / (D_4 * 31),
D_4 = 2^4 * 3^3 * 5^2 *7*11*13*17*19*23*29,


S_11 = 3 - 2221 / D_3,
S_12 = 3 + 89 / D_3,
D_3 = 2^3 * 3^2 * 5*7*11,

S_3 = 2 - 1/D_2,
S_4 = 2 + 1/(D_2*2),
D_2 = 2 * 3,
606592:2008/10/29(水) 23:35:18
>>592 の訂正, スマソ.

 (左辺) = a/(A+3t) + b/(B+3t) + c/(C+3t) = (3us + 8t^2)s/(6s^2・t^2 + 8t^3 + us^3).
607132人目の素数さん:2008/10/31(金) 21:53:26
>>600

↓の補題に x=√(a/b), √(b/c), √(c/a) を代入してたす。
  (左辺) < √(ab) + √(bc) + √(ca) < (1/3)(√a + √b + √c)^2,

〔補題〕
x>0, x≠1 のとき
 {x/(x^2 -1)}log(x^2) < 1,
(略証)
 f(x) = x -(1/x) -2log(x),
とおくと、f(1) =0,
平均値の定理より
 {f(x)-f(1)}/(x-1) = f '(ξ) = (1 - 1/ξ)^2 >0,   (ξは1とxの中間にある)
これに x/(x+1) を掛ける。

ハァハァ
608132人目の素数さん:2008/10/31(金) 22:19:48
(*゚∀゚)=3 ハァハァ…
609132人目の素数さん:2008/11/02(日) 04:23:18
正の実数x,y,zに対して次を示せ。
(xy)^3/(x^3+1)+(yz)^3/(y^3+1)+(zx)^3/(z^3+1) ≧ 6/{xyz(1+xyz)}
できる神いる?
610132人目の素数さん:2008/11/02(日) 06:46:30
>>609
x=y=z=1のときとか成り立たないんだが・・・
なんか間違えてねーか?
611132人目の素数さん:2008/11/02(日) 10:20:43
>>593>>598の変形し損ね?

612132人目の素数さん:2008/11/04(火) 04:11:04
>>600
次の問いに答えよ。
(1) xが正の数のとき│log x│≦│x-1│/√x を示せ。
(2) p, q, r がp + q + r =1を満たす正の数のときp^2+ q^2+ r^2 ≧1/3を示せ。
(3) a , b, c が相異なる正の数で、√a + √b + √c = 1を満たすとき、
{ab/(b - a)}・ log(b/a) + {bc/(c - b)}・ log(c/b) + {ca/(a - c)}・ log(a/c) ≦ 1/3
を示せ。             (2007 阪大)
613132人目の素数さん:2008/11/04(火) 10:21:31
>>612
誘導なしだったら、いい感じだね
614132人目の素数さん:2008/11/04(火) 20:40:08
test
615不等式だけの学会があるらしい:2008/11/04(火) 21:07:44
lemmma3
a1≧a2,b1≧b2 -> (a1-a2)(b1-b2)≧0 -> a1*b1+a2*b2≧a1*b2+a2*b1

TH2
任意の自然数nに対して:a1^n+a2^n+,,,+an^n≧n*a1*a2*,,,*an
証明)
n=1:a1≧a1
n=kの時成立していると仮定しn=k+1で成立する事を示す。
まず、a1≧a2≧,,,≧a(k+1)@と仮定しても一般性を失わない。

a1^(k+1)+a2^(k+1)+,,,+ak^(k+1)+a(k+1)^(k+1)
=a1^(k+1)+a2^(k+1)+,,,+ak^k*ak +a(k+1)^k*a(k+1)
≧a1^(k+1)+a2^(k+1)+,,,+ak^k*a(k+1)+a(k+1)^k*ak
=a1^(k+1)+a2^(k+1)+,,,,+a(k-1)^k*a(k-1)+ak^k*a(k+1)+a(k+1)^(k-1)*ak*a(k+1)
≧a1^(k+1)+a2^(k+1)+,,,,+a(k-1)^k*a(k+1)+ak^k*a(k+1)+a(k+1)^(k-1)*ak*a(k-1)
(ここまでの不等号は全てlemma3と@による)
,,,,
≧(a1^k+a2^k+,,,+ak^k)*a(k+1)+a1*a2*,,,*ak*a(k+1)
(,,,及び最後の不等号もlemmma3と@による。
ai^k*ai+a(k+1)^i*a(i+1)*,,,*ak*a(k+1)≧ai^k*a(k+1)+a(k+1)^(i-1)*ai*a(i+1)*,,,*ak*a(k+1)
がやはりlemmma3と@によって成立するので、この事が言える)

≧k*(a1*a2*,,,*ak)*a(k+1)+a1*a2*,,,*ak*a(k+1)
(この不等号は帰納法の仮定による)

=(k+1)*a1*a2*,,,*ak*a(k+1)

よってTH2が成立。

TH1.TH2において、Ak=ak^nと置いていけば、明らかな相加相乗平均の不等式が現れる。

という事が今年の夏、8/18だか8/19に日本の高校の教師が示された。
616132人目の素数さん:2008/11/04(火) 21:11:03
617不等式だけの学会があるらしい:2008/11/04(火) 21:19:36
日本の高校の教師によって示された。

俺はまず、ハーディーにあたってみたが、あの不等式の本ではもう少し一般化した式が
もう少し、めんどくさく示されており、ハーディーとリトルウッドの明晰でわかりやすいスタイルの中には入らない。

次に「天書の証明」にあたったが、コーシーがほんの一歩、めんどくさい証明をしており、
これが、美しい部類の物として、「載っていた」

シンプルであり、アルゴリズムの様な、簡単な、美しい証明だと思う。

「日本の高校の先生が「天書」から証明を盗んできた。」
618132人目の素数さん:2008/11/05(水) 00:00:11
>>617
「天書の証明」は、数ヲタとして持っておいたほうがいいですか?
本棚に飾っておいたほうがいいですか?
てか、オヌヌメですか?

最近、本を買っていないので何か買いたい気分です( ゚∀゚)
619132人目の素数さん:2008/11/05(水) 00:27:11
>>618
あれは持っておいて損はない。
俺は日本語版(第2版)と原書(第3版)を両方買った。
620132人目の素数さん:2008/11/05(水) 00:31:41
アルゴリズムの様な、簡単な、美しい証明

のアルゴリズムのようなっていう比喩が全く意味が分からん
621132人目の素数さん:2008/11/05(水) 00:47:21
>>620
俺は分かるぞ。手続きが明らかになる構成的証明だということだと思う。
相加平均と相乗平均という,全く形が異なるものの間を一気に飛ぶのではなく,
相加平均が,1つずつ項を入れ替えてゆくことで少しずつ小さくなってゆき,
やがて相乗平均に至るという,途中経過が明らかになる証明だ,という意味だろう。
俺も全く同感だ。
622132人目の素数さん:2008/11/05(水) 05:24:54
x,y,z≧0,x+y+z=1のとき
xy+yz+zx-2xyzの最大値、最小値を求めよ

ところで質問なんですが
任意の整数nに対して
n^2+an+b≧0
となるようなa,bの条件出すこと出来ますか?
623132人目の素数さん:2008/11/05(水) 07:24:12
>>622
(1-2x)(1-2y)(1-2z)を展開すればわかる。

a^2-4b≦0
624132人目の素数さん:2008/11/05(水) 07:32:47
すまん。整数だったな。
0<a^2-b≦1これも必要かな……。
625132人目の素数さん:2008/11/06(木) 20:53:25
基本対称式を使った初心者でも何とか解ける不等式を教えてください。
626132人目の素数さん:2008/11/06(木) 23:19:08

('A` ) プウ
ノヽノ) =3'A`)ノ ヒャー
  くく へヘノ ←>>625
627132人目の素数さん:2008/11/07(金) 00:08:42
>>625
x_1, ……, x_n を正の数とする。
これらの相加平均を A,
これらの二乗平均平方根を M ( =√{((x_1)^2+……+(x_n)^2)/n} ),
これらから作られる2次の基本対称式を S (=x_1x_2+……) とおく。

このとき,A≧M*n^{S/{n(n-1)M^2}-1/2} が成り立つことを示せ。(出典:Part2-847)

--------------
x_1, ……, x_n を正の数とする。
これらから作られる k 次の基本対称式を e_k とおき,
A_k=(e_k / C[n,k])^(1/k) とおく(C[n,k]は二項係数)。
このとき,
A_1≧A_2≧……≧A_n
が成り立つことを示せ。(出典:マクローリンの不等式)


628132人目の素数さん:2008/11/07(金) 19:37:45
>>620
>>621さんに付け加える事は何もないですが、要はlemma3がサブルーチンで、この証明ではほとんどが
このサブルーチンで片がついているのです。プログラムでも、すっきりした簡単なメインルーチンと
もし、サブルーチン一つでかなりな複雑な事柄が片付けば、それは「美しいプログラム」だと
思います。
 要はわかりやすく、読みやすい。と言う事かなと思います。話はむずかしくではなく、簡単でわかりやすい
方が「美しい」と思います。あなたが例えば、人様のノートをテスト前にコピーさせてもらった場合、要約もすばらしく、
論点も明確なノートなら、やはり、「美しい」と思うのではないでしょうか?
それと同じだと思います。
629>>615訂正:2008/11/07(金) 19:53:16
「まず、a1≧a2≧,,,≧a(k+1)@と仮定しても一般性を失わない。」の位置がおかいかったようです。
n=1の前に、
「まず、a1≧a2≧,,,≧an@と仮定しても一般性を失わない。」が正しいです。
630132人目の素数さん:2008/11/07(金) 20:07:16
反応がないのは>>437で既出だからだよ>>615くん
どこの山から出てきたんだ?
631132人目の素数さん:2008/11/07(金) 20:09:40
単発スレ立てる厨房よりはマシじゃね?
632132人目の素数さん:2008/11/07(金) 23:55:05
>>630-631
少し黙ってろ!
633132人目の素数さん:2008/11/08(土) 03:26:54
a,b,cは自然数で
(1/a)+(2/b)+(3/c)<1
を満たすとき
(1/a)+(2/b)+(3/c)の最大値を求めよ

f(a)=∫[0→π/4] |sinx−a cosx|dx
の最小値を求めよ

x≧0において
f’(x)>0,∫[0→x] f(t)dt≧x
ならば,x>0においてf(x)>1を示せ
634132人目の素数さん:2008/11/08(土) 03:32:44
>>633

f(a)=∫[0→π/4] |sinx−a cosx|dx
の最小値を求めよ

不等式では、ない。これ去年、代ゼミに通ってた友人が持ってきたテキストにあったな。
635132人目の素数さん:2008/11/08(土) 09:53:36
>>634
それは東工大の過去問だな。sinx=t と置換すれば ∫|f(t)-a|dt の形になるので,はみ出し削り論法で終わり。
636132人目の素数さん:2008/11/08(土) 20:04:09
>>633 (上)
 1-(1/1332),
(a,b,c) = (37,9,4) のとき.

>>633 (中)
 f(a) = 1 - (1+a)/√2,       (a≦0)
    = -1 - (1+a)/√2 +2√(1+a^2), (0≦a≦1)
    = -1 + (1+a)/√2,       (a≧1)
637132人目の素数さん:2008/11/08(土) 22:43:41
>>636
それf(a)求めただけやん(笑)
638132人目の素数さん:2008/11/08(土) 23:55:39
>>637
具体的に書けるから自明すぎてつまらないと言うメッセージなのかもしれない
639132人目の素数さん:2008/11/09(日) 01:17:19
>>638
おまいはテレパスか!
640132人目の素数さん:2008/11/09(日) 02:10:10
>>636
上教えてちょ
641636:2008/11/09(日) 21:47:35
>>637
 f '(a) = 0 から a = 1/√7, 最小値は
 f(a) = -1 + (√7 -1)/√2 = 0.16372191220042316839103000405343・・・
642132人目の素数さん:2008/11/09(日) 22:10:22
>>633 (下)

部分積分を使うらしい・・・
 ∫[0→x] f(t)dt = [ t・f(t) ](t=0→x) - ∫[0→x] t・f'(t)dt < [ t・f(t) ](t=0→x) = x・f(x).
643132人目の素数さん:2008/11/09(日) 23:17:35
その部分積分は名古屋大かどっかの問題にあったな
解いたことがある。もう忘れてたけど。
≧≦
645446:2008/11/10(月) 23:33:38
>>642
g'(x)≧0 かつ ∫[0→x] g(x)≧0 と同値だから lim[x→0] g(x)≧0 が自然に言えて解決.
646132人目の素数さん:2008/11/11(火) 08:00:00
1/37+2/9+3/4=1331/1332.
1/31+2/3+3/10=929/930.
1/5+2/41+3/4=819/820.
1/38+2/9+3/4=683/684.
1/15+2/11+3/4=659/660.
647132人目の素数さん:2008/11/13(木) 03:04:12
不等式のノート作ってる方とかいます?
648132人目の素数さん:2008/11/13(木) 06:56:52
>>647
名前を書かれると無性に不等式を証明したくなるとか?
649132人目の素数さん:2008/11/13(木) 13:03:58
>>647
てふでまとめていますが何か?
650132人目の素数さん:2008/11/14(金) 07:59:40
>>649
もううpせざるを得ないだろう
651132人目の素数さん:2008/11/14(金) 10:04:11
B5サイズで50枚以上になるからなぁ…、断るッ!
652132人目の素数さん:2008/11/18(火) 23:21:51
【(2nCn)/(n+1)】カタラン数【(2n)!/(n+1)!n!】より

64 名前:132人目の素数さん[sage] 投稿日:2008/01/03(木) 18:20:39
〔不等式064〕
 C[2m,m] = (4^m)/√(mπ) * exp(-1/8m + O(1/m^3)) 〜 (4^m)/√(mπ) *(1 - 1/(8m) + …),

(略証)
スターリングの不等式
 (n +1/2)log(n) -n +(1/2)log(2π) +1/(12n) -1/(360n^3) < log(n!) < (n +1/2)log(n) -n +(1/2)log(2π) +1/(12n),

 log(C[2m,m]) = log((2m)!) -2log(m!),
に代入する。
 (2log(2))m -(1/2)log(mπ) -1/(8m) -1/(2880m^3) < log(C[2m,m]) < (2log(2))m -(1/2)log(mπ) -1/(8m) +1/(180m^3),

65 名前:132人目の素数さん[sage] 投稿日:2008/01/20(日) 20:24:33
大学への数学1月号の宿題を解いたつわものはいる?
 lim[n→∞) {(1/2^(2n -1/2))*C[4n,2n]/C[2n,n]}^(2n)

http://science6.2ch.net/test/read.cgi/math/1200494361/113
さくらスレ235

66 名前:スターリング[sage] 投稿日:2008/01/20(日) 20:34:06
>65

log(n!) = (n +1/2)log(n) -n +(1/2)log(2π) + 1/(12n) -1/(360n^3) +O(1/n^5),

log(C[2n,n]) = log((2n)!) - 2*log(n!)
 = 2log(2)*n -(1/2)log(nπ) -1/(8n) +1/(192n^3) +O(1/n^5),

log(与式) = -(2n -1/2)log(2) +log(C[4n,2n]) -log(C[2n,n])
 = {1/(16n) -O(1/n^3)}*(2n)
 = (1/8) - O(1/n^2) → 1/8,  (n→∞)
653132人目の素数さん:2008/11/18(火) 23:56:52
〔問題202〕
任意の正の整数mに対して不等式
 |sin(a)| + |sin(2a)| + ・・・・ + |sin(ma)| > (m/2) + (1/4) - 1/|4sin(a)|.

(略証)
|sin(ka)| ≧ {sin(ka)}^2 = {1 - cos(2ka)}/2 = (1/2) - 2cos(2ka)sin(a)/(4sin(a)) = (1/2) - {sin((2k+1)a)-sin((2k-1)a)}/(4sin(a)),
k=1,2,・・・,m について和をとる。

 http://science6.2ch.net/test/read.cgi/math/1220115988/202
654132人目の素数さん:2008/11/19(水) 00:26:14
任意の正の整数mに対して不等式
 |sin(a)| + |sin(2a)| + ・・・・ + |sin(ma)| < √{m[(m/2) + (1/4) + 1/|4sin(a)|]}.
が成り立つ。
 
(略証)
 (左辺) ≦ √{mΣ[k=1,m] sin(ka)^2} = √{m[(m/2) - (sin((2m+1)a)-sin(a))/4sin(a) ]}
655132人目の素数さん:2008/11/19(水) 16:36:21
なんだこのスレwwww
おもすれーwwwうぇwwww
656132人目の素数さん:2008/11/19(水) 22:37:59
>>653-654
ワイルの一様分布定理から、

〔補題〕 a/π≠整数 ならば、
 (左辺)/m → (1/π)∫[0,π] sin(x)dx = 2/π.  (m→∞)
657656:2008/11/20(木) 22:30:32
訂正
〔補題〕 a/π ≠有理数 ならば、
658132人目の素数さん:2008/11/24(月) 20:12:49
f(x)=x^2-2mx+m+6 とする。

(1) すべてのxの値に対してf(x)≧0となる
  定数mの値の範囲は-2≦m≦3である。

(2) 0≦x≦8のすべてのxの値に対してf(x)>0となる
  定数mの値の範囲は-6<m<3である。

これを証明してください。
659132人目の素数さん:2008/11/24(月) 22:18:03
>>658
お前は勉強をやめた方がいい。
660132人目の素数さん:2008/11/24(月) 23:23:19
>>658
荒ら砂!
質問は質問スレに池!
661132人目の素数さん:2008/11/26(水) 01:34:42
http://science6.2ch.net/test/read.cgi/math/1227275638/700

1/π<x<πの時、
sinx・sin(1/x)の最大値を求めよ
662132人目の素数さん:2008/11/26(水) 21:32:47
うるさい。
663132人目の素数さん:2008/11/26(水) 22:31:04
   r;;;;;ノヾ        >>662
   ヒ‐=r=;'     ∬   口を慎みたまえ!
   'ヽニ/  っ━~~    君は不等式王の前にいるのだぞ!
 _と~,,  ~,,,ノ_  ∀   
    ミ,,,,/~). │ ┷┳━ 
  ̄ ̄ ̄.じ'J ̄ ̄| ┃
  ̄ ̄ ̄ ̄ ̄ ̄ ̄ ┻
664132人目の素数さん:2008/11/27(木) 09:56:45
nCrオタ向け
納k=0,n](k+2)*(k+1)*[2k+1]C[k]=?
665132人目の素数さん:2008/11/27(木) 23:22:39
>>661

[751] 微分法を使う。
 g(t) = log(sin(e^t)) とおくと
 g '(t) = (e^t)/tan(e^t)  は単調減少(*) 
 g "(t) = −(e^t){1 - sin(e^t)cos(e^t)}/{sin(t)}^2 < 0,
∴ f は上に凸。
 log(与式) = f(log(x)) + f(-log(x)) ≦ 2f(0) = log{sin(1)^2}

(*) {x/tan(x)} ' = 1/tan(x) - x/{sin(x)^2} = {sin(x)cos(x)-x}/{sin(x)^2} <0,
  より、x/tan(x) は単調減少。

[763] 無限乗積表示(オイラー積表示)を使う。
sin(x) = x・Π[n=1,∞) {1−(x/nπ)^2},
 {1−(x/nπ)^2}{1−1/(nπx)^2} = {1−1/(nπ)^2}^2 −(1/nπ)^2 (x−1/x)^2 ≦ {1−1/(nπ)^2}^2,
 等号成立は x=1 のとき,
∴ (与式) ≦ {sin(1)}^2.
666132人目の素数さん:2008/11/27(木) 23:24:24
>>664
それ本当に求まるのか?
Mathematicaにやらせてみたら
-((2 + n)*(3 + n)*Gamma[5 + 2*n]*Hypergeometric2F1Regularized[1, 5/2 + n, 3 + n, 4] +
2*Gamma[7 + 2*n]*Hypergeometric2F1Regularized[2, 7/2 + n, 4 + n, 4] +
8*(7 + 2*n)*Gamma[6 + 2*n]*Hypergeometric2F1Regularized[3, 9/2 + n, 5 + n, 4])/(2*Gamma[3 + n])
になったぞ。
667665:2008/11/27(木) 23:57:46
>>665 訂正

 f(x) = log(sin(x)) なので、
 log(与式) = f(x) + f(1/x) = g(log(x)) + g(-log(x)) ≦ 2g(0) = 2f(1) = log{sin(1)^2},
668132人目の素数さん:2008/11/28(金) 05:13:49
みなさんは不等式の必須手法みたいなのを何で学びましたか?
669132人目の素数さん:2008/11/28(金) 23:42:33
>>668
おまえには教えてやらねーよ!
670132人目の素数さん:2008/11/29(土) 00:20:54
不等式を制する者は解析を制する。
671132人目の素数さん:2008/11/29(土) 12:07:58
△ABC の辺 a、b、c に対して、次式を示せ

  3abc ≧ (b+c-a)a^2 + (c+a-b)b^2 + (a+b-c)c^2


   ∧_∧ 
  _ ( ゚∀゚)            たぶん、出したことないと思う…
 |≡(つc□≡|
 `T ̄∪∪ ̄T
゙゙゙゙゙゙゙゙゙゙゙゙゙゙゙゙゙゙゙゙゙゙゙゙゙゙゙゙゙゙゙
672132人目の素数さん:2008/11/29(土) 18:50:59
>>664
 (k+2)(k+1) = (2k+3)(2k+2)/3 - (k+1)k/3,

 (k+2)(k+1)*C[2k+1,k] = (1/3)(2k+3)(2k+2)*C[2k+1,k] - (1/3)(2k+1)(2k)*C[2k-1,k-1]
            = (1/3)(k+2)(k+1)*C[2k+3,k+1] - (1/3)(k+1)k*C[2k+1,k],

 (与式) = (1/3)(2n+3)(2n+2)*C[2n+1,n] = (1/3)(n+2)(n+1)*C[2n+3,n+1].


>>671
三角不等式の束縛からのがれるため
b+c-a = a' >0, c+a-b = b' >0, a+b-c = c' >0,
とおく。条件は a', b', c' >0 だけになった。両辺に
 a = (b'+c')/2, b = (c'+a')/2, c = (a'+b')/2,
を代入すれば、
 (左辺) - (右辺) = (3/8)(st-u) - (1/4)(3u+st) = (1/8)(st-9u) ≧0,
いつものように s = a'+b'+c' = a+b+c, t = a'b' + b'c' + c'a', u = a'b'c' とおいた。
等号成立は a'=b'=c' すなわち a=b=c のとき。

ハァハァ
673132人目の素数さん:2008/11/29(土) 19:19:09
>>671

移項したらSchur不等式・・・・
 (左辺) - (右辺) = a(a-b)(a-c) + b(b-c)(b-a) + c(c-a)(c-b) = F_1 ≧0,
三角条件なくても成立・・・・
674132人目の素数さん:2008/11/29(土) 20:11:59
さすが。
675132人目の素数さん:2008/11/29(土) 21:13:11
問題を作ったときには
(左辺)-(右辺)=(a-b)^2・(a+b-c)/2+(b-c)^2・(b+c-a)/2+(c-a)^2・(c+a-b)/2
から導いたと思われ
676671:2008/11/29(土) 22:30:45
毎度ながら、100歩前を行くレスに感心。
ありがとうございます。
677132人目の素数さん:2008/12/01(月) 20:28:22
1 ≤ a,b,c ≤ 2 のとき
(a+b+c)(1/a+1/b+1/c) ≥ 6(a/(b+c)+b/(c+a)+c/(a+b))

たのもー( ^ิิ,_ゝ^ิ)
678132人目の素数さん:2008/12/01(月) 22:14:56
homogeneousなのに何で1<=a,b,c<=2が必要?
679132人目の素数さん:2008/12/02(火) 16:40:46
1≦a,b,c≦2がないと問題が成り立たないから
680132人目の素数さん:2008/12/02(火) 20:31:05
別にk≦a,b,c≦2kでも良いけど
いずれにせよ或る一定範囲内に三つとも入ってないといけなくて
a=b=1、c=1000とかそういうのはダメってことでしょ。

それぞれa/kとかで置き換えて要らないkを消去したのが問題文と。
681132人目の素数さん:2008/12/03(水) 03:15:18
>>677 , 679
 >>341 の [A.435] でつね。

>>394 いわく、
 とりあえず、>>373-374 が解ければ [A.435] が解けることが分かった。

>>576 は微分法(未定乗数法)でそれを解こうとしたようだが・・・・
682132人目の素数さん:2008/12/03(水) 11:48:57
もっとすきっとした解法はないもんかねぇ
683132人目の素数さん:2008/12/03(水) 19:23:45
>>576 を高校レベルで解いたのが
>>583-585>>588-589
684132人目の素数さん:2008/12/03(水) 23:40:27
>>588
 r-q 平面のグラフが見たい・・・・
685132人目の素数さん:2008/12/07(日) 00:24:18
>>679
>>680
誤解してた、すまない。
686132人目の素数さん:2008/12/07(日) 04:23:34
1)a,b,cが正の実数のとき
a/(b+c^2)+b/(c+a^2)+c/(a+b^2)≧9/(a+b+c+3)
を示せ

2)a,b,cが相異なる実数のとき
{(4a-3b)/(a-b)}^2+{(4b-3c)/(b-c)}^2+{(4c-3a)/(c-a)}^2≧25
を示せ

3)a,b,cが正の実数のとき
{(b+c-a)^2}/{(b+c)^2+a^2}+{(c+a-b)^2}/{(c+a)^2+b^2}+{(a+b-c)^2}{(a+b)^2+c^2}≧3/5
を示せ(日本数学五輪1997)
687132人目の素数さん:2008/12/08(月) 01:45:32
688132人目の素数さん:2008/12/08(月) 03:44:10
>>686
(2)(3)は、ともに凸不等式に帰着できた
(1)に苦戦中
689132人目の素数さん:2008/12/08(月) 03:46:55
>>686
(3)は公式解答は汚い解法だったんだがエレガントに解けるのかな
俺はシュワたんで失敗した
690132人目の素数さん:2008/12/08(月) 04:58:11
>>686
(3)
Σはcycとして
a+b+c=1とおくと

与式
⇔Σ[(1-2a)^2/{(1-a)^2+a^2}]≧3/5
⇔Σ[1/(2a^2-2a+1)-9/5]≦0
⇔Σ[25/(2a^2-2a+1)-45-18(3a-1)]≦0 (∵Σ(3a-1)=3(a+b+c)-3=0)
⇔-Σ[(3a-1)^2*(6a+1)/{a^2+(1-a)^2}]≦0

a,b,c> 0よりこれは正しい。

691132人目の素数さん:2008/12/11(木) 13:39:30
実数上の任意の確率変数 X と、0以上の実数値が値域の関数f,gに関して
E[ f(X)g(X) ] ≦ E[ f(X) ] E[ g(X) ] が成り立つための関数f,gの条件として
∀x,y f(x)≦f(y) → g(x)≧g(y) が十分条件であると予想しているのですが
証明の仕方がわかりません。お願いします。
692132人目の素数さん:2008/12/11(木) 14:36:06
693132人目の素数さん:2008/12/11(木) 16:39:46
まさかのpdf、ありがとうございます。
自分の頭で理解できるか不安ですが
じっくり読まさせていただきます。
694132人目の素数さん:2008/12/11(木) 17:51:33
定理1のX(Y-E[Y])]≧X(x0)(Y-E[Y]) による
E[X(Y-E[Y])]≧E[X(x0)(Y-E[Y])]=0がコツですね。
どうもありがとうございました。
695132人目の素数さん:2008/12/18(木) 00:22:39
数蝉2月号は「不等式の世界」
http://www.nippyo.co.jp/magazine/maga_susemi.html

不等式ヲタとしては、買わねばなるまいな…
(*゚∀゚)=3 ハァハァ…
696132人目の素数さん:2008/12/18(木) 00:37:29
>>695
このスレの住民が満足できる内容ならいいね。
参考文献に付け加えられるくらいの内容を気盆濡!
697132人目の素数さん:2008/12/18(木) 22:53:51
>>588

Q,R を >>589 のようにおくと

(判別式) = 27R^2 - 4(-Q)^3
 = (q-1+2r){4q^2 +(32-4q)r -(11+q)r^2 +2r^3}   >>585
 = (q-1+2r){[2q - r(r+4)/4]^2 - (1/16)r(r-8)^3},
∴ r<8 には求める領域はない。

 rを固定したときの q の下限および上限は
 q_min = [r(r+4) - (√r)(r-8)^1.5]/8,
 q_max = min{[r(r+4) + (√r)(r-8)^1.5]/8, 2r-3}
    = [r(r+4) + (√r)(r-8)^1.5]/8     (8≦r≦9)
    = 2r-3    (r≧9)
 rが大きいほど細く鋭くなる。    (素手で触るな)

 r>9 のとき q < 2r-3 = (1/6)r(r+1) -(1/6)(r-2)(r-9) < (1/6)r(r+1)
 8<r<9 についても同様。
698132人目の素数さん:2008/12/19(金) 22:34:05
>>588, 684, 697
 8<r<9 のときは、
 r-8 < r/9,
 q < [r(r+4) + (√r)(r-8)^1.5]/8 < [r(r+4) + (1/3)r(r-8)]/8 = (1/6)r(r+1),

-------------------------------------

 (2r-3) - q_min = 8(r-9)/{r^2 -12r +24 +(√r)(r-8)^1.5} → 0 (r→∞)
699132人目の素数さん:2008/12/24(水) 12:09:03
>>677
(a+b+c)(1/a + 1/b + 1/c) - 9 = Σ(a-b)^2/ab
6(Σa/(b+c)) - 9 = 3Σ(a-b)^2/((a+c)(b+c))
より
Σ(a-b)^2/ab ≧ 3Σ(a-b)^2/((a+c)(b+c))
(a, b, c∈[1, 2])
を証明すればよい.
ここで,
S[a] = 1/bc - 3/((a+b)(a+c))
S[b] = 1/ca - 3/((b+c)(b+a))
S[c] = 1/ab - 3/((c+a)(c+b))
とおく.
また, a≧b≧c とおいても一般性を失わない。
S[a] = (a^2 + ab + ac - 2bc)/(bc(a+b)(a+c)) ≧ 0
(∵ ab≧bc, ac≧bc)
S[b] = (ab + b^2 + bc - 2ac)/(ca(b+c)(b+a)) ≧ 0
(∵ ab≧ac; a≦2, y, z≧1 より b+c≧a; よって b(b+c)≧ac)
ここで, S[b]+S[a]≧0 は明らかなので, S[b]+S[c]≧0 を証明する.
S[b]+S[c]
= (bc(b+c)^2 + a^2(b^2-4bc+c^2) + a(b^3+c^3))/(abc(a+b)(b+c)(c+a))
= (bc(b+c)^2 + a^2(b^2-4bc+c^2) + a(b+c)(b^2-bc+c^2))/(abc(a+b)(b+c)(c+a))
≧ (a^2(bc) + a^2(b^2-4bc+c^2) + a^2(b^2-bc+c^2))/(abc(a+b)(b+c)(c+a))
= (2a^2 (b-c)^2)/(abc(a+b)(b+c)(c+a))
≧ 0
よって SOS より ΣS[a](b-c)^2 ≧ 0.
等号成立は a=b=c or b=c=1, a=2 (cyc).
700132人目の素数さん:2008/12/24(水) 14:12:31
(*゚∀゚)=3 ハァハァ…
701132人目の素数さん:2008/12/24(水) 14:30:59
ネ申
702132人目の素数さん:2008/12/24(水) 15:50:36
自演乙
703132人目の素数さん:2008/12/24(水) 16:25:07
このスレで自演とか言ってるやつは新参者
自演は初代スレから恒例だ!
もうね アホガド バナナかと… ( ゚∀゚)
704132人目の素数さん:2008/12/24(水) 16:40:29
飯島愛死亡だとよ
705132人目の素数さん:2008/12/24(水) 16:49:15
>>704
死因は何?
706132人目の素数さん:2008/12/24(水) 16:52:47
1:春デブリφ ★[sage]
2008/12/24(水) 16:29:31 ID:???0
 元タレント・飯島愛さんが都内のマンションで死亡。

■ソース(日テレニュース24)
http://www.news24.jp/125696.html
※有志によるキャプチャ画像
http://tvde.web.infoseek.co.jp/cgi-bin/jlab-dat/s/347618.jpg
■前スレ(1の立った日時 12/24(水) 16:14:24)
http://mamono.2ch.net/test/read.cgi/newsplus/1230102864/

【訃報】飯島愛さん、東京都内のマンションで死亡★2
http://mamono.2ch.net/test/read.cgi/newsplus/1230103771/
707132人目の素数さん:2008/12/24(水) 16:59:52
   /  ≧ \
 /   _ノ  \
 |   ( ●)(●)  <おっと、スレ違いな発言はそこまでだ!
. |     (__人__)____
  |     ` ⌒/ ─' 'ー\
.  |       /( ○)  (○)\
.  ヽ     /  ⌒(n_人__)⌒ \
   ヽ   |、    (  ヨ    |
   /    `ー─−  厂   /
   |   、 _   __,,/     \
708132人目の素数さん:2008/12/25(木) 01:24:11
x,y,n∈N,(1/x)+(1/y)<1/n

max{(1/x)+(1/y)}=(n^2+2n+2)/{(n+1)(n^2+n+1)}
709132人目の素数さん:2008/12/25(木) 02:06:44
>>708
なん…だと!
710132人目の素数さん:2008/12/28(日) 08:06:19
>>708
どうやったんだよ
711132人目の素数さん:2008/12/28(日) 13:22:09
>>678,685
誤解してた、すまない。
三角不等式で十分だった。


>>699 の証明によれば・・・・
bはaとcの間にあるとしても、一般性を失なわない。
 c-a = (c-b) + (b-a) より
(左辺) - (右辺) = S[a](b-c)^2 + S[b](c-a)^2 + S[c](a-b)^2
        = (S[a]+S[b])(b-c)^2 + 2S[b](c-b)(b-a) + (S[b]+S[c])(a-b)^2,
また (c-b)(b-a)≧0,
したがって、S[a]+S[b] ≧0, S[b] ≧0, S[b]+S[c] ≧0 を示せばよい。
 S[a] + S[b] = {2(c^2)(a-b)^2 + (a+b-c)[(b+c)a^2 + (c+a)b^2]} / {abc(a+b)(b+c)(c+a)},
 S[b] = {b(a+b+c) -2ac} / {ca(a+b)(b+c)}
  = {2b^2 -Mm + (c-b)(b-a)} / {ca(a+b)(b+c)}  (← {M,m}={a,c}, m≦b≦M とした.)
  = {b(b+m-M) + (b-m)(b+M)^+ (c-b)(b-a)} / {ca(a+b)(b+c)},
 S[b] + S[c] = {2(a^2)(b-c)^2 + (b+c-a)[(c+a)b^2 + (a+b)c^2]} / {abc(a+b)(b+c)(c+a)},
これらが負にならないためには、a+b-c≧0, b+c-a≧0 (三角不等式)があれば十分。
712132人目の素数さん:2008/12/29(月) 18:19:53

[A.435] の拡張                      >>341
a,b,c が三角形の3辺をなすとき、 
 (a+b+c)(1/a + 1/b + 1/c) ≧ 6{a/(b+c) + b/(c+a) + c/(a+b)} ≧ 9,
 等号成立は (a,b,c) = (k,k,k), (2k,k,k), (k,2k,k), (k,k,2k) のとき。

そこで >>672 に習って
 b+c-a = a', c+a-b = b', a+b-c = c', a+b+c = a'+b'+c' = s,
とおく。上式に
 a = (b'+c')/2 = (s-a')/2,
 b = (c'+a')/2 = (s-b')/2,
 c = (a'+b')/2 = (s-c')/2,
を代入すると・・・

[A.435'] (正準形)
a',b',c' ≧0 のとき
 6 + 2{a'/(b'+c') + b'/(c'+a') + c'/(a'+b')} ≧ 6{(s-a')/(s+a') + (s-b')/(s+b') + (s-c')/(s+c')} ≧ 9.
 ここに s = a'+b'+c'.
 等号成立は (a',b',c') = (k,k,k), (0,2k,2k), (2k,0,2k), (2k,2k,0) のとき。

【系】
a,b,c ≧0 のとき
 (a+b+c)(1/a + 1/b + 1/c) ≧ 6{a/(b+c) + b/(c+a) + c/(a+b)} ≧ 18{(s-a)/(s+a) + (s-b)/(s+b) + (s-c)/(s+c) -1} ≧ 9.

http://science6.2ch.net/test/read.cgi/math/1179000000/677
713712:2008/12/29(月) 18:24:36
訂正、すまそ。

【系】
a,b,c が三角形の3辺をなすとき
 (a+b+c)(1/a + 1/b + 1/c) ≧ 6{a/(b+c) + b/(c+a) + c/(a+b)} ≧ 18{(s-a)/(s+a) + (s-b)/(s+b) + (s-c)/(s+c) -1} ≧ 9.
714Shapiro:2009/01/03(土) 19:56:03
>>712
 Sh_3(x,y,z) = x/(y+z) + y/(z+x) + z/(x+y) - 3/2,
とおく。

[A.435']
a',b',c' ≧ 0 のとき
 Sh_3(a',b',c')/Sh_3(b'+c', c'+a', a'+b') ≧ 3,
715132人目の素数さん:2009/01/16(金) 19:05:31
数セミ2月号出たね
716132人目の素数さん:2009/01/16(金) 23:11:26
>>715
うちの田舎は入荷数が少ないので予約注文したんだけど、
まだ連絡が来ないぜ… ('A`)
717132人目の素数さん:2009/01/17(土) 23:53:44
>>598 (593,596)

〔問題〕
abc = d^3, √3 -1 ≦ d ≦ (1+√3)/2 なる正の実数の組(a,b,c)に対して、次を示せ。
 1/{a(b+1)} + 1/{b(c+1)} + 1/{c(a+1)} ≧ 3/{d(d+1)},

(略証)
 abc = … と来たら a=d・z/y, b=d・x/z, c=d・y/x とおく。(これ定石)
ただし x,y,z >0

 (左辺) - (右辺) = (1/d){y/(dx+z) + z/(dy+x) + x/(dz+y) - 3/(d+1)}
  = {d(d+1)・S -(3d^2 -d-1)・T1 -d(-d^2 -d+3)・T2 -(d+1)(d-1)^2・(3xyz)} / {d(d+1)(dx+z)(dy+x)(dz+y)},
ここに、S = x^3 + y^3 + z^3, T1 = yz^2 + zx^2 + xy^2, T2 = zy^2 + xz^2 + yx^2 とおいた。

・√3 -1 ≦ d < 1.30 のとき
 {(左辺)-(右辺)}d(d+1)(dx+z)(dy+x)(dz+y) = {d(d^2 +2d-2)(S-T1) + d(-d^2 -d+3)(S-T2) + (d+1)(d-1)^2・(T1-3xyz)} ≧ 0,

・0.77 < d ≦ (1+√3)/2 のとき
 {(左辺)-(右辺)}d(d+1)(dx+z)(dy+x)(dz+y) = {(3d^2 -d-1)(S-T1) + (-2d^2 +2d+1)(S-T2) + (d+1)(d-1)^2・(T2-3xyz)} ≧ 0,
718717:2009/01/17(土) 23:56:03
>>598

〔補題〕↑ のようにおくとき
 S ≧ T1 ≧ 3xyz, S ≧ T2 ≧ 3xyz,

(左側) チェビシェフ不等式から、あるいは
 (y^3 + 2z^3)/3 - yz^2 = (1/3)(y+2z)(y-z)^2 ≧ 0,  (相加・相乗平均)
これを循環的にたすと S - T1 ≧ 0,
 (2y^3 + z^3)/3 - zy^2 = (1/3)(2y+z)(y-z)^2 ≧ 0,  (相加・相乗平均)
これを循環的にたすと S - T2 ≧ 0,

(右側)も 相加・相乗平均。
719132人目の素数さん:2009/01/20(火) 00:55:25
>>2に追加

数学セミナー vol.48 no.2_569,日本評論社,2009年2月号
720132人目の素数さん:2009/01/28(水) 07:31:59
721132人目の素数さん:2009/01/28(水) 13:41:41
>>720     ハァハァ   ∩           不等式と聞ゐちゃぁ
               ( ⌒)_   ∩_ _     黙っちゃゐられねゑ…
グッジョブ!! .___  //,. ノ≧ \ .i .,,E)__   
     / nCr  \| / /\ ./ |/ / cos \ ハァハァ    //
  _n  .|::::\ ./  |/ /(● (● | ノ\ ./ |       / /___
 ( l  |::●) ●) .| /:::... .ワ ....ノ/(● (● |      ./  /   Σ \
  \ \ヽ:::::.∀   .ノ      /ヽ:::::... .▽....ノ  n   / ∩.|:::: \ ./  |
   ヽ__ ̄   ノ ヽ      |  ̄     \    ( E) / .| | | (● (●)|_
     /    /    \    ヽ フ    / ヽ ヽ_//.// | | ヽ:::::. へ  ノ/
722132人目の素数さん:2009/01/30(金) 20:57:27
非等式と不等式の違いはなんですか。
723132人目の素数さん:2009/01/30(金) 21:43:48
a,b,c>0, a+b+c=1 のとき
(a+1/b)(b+1/c)(c+1/a)≧(41/5)(a/b+b/c+c/a-3)+1000/27
724132人目の素数さん:2009/02/05(木) 00:42:33
他スレで見かけたお。

次の等式を証明せよ。
nHr=Σ[m=1,n]{Σ[l=1,m]…(Σ[j=1,k]j) }
※右辺のΣの数はr-1個

↓例
8H3=Σ[m=1,8](Σ[l=1,m]l)
5H4=Σ[m=1,5]{Σ[l=1,m](Σ[k=1,l]k)}

面白い問題おしえて〜な 十五問目
http://science6.2ch.net/test/read.cgi/math/1231110000/99
725132人目の素数さん:2009/02/07(土) 20:47:34
このスレを見てる人は、もれなくそっちも見てると思
726132人目の素数さん:2009/02/07(土) 23:07:15
>>725
分かってないな〜チミ
そんなこと百も承知の助さ〜
ここに保存しておけば、後で探すときにどのスレだったか困らないだろ〜う
727132人目の素数さん:2009/02/08(日) 03:05:54
次の不等式は一見シンプルにみえますが、
左辺は対称式でないせいか、(私には)証明がうまくいかないです。

任意の非負実数a,b,cに対して、次が成立する。
{(a+2b)(b+2c)(c+2a)}^2 ≧ 27(ab+bc+ca)^3

もしよろしければ、どなたかご教授おねがい致します。
728132人目の素数さん:2009/02/08(日) 04:40:48
s、t、uでズコバコするといいよ
729132人目の素数さん:2009/02/08(日) 09:00:45
>>727
相加相乗から
(a+a+b)/3 ≧ (a*a*b)^(1/3)
2a+b ≧ 3(a^2b)^(1/3)
以下略
730132人目の素数さん:2009/02/08(日) 09:01:38
問題見まちがった
731132人目の素数さん:2009/02/08(日) 19:32:27
>>727 力づくで解いた

a+b+c=0 のとき題意は明らかなので a+b+c > 0 としてよい
左辺-右辺 は同次式なので a+b+c=3 とする
a-1, b-1, c-1 のうち符号が同じ(または片方が 0)のものを
a-1, b-1 としても一般性を失わない
つまり (a-1)(b-1)≧0 とする

a = 1+x, b = 1+y, c = 1-x-y とする
a,b,c≧0 より、 x,y≧-1, x+y≦1 …(1)
(a-1)(b-1)≧0 より、 xy≧0 …(2)
(1)(2) より、 -1≦x,y≦ 1 …(3)

s = x-y, u = x^2+xy+y^2 として
((a+2b)(b+2c)(c+3a))^2 - 27(ab+bc+ca)^3
= 9(3-u){3u(3-u) - 2s(3u-s^2)} + s^2(3u-s^2)^2
(3) より 3-u≧0 なので { } の中が非負になることを言えばよい
3u-s^2 = 2x^2+5xy+2y^2 は (2) より非負
s<0 のとき { } の中は明らかに非負なので、以降
s ≧ 0 …(4)
とする。また (2)(3) より s ≦ 1 …(5)

t = xy として
{ } の中 = -27t^2 + 9t(3-2s-2s^2) + s^2(9-4s-3s^2) …(6)
(6) で s を一定と見て t を動かす。t の動く範囲は (2)(3)(4) より
0 ≦ t ≦ 1-s
t^2 の係数が負なので、端点の t = 0, 1-s について (6) が
非負になることを確認すればよい
t = 0 のとき (6) が正なのは (4)(5) より明らか
t = 1-s のとき
(6) = 9(1-s)^2 + s^2(5-3s)
これが正なのは (5) より明らか■
732731:2009/02/08(日) 22:58:40
いろいろ間違ってた
× ((a+2b)(b+2c)(c+3a))^2 - 27(ab+bc+ca)^3
○ ((a+2b)(b+2c)(c+2a))^2 - 27(ab+bc+ca)^3

あと、下から1行目と4行目の「正」は「非負」に直しといて

因みに、等号成立の必要条件は
a=b=c ∨ 3-u=0 ∨ s=t=0
なのは議論をたどれば分かる
結局、等号成立条件は
a=b=c か a,b,c のうちふたつ以上が 0 であること
733132人目の素数さん:2009/02/08(日) 23:03:07
V
734132人目の素数さん:2009/02/09(月) 16:59:31
>>727
x=a+2b, y=b+2c, z=c+2a とおくと、
右辺 = [ (-2/9)(x+y+z)^2 + xy+yz+zx ]^3.

X=[(x^2)/(yz)]^(1/3), Y=[(y^2)/(zx)]^(1/3), Z=[(z^2)/(xy)]^(1/3)
とし、X, Y, Z に関する相加相乗調和平均をそれぞれ A, G(=1), H とすると、
右辺/左辺 = (-2A^2 + 3/H)^3.
735734:2009/02/09(月) 20:55:19
とはしてみたものの、もうだめかもわからんね
736132人目の素数さん:2009/02/09(月) 21:01:40
5行で証明できたのかと思って一瞬驚愕した
737132人目の素数さん:2009/02/11(水) 22:32:08
>>734-735
s = a+b+c, t = ab+bc+ca, u=abc とおくと     >>728
 (a+2b)(b+2c)(c+2a) = 3st + (a-b)(b-c)(c-a) = 3st + ,
だから↓を示せればいいのだが。。。

〔補題〕
a,b,c≧0 のとき |處 ≦ (3t/2s)(s^2 -3t),
ここに、 = (a-b)(b-c)(c-a).
738132人目の素数さん:2009/02/12(木) 00:57:10
107 名前:132人目の素数さん[sage] 投稿日:2009/02/11(水) 21:17:45
自然数nについての不等式
(n^n)/(e^(n-1))≦n!≦(n^(n+1))/(e^(n-1))
を証明せよ。
ただしeはネイピアの数

108 名前:132人目の素数さん[sage ] 投稿日:2009/02/11(水) 23:35:25
>>107
n=1 のときは 等号成立。
n>1 のときは log(1+x) < x を使う。
 k・log(k) - (k-1)log(k-1) -1 = k・log(k) - (k-1){log(k) - log(k/(k-1))} -1
 = log(k) + (k-1)log(1 + 1/(k-1)) -1 < log(k) +1 -1 = log(k),
 (k+1)log(k) - k・log(k-1) -1 = (k+1)log(k) - k{log(k) + log((k-1)/k)} -1
 = log(k) - k・log(1 - 1/k) -1 > log(k) +1 -1 = log(k),
k=2,3,・・・,n について たす。
 n・log(n) - (n-1) < log(n!) < (n+1)log(n) - (n-1),
739132人目の素数さん:2009/02/14(土) 00:12:52
>>737
〔補題〕 a,b,c≧0 のとき |處 ≦ (2/√3)(t/s)(s^2 -3t),

(略証)
min(a,b,c) = m とおき、{a,b,c} = {m, m+x, m+x+y} とする。(x,y≧0)
然らば、 |處 = xy(x+y), s = 3m+2x+y, t = 3m^2 + 2m(2x+y) + x(x+y), s^2 -3t = x^2 +xy +y^2,
∴ t(s^2 -3t) - ((√3)/2)s|處 = 3m^2・(x^2 +xy +y^2) + m・{4x^3 + 3(1-(√3)/2)xy(x+y) +2y^3} + x(x+y){x - ((√3 -1)/2)y}^2 ≧0,
等号成立は m=0 かつ x/y = (√3 -1)/2 のとき。

>>727
 (左辺) - (右辺) = (3st+)^2 - 27t^3 = 9(t^2)(s^2 -3t) +6st + 竸2
 ≧ (9-4√3)(t^2)(s^2 -3t) ≧ 0,
740132人目の素数さん:2009/02/21(土) 03:25:55
〔問題〕
n は自然数, N = 3n(2n^2 +1) のとき
 {N + (n^2)/N}^2 < (2n^2 +1)(3n^2 +1)(6n^2 +1) < {N + 1/(6n)}^2,

(略証)
・左側
 (左辺) = N^2 + 2n^2 + {(n^2)/N}^2 < N^2 +2n^2 +1 = N{N + 1/(3n)} = (中辺),
・右側
 (中辺) = N{N +1/(3n)} < {N +1/(6n)}^2,

http://science6.2ch.net/test/read.cgi/math/1219648297/555
京大入試作問者スレ@
741132人目の素数さん:2009/02/21(土) 03:52:11
(*゚∀゚)=3 ハァハァ…
742132人目の素数さん:2009/02/21(土) 18:13:33
不等式に興味が出たんだけど
とりあえずモノグラフ注文してみたよ
まとめwikiの「よく使う不等式」すらわかんないレベルだけどねorz
群とかわかんないし・・・
743132人目の素数さん:2009/02/26(木) 13:49:15
>>742
分からないことを自分で調べていけば一生楽しめるぜ
744132人目の素数さん:2009/02/26(木) 23:01:51
前スレ49が気になったので注文した…
楽しみだぜ!

(*゚∀゚)=3 ハァハァ…
745132人目の素数さん:2009/03/02(月) 17:46:17
ミラーみれなくね?
746132人目の素数さん:2009/03/03(火) 23:52:29
747132人目の素数さん:2009/03/04(水) 00:07:59
>>745
Yahoo ブリーフケースの有料化に伴い,
http://cid-d357afbb34f5b26f.skydrive.live.com/browse.aspx/.Public/
に避難しました。
748132人目の素数さん:2009/03/04(水) 00:23:39
>>746
グッジョブ!
749132人目の素数さん:2009/03/04(水) 00:32:35
a≧b≧0,c≧d≧0のとき

√(a^2+ab+b^2)+√(c^2+cd+d^2)≧√(a^2+ac+c^2)+√(b^2+bd+d^2)

750132人目の素数さん:2009/03/04(水) 00:47:23
>>749は今月号の大数の宿題。
ネタバレになるから回答しないように。
751132人目の素数さん:2009/03/04(水) 01:18:18
>>749
簡単すぎ
752132人目の素数さん:2009/03/04(水) 01:38:52
>>749
過去に解いたことがある
入試問題かな?
753132人目の素数さん:2009/03/04(水) 03:06:05
>>749
泥沼にはまった予感
754132人目の素数さん:2009/03/05(木) 01:59:49
√(x^2)+√(1−x^2)
の最大値の求め方って何通りありますかね?
755132人目の素数さん:2009/03/05(木) 02:21:41
0
756132人目の素数さん:2009/03/05(木) 17:02:58
鳥取市の誘致企業リコーマイクロエレクトロニクスにアルバイトに行っていた。
勤務態度不良でリコーのアルバイトをクビ同然で辞めた。

その後、鳥取市のテスコという工場に勤め真面目に働いていた。
「真面目に働いているのはリコーに対する報復(あてつけ?)」という噂でテスコをクビになった。
直後、テスコの社長から雇用保険の書類をとりに来るよう泣きそうな声で電話があった。
噂は嘘だと知ったのだろう。

雇用保険の手続きのため職安に行った。
職安の次長と相談すると、口止めをされた。
職安と会社は連絡を取り合っていたらしい。

しかし噂は狭い鳥取市である程度広がっているようだ。

リコーマイクロエレクトロニクスに電話を掛けた。
「君はうちのような一流企業が組織ぐるみでやったとでも思っているのかね?」
「そんなことはありませんけど」
「じゃあ会社には関係ないじゃないか」

しかし公的機関(職安)も巻き込んだ組織ぐるみの人権侵害の揉み消しである。
757132人目の素数さん:2009/03/05(木) 21:51:51
(1) 実数xが-1<x<1,x≠0を満たすとき,次の不等式を示せ.

(1-x)^{1-(1/x)}<(1+x)^(1/x)

(2) 次の不等式を示せ.

0.9999^101<0.99<0.9999^100
758132人目の素数さん:2009/03/05(木) 22:02:25
>>757
なんで命令形なの?むかつくんだけど
759132人目の素数さん:2009/03/05(木) 22:38:50
>>757-758
今年の東大入試の第5問のコピペだから。
760132人目の素数さん:2009/03/05(木) 23:20:42
>>758
消えろ!カーッ(゚Д゚≡゚д゚)、ペッ
761132人目の素数さん:2009/03/06(金) 12:53:46
>>757
これに30分もかけなければ今頃俺は
762132人目の素数さん:2009/03/07(土) 00:22:41
>>757

(1) f(x)=log(1-x) は上に凸だから、平均変化率 g(x)={f(x)-f(0)}/(x-0) は単調減少。
 x<0 のとき g(x^2) > g(x),
 x>0 のとき g(x^2) < g(x),
よって
 x・g(x^2) < x・g(x),
 f(x^2)/x < f(x),
 (1/x)log(1-x^2) < log(1-x),

763132人目の素数さん:2009/03/07(土) 00:43:26
>>757

(1) f(x)=log(1-x) は上に凸で f(0)=0.
 平均変化率 g(x)={f(x)-f(0)}/(x-0) は単調減少。
 x<0 のとき g(x) > g(x^2),
 x>0 のとき g(x) < g(x^2),
よって
 x・g(x) < x・g(x^2),
 f(x) < f(x^2)/x,
 log(1-x) < (1/x)log(1-x^2),
 1-x < (1-x^2)^(1/x),

(2) (1-x^2)^{1+(1/x)} < 1-x < (1-x^2)^(1/x),
を示す....
764132人目の素数さん:2009/03/07(土) 02:06:15
みんな解いた問題って保存してるの?
765132人目の素数さん:2009/03/07(土) 02:11:03
>>757を改めて試験中の様にエレガントに解いてみせようとしたら間違いに気づいた。
落ちてそうだから死にたい。
766132人目の素数さん:2009/03/07(土) 08:45:08
>>764
最近は時間がなくてのぉ…
このスレを保存するだけで精一杯さ ('A`)
767132人目の素数さん:2009/03/07(土) 22:26:28
>>763 (2) の左側

(1) で x→-x として, 1+x = (1-x^2)/(1-x) を使えば出る。
768132人目の素数さん:2009/03/08(日) 01:00:22
>>754
 f(x) = |x| + √(1-x^2),
とおく。

(1) f(x)^2 = 1 + √{4x^2・(1-x^2)} = 1 + √{1 - (1-2x^2)^2} ≦ 2,
等号成立は x = ±1/√2 のとき。

(2) x=cosθ (0≦θ≦π/2) とおく。
 f(x) = cosθ + sinθ = (√2)sin(θ + π/4) ≦ √2,
等号成立は θ=π/4 のとき。
769132人目の素数さん:2009/03/08(日) 16:10:21
コーシー・シュワルツから
(1*|x|+1*√(1-x)^2)^2≦(1+1)(x^2+(1-x^2))=2
770132人目の素数さん:2009/03/08(日) 19:05:38
>>757
もし(2)の不等式だけ出たらやっぱ(1)の不等式を示すことに帰着されるんかな
771132人目の素数さん:2009/03/08(日) 19:57:44
不等式は嫌いなんだ
772132人目の素数さん:2009/03/08(日) 23:43:39
>>771
ありえん!?
一度医者に見てもらったほうがいい…
773132人目の素数さん:2009/03/09(月) 00:02:16
この本に出てくる形の不等式って本当にそのまま論文で出版されてんのか?
他の議論をしているときに「たまたま現れました」っていう不等式や、
もっと洗練されたものの特殊ヴァージョンじゃないのか?
774132人目の素数さん:2009/03/09(月) 00:15:14
>>773だがさ、蛇足するが、
例えば、(「不等式への招待」)第5章「不等式の作成と証明法」
の例題1の不等式が本当にそのまま論文で出版されているのか、
ということを聞いた訳さ。
ちょっと読みにくかったり曖昧に書かれている箇所がところどころあるんだがな。
775132人目の素数さん:2009/03/09(月) 22:50:15
>>757 >>763
(2)
 |x| << 1 のとき、マクローリン展開から
 (1 + 1/x)log(1-x^2) = -x -   x^2 -O(x^3),
       log(1-x) = -x -(1/2)x^2 -O(x^3),
   (1/x)log(1-x^2) = -x      -O(x^3),
・・・ぢゃあダメだろうな。
776132人目の素数さん:2009/03/11(水) 00:00:41
xとyはともに正の実数でx+4y=3のとき、1/x+1/yの最小値は。

1文字消して微分したらできたけど全く愉快じゃないので、
かっこいい解法をお願いします。
777132人目の素数さん:2009/03/11(水) 00:10:00
(1^(1/2)+4^(1/2))^2<=(x+4y)(1/x+1/y).
778776:2009/03/11(水) 00:33:11
はやっ。
(√(ax),√(bx)) と (1/√x,1/√y) にCauchy-schwarzってことですね。
かっこいいす。ありがとう。
779132人目の素数さん:2009/03/11(水) 21:00:44
モノグラフって調和平均とか重みつきとかないよね
受験ではいらないってことか
他の不等式を学びたい場合はどんな本がいい?
780132人目の素数さん:2009/03/11(水) 23:00:18
>>779

>>2を読め!
781132人目の素数さん:2009/03/11(水) 23:28:00
>746のprime_132ってこのスレの住人?
782132人目の素数さん:2009/03/11(水) 23:34:10
あぁ、だがそれ以上は詮索しないでもらいたい
783132人目の素数さん:2009/03/13(金) 16:40:42
そろそろ>>749の大数の宿題は〆切?
784132人目の素数さん:2009/03/13(金) 20:10:07
785132人目の素数さん:2009/03/13(金) 20:46:55
F(a,b)=√(a^2+ab+b^2)
Fa=(2a+b).5/()^.5=0 a=-.5b
Fb=0 b=-.5a
F(a,b)=(a^2-.5a^2+.25a^2)^.5=(.75)^.5a
a=rcost,b=rsint a>b>0->cost>sint>0
F=r(1+costsint)^.5
786132人目の素数さん:2009/03/13(金) 23:45:21
>>783
3/15締め切りです。
787132人目の素数さん:2009/03/14(土) 23:52:56
ある直方体の12辺の長さの和を4L、表面積をS、体積をVとする
(1)L、Sが一定のときVのとりうる値を答えよ
(2)L、Vが一定のときSのとりうる値を答えよ
(3)S、Vが一定のときLのとりうる値を答えよ
788132人目の素数さん:2009/03/15(日) 00:22:26
>746 から一題・・・

〔問題〕
 0 < x のとき e^x > 3sin(x) を示せ。

http://www.casphy.com/bbs/test/read.cgi/highmath/1169210077/19
789132人目の素数さん:2009/03/15(日) 03:04:09
>>788
グラ…接…
790132人目の素数さん:2009/03/15(日) 20:34:14
>>749
後四時間で解禁
791132人目の素数さん:2009/03/15(日) 20:56:47
>>749 は問題としては簡単だが,何がしか背景があるのだろう
792132人目の素数さん:2009/03/15(日) 20:57:41
120°
793132人目の素数さん:2009/03/15(日) 23:03:12
>746 からもう一題・・・(外出だったらスマソ)

〔出題87〕
正数列 a[n] >0 の初項から第n項までの総和を S[n] とおく:
 S[n] = Σ[k=1,n] a[k].
このとき,
 {a[1]/S[n+1]}^(1/n) + 納m=1,n+1] {a[m]/(n・S[m])} ≦ 1 + (1/n),

http://www.casphy.com/bbs/test/read.cgi/highmath/1169210077/87, 121
794132人目の素数さん:2009/03/16(月) 00:04:47
〆切あげ
795132人目の素数さん:2009/03/16(月) 00:22:04
>746 から・・・

〔出題95〕
x, yを正の実数とし, x,yの調和平均, 相乗平均, 相加平均, 2乗平均をそれぞれH, G, A, Q とおく.
すなわち,
 H = 2xy/(x+y), G = √(xy), A = (x+y)/2, Q = √{(x^2+y^2)/2}
とおく.
(1) H ≦ G ≦ A ≦ Q を示せ.
(2) G-H ≦ Q-A ≦ A-G を示せ。

--------------------------------------------
H,G,A は等比数列だから
 (A+H)/2 ≧ √(AH) = G,
 G-H ≦ A-G,
また G^2, A^2, Q^2 は等差数列で、公差は
  = Q^2 - A^2 = A^2 - G^2 = (1/4)(x-y)^2 ≧ 0,
 (Q+A)(Q-A) = (A+G)(A-G)     ・・・・ (*)
よって
 G ≦ A ≦ Q,
 Q+A ≧ A+G,
これで (*) を割ると
 Q-A ≦ A-G,
あとは
 G-H ≦ Q-A,
を示せれば・・・
 G^2 - H^2 = (H/G)^2・ = (G/A)^2・ = (H/A)・,

http://www.casphy.com/bbs/test/read.cgi/highmath/1169210077/95, 100
796132人目の素数さん:2009/03/16(月) 01:18:31
今年の東北大入試問題から

a+b≧cであるとき
a^3+b^3+3abc≧c^3
797132人目の素数さん:2009/03/16(月) 01:53:36
それは易しすぎるだろ・・・
湘南工科大レベル
798132人目の素数さん:2009/03/16(月) 01:59:18
>>796
3乗の因数分解の公式そのもの。
レベル0
高校入学時の始業式の翌日にやる試験レベル
799132人目の素数さん:2009/03/16(月) 02:00:00
c=0。
d=0。

b=0。
d=0。
800132人目の素数さん:2009/03/16(月) 02:55:19
>>796
これはひどい
こんなのが出るのが今の入試は
801132人目の素数さん:2009/03/16(月) 02:57:01
もしかして文系学部の問題?
802132人目の素数さん:2009/03/16(月) 03:12:29
803132人目の素数さん:2009/03/16(月) 03:33:06
>>802
ガセネタかと思ったら、マジかよ!
オhル
804132人目の素数さん:2009/03/16(月) 04:01:21
おまえら不等式には厳しいなw
805132人目の素数さん:2009/03/16(月) 04:02:27
>>797 >>798 >>800 >>801 >>803
勘違いしてないか?そこまで簡単じゃないよ。
806132人目の素数さん:2009/03/16(月) 04:08:35
>>805
アー、アー、キコエマスカー?
どの辺が難しいのかな?

因数分解の公式かな?
それともその後の平方完成の仕方かな?
807132人目の素数さん:2009/03/16(月) 04:10:09
>>796
ウソだろ・・・高1の1学期の中間レベルだと・・・?
808132人目の素数さん:2009/03/16(月) 04:10:12
>>805
Fランのお前には難しいのだろう
国立行けるレベルの理系には簡単すぎるよ
809132人目の素数さん:2009/03/16(月) 04:14:48
講評では難易度は標準って書かれてるんだけどな
810132人目の素数さん:2009/03/16(月) 04:16:33
ゆとり的には標準なんじゃないか
811132人目の素数さん:2009/03/16(月) 04:20:19
>>809
そのとおり。
言うほど簡単じゃない。模範解答見ればやさしいけどね。
問い1としての難易度は適切だと思う。
良問かどうかはわかりかねるが、東北大の入試の倍率が3倍だとすると、
この問い1で受験生のうち下3分の1くらいは落とせるんじゃないかな。
812132人目の素数さん:2009/03/16(月) 04:25:08
>>811
どこが難しいのかkwsk
813132人目の素数さん:2009/03/16(月) 04:25:12
まず、下10%くらいは、
a+b>=c <=> a^3+b^3+3ab(a+b)>=c^3
として、a+bをcで置き換えて証明終わり
とする(東北大入試の)受験生はでてくる。
 そういった論理性の欠ける人を落とすのがひとつの目的だと推測
814132人目の素数さん:2009/03/16(月) 04:27:21
>>813
え?
815132人目の素数さん:2009/03/16(月) 04:30:04
(1)は=だから、>>813のとおりにやって良いんだけど、
(2)は不等号だから、だめ。
講評では(1)がヒントになる、と書いてあるけれど、かえって(1)の
おかげで間違える人も10%(いいすぎか5%)はいると思う。
816132人目の素数さん:2009/03/16(月) 04:32:04
だからいくらなんでも
>>798
>3乗の因数分解の公式そのもの。
>レベル0
>高校入学時の始業式の翌日にやる試験レベル
は言いすぎだろ。
817132人目の素数さん:2009/03/16(月) 04:34:27
>>816
え?高1はじめの中間に出るレベルだけど?
818132人目の素数さん:2009/03/16(月) 04:40:21
その入試問題1−6全部みるとわかるけど、問題1以外は証明問題は
問題5(1)だけで、あとは、値を求めさせる問題。
問題1は、証明がちゃんと書けるか、を見るのがねらいなんじゃないの?
問題1がなかったら、論証がいいかげんで答えを見つけるのが得意な
人だけが入ってきちゃう。
 それに、数学科志望生だけが受けるレベルじゃないからね。

>>817
でも、中間テストでみんなが満点なわけじゃないと思うよ。
しかも公式を習った直後で範囲が狭まっている時に出題すれば
得点率上がるのは当たり前。
君は高1か?予備校の模試受けたら(できれば東北大限定模試)、
各問題の平均点見てみ。
そんなに高くないよ。制限時間内ではね。
問題1としては適当。(6題全部がこれだったらやさしいけどね。)
819132人目の素数さん:2009/03/16(月) 04:42:16
>>817
君の学校は、始業式の翌日に中間テストをやるのか?
820132人目の素数さん:2009/03/16(月) 04:48:50
>>818
そうだね
ごめん
821132人目の素数さん:2009/03/16(月) 04:54:54
今年東北落ちた人か?
河合は標準、駿台はやや易だった
822132人目の素数さん:2009/03/16(月) 05:01:08
>>821
>今年東北落ちた人か?
落ちてない。落ちるほどの学力なら、あぁ、俺には難しいけど、
受かった人にとっては簡単なのかも、、と思って難易度は断定できない。

>河合は標準、駿台はやや易だった

そうだろ、標準か、やや易、だろ。そんなもんだ。
びっくりするほど易しすぎはしない。入試問題として。
823132人目の素数さん:2009/03/16(月) 07:56:31
>>819
>>817のどこに始業式の翌日にやるってかいてあるんだよ
824132人目の素数さん:2009/03/16(月) 07:59:47
>>811
真剣な話、どこが難しいのかな?

>>819
入学式の翌日に実力テストをやりましたが何か?
範囲は2次関数の終わりまで。
825132人目の素数さん:2009/03/16(月) 08:06:16
そろそろどこか他のところでやってくれ。
826132人目の素数さん:2009/03/16(月) 08:40:09
>>811は解けなかっただけだろ
827132人目の素数さん:2009/03/16(月) 09:17:11
件の人はa^3 + b^3 + c^3 - 3abcの因数分解の公式知らんのかね。

つうかこのスレは大学入試問題レベルの簡単な問題を
扱うような感じじゃないと思うけど。
828132人目の素数さん:2009/03/16(月) 09:21:55
少なくともこのスレ的には対称式に対する標準的な処方箋で解ける問題.
因数分解の公式なんて忘れても,基本対称式で書こうとするだけでいい.
829132人目の素数さん:2009/03/16(月) 19:32:17
>>749の正しい問題文は何だろう
830132人目の素数さん:2009/03/16(月) 22:31:37
>>795
 G-H ≦ Q-A を示そう。
 (A-H) = (A^2 -G^2)/A = (Q^2 -G^2)/(2A) = (Q-G){(Q+G)/(2A)},
 (Q-A) - (G-H) = (Q-G) - (A-H) = (Q-G){1-(Q+G)/(2A)} = {(Q-G)/(2A)}{(A-G)-(Q-A)} ・・・・・・ (**)
∴ Q-A は G-H と A-G の間にある(G-H寄り)。

**) 右辺の係数は 0 < (Q-G)/(2A) ≦ (Q+G)/(2A) < 1,

よって
(3) (Q-A)-(G-H) ≦ (A-G)-(Q-A),

〔問題〕
 3変数(x,y,z) のときは (1) のみが成り立つことを示せ。
831132人目の素数さん:2009/03/16(月) 23:39:12
東大入試数学過去問
http://hwm5.gyao.ne.jp/yonemura/t_archives.html

京大入試数学過去問
http://hwm5.gyao.ne.jp/yonemura/archives.html

大学入試数学過去問
http://www.densu.jp/

いくつかは不等式の問題拾えるんじゃない?
832132人目の素数さん:2009/03/16(月) 23:49:27
そんなもん見るよりジャーナル見るほうが有意義だ
833132人目の素数さん:2009/03/17(火) 00:00:13
>>830

(1)
 H = 3/(1/x + 1/y + 1/z), G = (xyz)^(1/3), A = (x+y+z)/3, Q = √{(x^2 + y^2 + z^2)/3},  
 Q^2 - A^2 = (x^2 +y^2 +z^2)/3 - (1/9)(x+y+z)^2 = (1/9){(x-y)^2 + (y-z)^2 + (z-x)^2} ≧ 0,
 A^3 - G^3 = {(x+y+z)/3}^3 -xyz = {(x+y+7z)/2}(x-y)^2 + {(7x+y+z)/2}(y-z)^2 + {(x+7y+z)/2}(z-x)^2 ≧ 0,
 (1/H)^3 - (1/G)^3 = {[(1/x)+(1/y)+(1/z)]/3}^3 - 1/(xyz) = {(x'+y'+z')/3}^3 - x'y'z' ≧ 0,
∴ H ≦ G ≦ A ≦ Q,

(2) y=z=1 の場合を考えると
 H = 3x/(1+2x), G = x^(1/3), A = (2+x)/3, Q = √{(2+x^2)/3},
 x<1 のとき G-H > A-G > Q-A,
 x>1 のとき G-H < A-G < Q-A,
834830:2009/03/17(火) 00:05:05
>>833
GJ!!

されど、3変数のときはQよりも
 T = {(x^3+y^3+z^3)/3}^(1/3),
使った方が良くね?
835132人目の素数さん:2009/03/17(火) 23:05:37
>>830, 833

(2) y=z=1 の場合は・・・・
 0 < x < 0.00415949095310635… のとき、      G-H < Q-A < A-G,
 0.00415949095310635… < x < 0.15064425… のとき, Q-A < G-H < A-G,
 0.15064425… < x < 1 のとき,           Q-A < A-G < G-H,
 1 < x < 9.33372455・・・ のとき、          G-H < Q-A < A-G,
 9.33372455・・・ < x のとき、            G-H < A-G < Q-A,
と成増とんねるず。

但し、区間の端点は
0.15064425142615432931841204604911・・・・ = 1/{2cos(π/9)}^3, x + 3・x^(2/3) -1 =0 の根。
9.3337245536744706772511885807731・・・・ = {(2√3)cos([π-arccos(25/27)]/6) - 1}^3, x + 3・x^(2/3) -6・x^(1/3) -10 =0 の根。
836132人目の素数さん:2009/03/17(火) 23:15:33
a≧b≧0,c≧d≧0のとき

√(a^2+ab+b^2)+√(c^2+cd+d^2)≧√(a^2+ac+c^2)+√(b^2+bd+d^2)
837132人目の素数さん:2009/03/18(水) 03:27:30
a,b,cを実数とする
a+b+c=0のとき
(|a|+|b|+|c|)^2≧2(a^2+b^2+c^2)
838132人目の素数さん:2009/03/18(水) 16:53:54
2(a^2+b^2+c^2) = (a^2+b^2+c^2)+(a+b+c)^2-2(ab+bc+ca)
=(a^2+b^2+c^2)-2(ab+bc+ca)
≦(a^2+b^2+c^2)+2(|ab|+|bc|+|ca|) = (|a|+|b|+|c|)^2
839835:2009/03/18(水) 22:01:22
>>830, >>833,

 0.00415949095310635… は x^3 +3x^(8/3) +3x^(7/3) +(9/2)x^2 +3x^(5/3) +3x^(4/3) +(39/8)x +(33/8)x^(2/3) + (3/4)x^(1/3) -(1/4) =0 の根。
840132人目の素数さん:2009/03/19(木) 16:04:37
>>836
チェビシェフの不等式
841132人目の素数さん:2009/03/19(木) 22:36:19
>>836
三角不等式
842132人目の素数さん:2009/03/19(木) 23:37:37
>>749>>799>>829>>836
a=1,b=1,c=0,d=0.
√(3)≧2.
843132人目の素数さん:2009/03/20(金) 00:22:31
a≧b≧0,c≧d≧0のとき

√(a^2+ad+d^2)+√(b^2+bc+c^2)≧√(a^2+ac+c^2)+√(b^2+bd+d^2)

じゃね?
844132人目の素数さん:2009/03/20(金) 01:39:13
>>843
bingo!
845132人目の素数さん:2009/03/22(日) 07:14:22
>>843

∠UOV = 120゚ なる半直線 OU,OV を考える。 >>792
OU上に OA=a, OB=b なる点A,B をとる。
OV上に OC=c, OD=d なる点C,D をとる。

題意により、2つの線分AD, BC は交わるから、交点を X とする。2つの線分AC, BD は交わらない。
 (左辺) = AD + BC = (AX + XD) + (BX +XC) = (AX + XC) + (BX + XD) ≧ AC + BD = (右辺).

[初代スレ.068, 071] の類題ぢゃね?
846132人目の素数さん:2009/03/25(水) 01:42:40
847132人目の素数さん:2009/03/28(土) 00:40:44
>>846

Problem 1.
a,b,c は正の実数で、a+b+c=1 を満たすとき
 a^(1-a) * b^(1-b) * c^(1-c) ≦ 1/9,

 Austrian M.O. 2008, Final round (part 2)


Problem 312.
 a,b,c を正の実数とするき
(a+1)^4 /(b^2) + (b+1)^4 /(c^2) + (c+1)^4 /(a^2) ≧ 48,

Problem 316.
 n>6 のとき, 凸n角形A0A1……An に対して適当な i≠j が存在して
  |cos(∠Ai) −cos(∠Aj)| < 1/{2(n-6)},
848132人目の素数さん:2009/03/28(土) 00:59:42
>>847

Solution 1.
 f(x) = (1-x)log(x), とおくと
 f "(x) = -(1+x)/(x^2) < 0,
∴ y=f(x) は上に凸。
∴ log(左辺) = f(a) + f(b) + f(c) ≦ 3f((a+b+c)/3) = 3f(1/3) = 2log(1/3) = log(1/9) = log(右辺),

Solution 312.
相加相乗平均2回
 (a+1)^2 = (a-1)^2 + 4a ≧ 4a, etc.
 (左辺) ≧ 16{(a/b)^2 + (b/c)^2 + (c/a)^2}
  = 16{(aac)^2 + (bba)^2 + (ccb)^2}/(abc)^2
  = 48 + {(aac)^2 + (bba)^2 + (ccb)^2 -3(abc)^2}/(abc)^2
  = 48 + (X^3 + Y^3 + Z^3 -3XYZ)/(abc)^2
  ≧ 48 = (右辺).
ここに X=(AAC)^(2/3), Y=(BBA)^(2/3), Z=(CCB)^(2/3),
 X^3 +Y^3 +Z^3 -3XYZ = (1/2)(X+Y+Z){(X-Y)^2 +(Y-Z)^2 +(Z-X)^2} ≧ 0,
849132人目の素数さん:2009/03/29(日) 01:26:28
>>847

Solution 316.
外角 π-A_i の和は2πである:
 (π-A_1) + (π-A_2) + …… + (π-A_n) = 2π,
n>k とする。
π-A_i > 2π/k となる A_i は k-1 個以下。
残りの n-k+1 個以上については 0 <π-A_i ≦ 2π/k,
 -1 < cos(A_i) ≦ - cos(2π/k),
ディリクレの引き出し論法(鳩ノ巣原理)により、
 | cos(A_i) - cos(A_j) | < {1 - cos(2π/k)}/(n-k),
本問では k=6.

〔蛇足〕
nを固定すると、k ≒ 2n/3 の辺りで最小になると・・・
 (右辺) < (2π^2)/{(n-k)k^2} < (27/2)π^2 / n^3,
850849:2009/04/05(日) 19:45:07
↑は
http://www.math.ust.hk/excalibur/v14_n1.pdf
のp.3に出てた。orz
しかたないので一題・・・

Problem 2.
Let a_1 〜 a_5 be real numbers satisfying the following equations:
 a_1/(1+k^2) + a_2/(2+k^2) + a_3/(3+k^2) + a_4/(4+k^2) + a_5/(5+k^2),
for k=1〜5. Find the value of
 a_1/37 + a_2/38 + a_3/39 + a_4/40 + a_5/41,
 (Express the value in a single fraction.)
851132人目の素数さん:2009/04/05(日) 19:51:15
>>850
結果だけ並べると・・・

 a_1 = 1105/72,
 a_2 = -2673/40,
 a_3 = 1862/15,
 a_4 = -1885/18,
 a_5 = 1323/40,
より
 b_6 = 187465/(3*37*38*39*41)    ≒ 1.00061649483987・・・ / 36,
 b_7 =  1197/(5*13*17*53)      ≒ 1.00150260394436・・・ / 49,
 b_8 =  85345/(16*13*17*23*67)    ≒ 1.00240485551780・・・ / 64,
 b_9 = 277289/(9*17*41*43*83)    ≒ 1.00321917612728・・・ / 81,
 b_10=12117378/(3*25*7*13*17*101*103) ≒ 1.00391855290609・・・ / 100,
 b_0 = 13489 / 3600 ≒ 3.74694444444444・・・
ここに b_k = a_1/(1+k^2) + a_2/(2+k^2) + a_3/(3+k^2) + a_4/84+k^2) + a_5/(5+k^2),
852132人目の素数さん:2009/04/05(日) 23:12:36
不等式バンジャイ!
853850:2009/04/07(火) 21:04:51
スレ違いだったか・・・・・  ---> 線形代数/線型代数スレ
ぢゃあ もう一題

〔問題322'〕
Let a,b,c be positive real numbers satisfying the condition a+b+c=s. 
Prove that
 (a^2)(3b+s)/(as+bs+3ab) + (b^2)(3c+s)/(bs+cs+3bc) + (c^2)(3a+s)/(cs+as+3ca) ≧ 2,

854132人目の素数さん:2009/04/07(火) 23:31:17
>>853
忙しいので、とりあえずハァハァしておく!
(;´ρ`) ハァハァ…
855 ◆BhMath2chk :2009/04/08(水) 00:00:00
 a(1)/(x+1)+a(2)/(x+2)+a(3)/(x+3)+a(4)/(x+4)+a(5)/(x+5)−1/x
=(x−1)(x−4)(x−9)(x−16)(x−25)/120x(x+1)(x+2)(x+3)(x+4)(x+5)。
856132人目の素数さん:2009/04/08(水) 00:09:21
>>855
857132人目の素数さん:2009/04/08(水) 01:16:17
>>853
a=b=c=1/2とかで不等式が成立しない気が
858132人目の素数さん:2009/04/08(水) 23:41:08
>>857
 スマン。↓に訂正。

 (a^2)(3b+s)/(as+bs+3ab) + (b^2)(3c+s)/(bs+cs+3bc) + (c^2)(3a+s)/(cs+as+3ca) ≧ (2/3)s,
859132人目の素数さん:2009/04/09(木) 19:57:31
>>686 2)

 (pa-qb)/(a-b) =X, (pb-qc)/(b-c) =Y, (pc-qa)/(c-a) =Z,
とおくと、
 X^2 + Y^2 + Z^2 = p^2 + q^2 + (X+Y+Z-p-q)^2 ≧ p^2 + q^2,

本問では p=4, q=3,
860132人目の素数さん:2009/04/09(木) 21:05:57
>>858
(左辺)={a-abs/(as+bs+3ab)}+{b-bcs/(bs+cs+3bc)}+{c-cas/(cs+as+3ca)}
    ≧{a-(b+a+s/3)/9}+{b-(c+b+s/3)/9}+{c-(a+c+s/3)/9}  (-調和≧-相加)
    =(2/3)s
861132人目の素数さん:2009/04/10(金) 16:55:30
>>686 1)
a≧b≧c,x≧y≧z,X≧Y≧Zのとき
a/(y+Z)+b/(z+X)+c/(x+Y)≧a/(x+X)+b/(y+Y)+c/(z+Z)
が成り立てば示せるが…
成り立つ?
862132人目の素数さん:2009/04/11(土) 16:35:09
>>686 2), >>859 の略証・・・

 X = {p+q+(p-q)x}/2, Y = {p+q+(p-q)y}/2, Z = {p+q+(p-q)z}/2,
とおくと
 X^2 + Y^2 + Z^2 = (X+Y+Z)^2 -2(XY+YZ+ZX)
  = (X+Y+Z-p-q)^2 -(p+q)^2 +2(p+q)(X+Y+Z) -2(XY+YZ+ZX)
  = (X+Y+Z-p-q)^2 -(p+q)^2 +3(p+q)^2 -(3/2)(p+q)^2 -(1/2)(p-q)^2・(xy+yz+zx)
  = p^2 + q^2 + (X+Y+Z-p-q)^2 -(1/2)(p-q)^2・(xy+yz+zx+1),
ところで、題意から
 x = (a+b)(a-b), y = (b+c)/(b-c), z = (c+a)/(c-a),
∴ xy + yz + zx + 1 = 0,
863132人目の素数さん:2009/04/16(木) 01:49:31
問題投下
3辺がa,b,cの三角形の面積と3辺が1/a,1/b,1/cの三角形の面積の積が3/16を超えないことを示せ

ヘロンでどぞー
864132人目の素数さん:2009/04/16(木) 02:05:16
キタコレ!
865132人目の素数さん:2009/04/18(土) 10:54:55
>>863
  = (1/2)ab・sin(C) = (1/2)bc・sin(A) = (1/2)ca・sin(B),
でも解けるお。
  = (1/2)(abc)^(2/3)・{sin(A)sin(B)sin(C)}^(1/3)
   ≦ (1/2)(abc)^(2/3)・{sin(A)+sin(B)+sin(C)}/3   (相加・相乗平均)
   ≦ (1/2)(abc)^(2/3)・sin((A+B+C)/3)       (0〜πで上に凸)
   = (1/2)(abc)^(2/3)・sin(π/3)
   = (√3)/4・(abc)^(2/3),     (等号成立はA=B=C(正三角形)のとき.)
  '≦ (√3)/4・(1/abc)^(2/3),
辺々掛ける。
866132人目の素数さん:2009/04/18(土) 11:52:00
>>863
せっかくのヒントなのでヘロンで・・・・・

本問では、 (a+b+c)/2 =s とおく。
 (s-a) + (s-b) + (s-c) = s,
  = √{s(s-a)(s-b)(s-c)}
  ≦ √{s(s/3)^3}       (相加・相乗平均)
  = (1/√27)s^2
  = (√3)/4・{(a+b+c)/3}^2
  ≦ (√3)/4・(abc)^(2/3),   (相加・相乗平均)
  '≦ (√3)/4・(1/abc)^(2/3),
辺々掛ける。
867132人目の素数さん:2009/04/18(土) 11:54:39
>>866 はまちがい。
 無視してください。
868132人目の素数さん:2009/04/18(土) 11:55:55
無視しません!
869132人目の素数さん:2009/04/18(土) 13:48:39
黙殺する
870866:2009/04/18(土) 23:39:33
>>863
せっかくのヒントなのでヘロンで・・・・・

本問では、 (a+b+c)/2 =s とおく。また
 (s-a) + (s-b) + (s-c) = s,
 (s-a)(s-b) + (s-b)(s-c) + (s-c)(s-a) = t,
 (s-a)(s-b)(s-c) = u,
とおく。
  = √(su)
  = {s・(√su)・u}^(1/3)
  ≦ {(1/√3)st・u}^(1/3)       (3su≦t^2)
  ≦ (√3)/4・(st-u)^(2/3)      (*)
  = (√3)/4・(abc)^(2/3),

※ st-u ≧ (8/9)st ≧ 8u より
 st≦(9/8)(st-u), u≦(1/8)(st-u),
871132人目の素数さん:2009/04/22(水) 19:51:53
n:自然数とする。

(1) 2数 x、y の和、積を考え
 x+y=p、xy=q この p、q が共に整数ならば
 x^n + y^n は整数であることを証明せよ。

(2) x>0、y>0 のとき
 ( (x+y)/2 )^n ≦ (x^n + y^n )/2 であることを証明せよ。 
872132人目の素数さん:2009/04/22(水) 20:57:26
>>871
馬鹿か?
873132人目の素数さん:2009/04/22(水) 22:15:48
>>861

〔命題〕
a,b,c >0, x,y,z >0 のとき
 f(x,y,z) = a/(b+cx) + b/(c+ay) + c/(a+bz) - 9/(x+y+z+3) ≧0,
が成り立てば示せるが…
成り立つ?


 f(0,0,0) = (a/b) + (b/c) + (c/a) -3 ≧0,
 x,y,z のいずれかが∞となるとき、(右辺) → 0 で成立。
f(,,)に極値があるとすれば
 ∂f/∂x = -ca/(b+cx)^2 + 9/(x+y+z+3)^2 =0,
 ∂f/∂y = -ab/(c+ay)^2 + 9/(x+y+z+3)^2 =0,
 ∂f/∂z = -bc/(a+bz)^2 + 9/(x+y+z+3)^2 =0,
b+cx >0, c+ay >0, a+bz >0, x+y+z+3 >0 から
 a/(b+cx) = 3{√(a/c)}/(x+y+z+3),
 b/(c+ay) = 3{√(b/a)}/(x+y+z+3),
 c/(a+bz) = 3{√(c/b)}/(x+y+z+3),
に限る。この点でf(,,)が極大なら         (←これが問題だが・・・・)
 f(x,y,z) ≧ 3{√(a/c) + √(b/a) + √(c/b) -3}/(x+y+z+3) ≧0,
874132人目の素数さん:2009/04/22(水) 23:47:37
>>872
馬鹿か?
875132人目の素数さん:2009/04/23(木) 08:55:27
( ゚∀゚)<荒らしイクナイ!
876132人目の素数さん:2009/04/23(木) 09:10:06
こんなスレがあったとは!!最近不等式に興味持ち出していろいろやってます。

ところSOS不等式って何ですか?
877132人目の素数さん:2009/04/23(木) 10:24:04
>>876
sum of squares inequality: Σa_k^2 ≧ 0

「任意の非負有理式は有理式の自乗和で書ける」というArtinの結果があるので,
有理式≧0 は,左辺を記述する有理式の自乗和を構成する方法で必ず証明できる.
(もちろん,それを見つけるのが簡単とか難しいとかはあるけれど)
878132人目の素数さん:2009/04/23(木) 13:53:03
マジ?
SOS不等式ってもは初めて聞いたし、877の解説が面白くて、ネタに見えて仕方がないんだけど…
879132人目の素数さん:2009/04/23(木) 15:32:04
>>878
SOS不等式と呼ばれている不等式が何種類かあって紛らわしいから,
普通は前後の脈絡無しに「SOS不等式」って言うことはないんだけど,
最近最適化の専門家の間で >>877 の「SOS不等式」が盛んに研究されてて,
個人的にも今その辺がホットだから,勢いで書いちゃった.

数学的な内容としては >>877 の結果は正しくて,
特にここ10年くらいで,この方法を使った多項式の最小値を計算する
アルゴリズムが実用的になってきてる.

興味があるなら,古典的な話題はHilbertの第17問題で調べるとよくて,
最近は Pablo A. Parrilo って人が活発にやってる.
880132人目の素数さん:2009/04/24(金) 00:09:49
>>879
勉強になりますた!
881132人目の素数さん:2009/05/07(木) 21:53:59
〔問題857〕
xが自然数 のとき
 3^(x-1) ≧ LCM(1,2,3,・・・・,x) ≧ 2^(x-1),

http://science6.2ch.net/test/read.cgi/math/1220115988/857
東大入試作問者スレ16
882132人目の素数さん:2009/05/08(金) 03:29:09
質問です。
一応、有名不等式(Weighted AM-GM,Cauchy-schwartz,Holder,Rearrangement,
Jensen,Muirhead,Schur,etc...)などについての知識やその証明は理解したのですが
実際に問題に取り組む時に「どんな場合にどの有名不等式を用いるべきか」が見えてきません。
不等式が得意な方々の解法などを眺めていても妙に突拍子なアイディアにしか見えないんです。

数多くの問題に当たってるうちにこういう直観的なものは磨かれどれをいつ適用するかなど
見えてくるものなんでしょうか?
883132人目の素数さん:2009/05/08(金) 03:44:16
>>882
そうです!
甘ったれないで下さい!
884132人目の素数さん:2009/05/09(土) 19:20:05
>>882
職人芸修行  文献を大量に勉強してどの方法はどこで使うか博覧強記
イメージ戦略 解きたい問題が解けると信じる直観的理由と同じ意味の不等式を利用
試行錯誤   各場面毎に片端から使って出てくる不等式をノートに蓄積
他にもあるが時間が無くなったので
885132人目の素数さん:2009/05/09(土) 23:26:18
>>882
とりあえず、片っ端から不等式とその証明(別解も全て)をコレクションし、Texでまとめるんだ!
886132人目の素数さん:2009/05/09(土) 23:31:21
>>881
休憩が終わったら、他スレの不等式をここに貼る作業に戻るんだ!

>>882
休憩が終わったら、刺身の上にタンポポを乗せる仕事に戻るんだ!

>>884
休憩が終わったら、不等式を証明する作業に戻るんだ!

>>885
休憩が終わったら、不等式まとめサイトを更新する作業に戻るんだ!

>>886
休憩が終わったら、不等式を収集する作業に戻るんだ!
887132人目の素数さん:2009/05/10(日) 03:49:35
>>876
SOS を具体的に用いて解いた解法は >>699 にありますよ。

簡単に言ってしまえば SOS ineq は
それぞれ S[a], S[b], S[c] は a, b, c の関数とし,
S = f(a, b, c) = S[a](b-c)^2 + S[b](c-a)^2 + S[c](a-b)^2
とおいたとき, S≧0 を証明するために使われる手法です。


>>887, >>889 で言われているものも SOS ineq ですが, >>876 さんが知りたいのはこういう類の方の ineq ですよね?
888132人目の素数さん:2009/05/10(日) 21:46:04
〔問題〕
 a,b,c,p,q >0, 1/2 ≦ p/q ≦ 2 のとき
 a/(pb+qc) + b/(pc+qa) + c/(pa+qb) ≧ (p+q)^2 (a+b+c)^3 /{9(pb+qc)(pc+qa)(pa+qb)}
   ≧ (1/3)(a+b+c){1/(pb+qc) + 1/(pc+qa) + 1/(pa+qb)}
   ≧ 3/(p+q),
   (Shapiro不等式の一拡張)
889132人目の素数さん:2009/05/10(日) 21:59:17
>>888
見かけほど難しくない(?)

左側:
 a/(pb+qc) + b/(pc+qa) + c/(pa+qb) - (p+q)^2 (a+b+c)^3 /{9(pb+qc)(pc+qa)(pa+qb)}
 = {(2p-q)(2q-p)F_1 + (p+q)(2p-q)G + (p+q)(2q-p)H}/{9(pb+qc)(pc+qa)(pa+qb)} ≧ 0,
ここに
 F_1 = a(a-b)(a-c) + b(b-c)(b-a) + c(c-a)(c-b) = s^3 -4st +9u ≧ 0,
 G = F_1 + (st-9u+3)/2 = a(a-b)^2 + b(b-c)^2 + c(c-a)^2 ≧ 0,
 H = F_1 + (st-9u-3)/2 = a(a-c)^2 + b(b-a)^2 + c(c-b)^2 ≧ 0,
ここに
 s = a+b+c, t = ab+bc+ca, u = abc, 基本対称式
  = (a-b)(b-c)(c-a), 差積

中央と右側:
 pb+qc = x, pc+qa = y, pa+qb = z, とおく。
 a+b+c = (x+y+z)/(p+q),
よって 相加・調和平均より
 (x+y+z)^3 /(9xyz) = (x+y+z){F_0 + 3(xy+yz+zx)}/(9xyz) ≧ (1/3)(x+y+z)(1/x + 1/y + 1/z) ≧ 3,
これを (p+q) で割る。ここに
 F_0 = (x-y)(x-z) + (y-z)(y-x) + (z-x)(z-y)
  = (x^2 + y^2 + z^2) - (xy+yz+zx)
  = (1/2){(x-y)^2 + (y-z)^2 + (z-x)^2} ≧ 0,
890132人目の素数さん:2009/05/11(月) 20:17:25
nを正の整数とする。
(n+2)角形A1A2……AnA(n+1)A(n+2)について、面積をS
正整数kに対して辺AkA(k+1)の長さをx(k)とする。
このとき
納k=1_n] {(k^2-k+1)*(x(k))^2} + n*x(n+1)^2 ≧4S
を示せ。
891132人目の素数さん:2009/05/11(月) 20:28:43
〔問題895〕
正の実数a,b,cに対して不等式

 a/{(s/3)+2b} + b/{(s/3)+2c} + c/{(s/3)+2a} ≧ 1,

が成立することを示せ。 ただし、s = a+b+c.

http://science6.2ch.net/test/read.cgi/math/1220115988/895
東大入試作問者スレ16

 a/{(s/3) +2b} = a/{s +2(b -s/3)} ≧ a{s -2(b -s/3)}/(s^2) = a(5s-6b)/(3s^2),
巡回的にたす。
 (左辺) ≧ {5s^2 -6(ab+bc+ca)}/(3s^2) = (3s^2 +2F_0)/(3s^2) ≧ 1,
ここに
 F_0 = s^2 - 3(ab+bc+ca) = (a-b)(a-c) + (b-c)(b-a) + (c-a)(c-b) = (1/2){(a-b)^2 + (b-c)^2 + (c-a)^2} ≧ 0,
892132人目の素数さん:2009/05/11(月) 22:56:21
893132人目の素数さん:2009/05/12(火) 02:26:44
>>891

 ab+bc+ca ≦ (1/3)s^2,
 f(x) = 1/{(s/3)+2x} は単調減少かつ下に凸。
 (左辺) = a・f(b) + b・f(c) + c・f(a) ≧ s・f((ab+bc+ca)/s) ≧ s・f(s/3) = 1,
894132人目の素数さん:2009/05/12(火) 02:34:51
パネェっす
895132人目の素数さん:2009/05/13(水) 03:26:07
nを正の整数とする。
(n+2)角形 A1A2……AnA(n+1)A(n+2) について、面積をS,
正整数kに対して、辺AkA(k+1) の長さをx(k)とする。(1≦k≦n+1)
このとき
 (1/2)納1≦i<j≦n+1] x_i・x_j ≧ S,
を示せ。
896132人目の素数さん:2009/05/13(水) 05:00:00
二年。
897132人目の素数さん:2009/05/13(水) 23:27:15
>>895

180度より大きい内角が存在するような図形も考慮するの?
898132人目の素数さん:2009/05/16(土) 15:02:41
>>892
Problem 322.
 a+b+c=3 のとき、
 a^2・(b+1)/(a+b+ab) + b^2・(c+1)/(b+c+bc) + c^2・(a+1)/(c+a+ca) ≧ 2,

(略証)
 a+b+c=s とする。
 D = (a+b+ab)/(b+1) + (b+c+bc)/(c+1) + (c+a+ca)/(a+1)
  = (a+1) -1/(b+1) + (b+1) -1/(c+1) + (c+1) -1/(a+1)
  = s + 3 -1/(a+1) -1/(b+1) -1/(c+1),
ところで、
 1/(x+1) = 1/{2 + (x-1)} ≧ (1/4){2 - (x-1)} = (3-x)/4,
より
 1/(a+1) + 1/(b+1) + 1/(c+1) ≧ (9-s)/4,
あるいは、y=1/(x+1) は下に凸だから、
 1/(a+1) + 1/(b+1) + 1/(c+1) ≧ 3/(s/3 +1) = 9/(s+3),
よって
 D ≦ s+3 - (9-s)/4 = (5s+3)/4 = 9/2, あるいは
 D ≦ s+3 - 9/(s+3) = 9/2,
コーシー不等式より
 (左辺) ≧ s^2 /D ≧ s^2 /{(5s+3)/4} = 2, あるいは
 (左辺) ≧ s^2 /D ≧ s^2 /{(s+3) -9/(s+3)} = 2,
899132人目の素数さん:2009/05/16(土) 15:58:00
>>892
Problem 322.
 a,b,c>0, a+b+c=3 のとき、
 a^2・(b+1)/(a+b+ab) + b^2・(c+1)/(b+c+bc) + c^2・(a+1)/(c+a+ca) ≧ 2,

(略証)
 xy ≦ (1/4)(x+y)^2, より
 (左辺) = (a-1) + (a+b)/(a+b+ab) + (b-1) + (b+c)/(b+c+bc) + (c-1) + (c+a)/(c+a+ca)
   = s-3 +(a+b)/(a+b+ab) + (b+c)/(b+c+bc) + (c+a)/(c+a+ca)
  ≧ s-3 + 1/{1 + (a+b)/4} + 1/{1 + (b+c)/4} + 1/{1 + (c+a)/4}
  ≧ s-3 + 9/(3 + s/2)            (← 相加・調和平均)
  = 2,
900132人目の素数さん:2009/05/16(土) 16:43:25
>>895
凸でない場合は、凸でない部分を折り返すことで
辺の長さの構成を変えずにより面積を大きくできるので
凸の場合を考えればよい。
(n+2)角形を、点A1を端点の一つとする対角線で分割し
それぞれで三角不等式を用いて上からおさえれば示せる。
901132人目の素数さん:2009/05/16(土) 18:33:49
>>900
 正解でつ!!

三角形 A1AjA(j+1) の面積は
 (1/2)A1Aj・x_j・sin(∠A1AjA(j+1)) ≦ (1/2)A1Aj・x_j ≦ (1/2){x_1 + x_2 + ・・・・・ + x_(j-1)}x_j
これを j=2 から j=n+1 までたす。
 x_(n+2) を含まないところがミソ。この辺が重なるように2つ並べると・・・

〔系〕
点対称または線対称な2n+2角形の 面積を S, 周長を
 L = 2(x_1 + x_2 + ・・・・・ + x_n + x_(n+1)),
とすると、
 {n/(8(n+1))}L^2 ≧ 納1≦i<j≦n+1] x_i・x_j ≧ S,

※ 等周問題からは {1/(4π)}L^2 ≧ S, (等号成立は円のとき)
902132人目の素数さん:2009/05/26(火) 03:04:31
a,b,cは正の実数でa+b+c=1を満たす。nを正の整数とするとき
Π(k=0_n) 1/{1+a^(2^k)}{1+b^(2^k)}{1+c^(2^k)} > 8abc
を示せ。
903132人目の素数さん:2009/05/26(火) 09:33:40
>>902
難解すぐる…
904132人目の素数さん:2009/05/26(火) 21:46:19
>>902
左辺に
 1 + a^(2^k) = {1 - a^(2^(k+1))}/{1 - a^(2^k)}, 等
を代入して
 Π(k=0,n) 1/{1 + a^(2^k)} = (1-a)/{1 - a^(2^(n+1))} > 1-a, 等(0<a,b,c<1)
ここで a+b+c = s とおくと、
 (左辺) - (右辺) > (s-a)(s-b)(s-c) - 8abc
  = s(ab+bc+ca) - 9abc
  = a(b-c)^2 + b(c-a)^2 + c(a-b)^2 ≧ 0,

ハァハァ
905132人目の素数さん:2009/05/27(水) 01:16:53
Σ(゚Д゚ )!

ふ、ふつくしい…
906132人目の素数さん:2009/05/27(水) 02:20:23
どの角も鈍角でない三角形ABCの三辺の長さをa,b,cとする。このとき
(1/a^2+1/b^2+1/c^2)(a^2+b^2)≧5
を示せ。
907132人目の素数さん:2009/05/27(水) 02:39:42
>>906
a,bを固定すると、c^2が最大のとき左辺は最小。
鈍角が無いからc^2=a^2+b^2が最大値である。

一般性を失うことなくa^2+b^2=1とすると、
左辺=1/a^2+1/b^2+1 なので、
a^2=b^2=1/2のときに最小値を取る。
908132人目の素数さん:2009/05/27(水) 07:01:24
>>906
左辺に c^2 がないから、タイプミスかと思っていたぜ…
909132人目の素数さん:2009/05/28(木) 01:42:00
>>2の本でお薦めはありますか?
910132人目の素数さん:2009/05/28(木) 05:48:50
>>909
全てだ!
911132人目の素数さん:2009/05/28(木) 15:20:08
実数x,y,zが、xyz=1,0<x<y≦1を満たすとき
z/(y-x)≧4
を示せ。
912132人目の素数さん:2009/05/28(木) 15:29:02
xyz=1 なので z=1/(xy). これをz/(y-x)≧4 に代入して整理すると、
xy(y-x)≦1/4 を示せばよいことがわかる。
相加相乗平均の関係式より x(y-x)≦y^2/4 なので、
xy(y-x)≦y^3/4≦1/4.
913132人目の素数さん:2009/05/28(木) 19:58:50
〔Stirlingの不等式〕
nが自然数のとき、
 √(2π)・n^(n +1/2)・e^(-n) < n! ≦ e・n^(n +1/2)・e^(-n),
を示してくださいです。
できれば代数的に・・・

http://science6.2ch.net/test/read.cgi/math/1242389481/50
東大入試作問者スレ17
914132人目の素数さん:2009/05/28(木) 22:08:23
>>913
代数的とは?
915132人目の素数さん:2009/05/28(木) 22:50:08
>>914
 ビブンのことはしない、ってことぢゃね?
916132人目の素数さん:2009/05/28(木) 23:07:10
解析使わないってことでしょ
917132人目の素数さん:2009/05/28(木) 23:42:26
積分による不等式評価もだめかしら。
expをどうやって定義しようか。
918132人目の素数さん:2009/05/29(金) 05:18:05
>>913
オイラーの無限解析に書いてあるよとか確認せずに言って見るテスト
919132人目の素数さん:2009/05/29(金) 06:10:07
>>918
責任もって確認してくるように!
920132人目の素数さん:2009/05/30(土) 23:38:14

ノート派ですか?ルーズリーフですか?

□□□示すべき不等式□□□

(証明)
・・・・・・・・・
・・・・・・・・・
・・・・・・・・・

(証明2)
・・・・・・・・・
・・・・・・・・・
・・・・・・・・・

こんな感じで書いてるんですが、皆さんはどうですか?
921132人目の素数さん:2009/05/30(土) 23:56:10
>>920
TeXで書いて、別の証明があったら付け加えて…、の繰り返しだわな… ( ゚∀゚)テヘッ!
922132人目の素数さん:2009/05/31(日) 00:48:23
0<a,b,cかつa+b+c=6のとき
(a^a)(b^b)(c^c)≧(abc)^2
923132人目の素数さん:2009/05/31(日) 08:50:57
たいしょうせいよりうんたらかんたら
たいすうとってちぇびしぇふ
924132人目の素数さん:2009/05/31(日) 10:07:41
>>923
なるほど,上手いね!
925132人目の素数さん:2009/05/31(日) 23:08:05
>>906
は C≦90゚ ならおk.

蛇足だが、C>90゚ も許すと、
 (1/a^2 + 1/b^2 + 1/c^2)(a^2 + b^2) > 9/2,

(略証)
コーシーより
 (1/a^2 + 1/b^2)(a^2 + b^2) = 4 + (a/b - b/a)^2 ≧ 4,
c<a+b より
 c^2 < (a+b)^2 = 2(a^2 + b^2) - (a-b)^2 ≦ 2(a^2 + b^2),
 (1/c^2)(a^2 + b^2) > 1/2,
926132人目の素数さん:2009/06/01(月) 03:22:29
>>920
ルーズリーフに大きく不等式を書いてセクションみたくして
次ページから証明を書けばよい
そうすれば後から追加し放題じゃね?
927132人目の素数さん:2009/06/01(月) 14:42:43
>>926
なるほどなー
TeXが使えなかったら、きっとそうしていたね!
928926:2009/06/02(火) 10:42:43
ページの片側だけ使うようにすれば,さらに視認性が上がる(ひっくり返さなくて済む)
ただしページが倍になる...
929132人目の素数さん:2009/06/02(火) 22:12:20
>>925

C≧90゚ のときは
 cos(C) ≦ 0,
 c^2 = a^2 + b^2 -2ab・cos(C) ≦ {1-cos(C)}(a^2 +b^2),  第二余弦定理
 (1/c^2)(a^2 + b^2) ≧ 1/{1-cos(C)},
 (906の左辺) ≧ 4 + 1/{1-cos(C)},
930132人目の素数さん:2009/06/02(火) 22:15:36
>>926
???
931926:2009/06/02(火) 22:35:55
>>930
>>926>>928方式だと
ルーズリーフの
1枚目表: 不等式(1)
2枚目表: 証明(1)
・・・3枚証明に割く・・・
5枚目表: 証明(2)
・・・2枚証明に割く・・・

7枚目表: 不等式(2)
8枚目表: 証明(1)
・・・以下続く・・・

不等式(1)の証明が増えたら,7枚目の前に入れていく.
セクションとは区切りね.不等式ごとに区切る.物理的という意味ではないが,目印として付箋貼っとくとかインデックスシール貼るとか,そのまんまルーズリーフ区切り入れるとか
932132人目の素数さん:2009/06/03(水) 02:06:48
>>931
どのくらいのレベルの不等式から取捨選択するかによって大きく異なりそう

長方形ABCDの辺AD,CD(頂点は除く)上にそれぞれ点P,Qをとる
PB+PQ<AB+AQ

a,b,cは自然数とする
2^(a+b)+2^(b+c)+2^(c+a)≧2^(a+b+c+1)+1

933132人目の素数さん:2009/06/04(木) 01:12:43
 2^(a+b) + 2^(b+c) + 2^(c+a) ≦ 2^(a+b+c) + 4,

(略証)
A,B,C≧0 のとき
 (2+A)(2+B)(2+C) +4 -(2+A)(2+B) -(2+B)(2+C) - (2+C)(2+A) = AB+BC+CA + ABC ≧ 0,
934132人目の素数さん:2009/06/05(金) 23:06:39
         ____
       /      \  宿題が終わらないお
     /  _ノ  ヽ、_  \
    / o゚((●)) ((●))゚o \
    |     (__人__)    |
    \     ` ⌒´     /
   /´           `\
  /  /          l  l   .___
__l  l_¶______/_/__/     ヽ
  \, ´-'ヽ  ̄| ̄ ̄ ̄ ̄|   l二二二二l
    ヾ_ノ   | '''' '   |   l二二二二l
   | 9=ε-8. | '''..--  |   l二二二二l:::..
   |   ..''  |  ''-.  ,|
935132人目の素数さん:2009/06/06(土) 00:57:38
2^x ≧ 2φ(x) のとき
 2^(a+b)φ(c) + 2^(b+c)φ(a) + 2^(c+a)φ(b) ≦ 2^(a+b+c) + 4φ(a)φ(b)φ(c),

(略証)
 2^a - 2φ(a) = A, 2^b -2φ(b) = B, 2^c -2φ(c) = Cとおくと、
題意により A,B,C≧0,
 2^(a+b+c) + 4φ(a)φ(b)φ(c) - 2^(a+b)φ(c) - 2^(b+c)φ(a) - 2^(c+a)φ(b)
 = {2φ(a)+A}{2φ(b)+B}{2φ(c)+C} + 4φ(a)φ(b)φ(c) - φ(c){2φ(a)+A}{2φ(b)+B} - φ(a){2φ(b)+B}{2φ(c)+C} - φ(b){2φ(c)+C}{2φ(a)+A}
 = ABφ(c) + BCφ(a) + CAφ(b) +ABC ≧ 0,

(例)
 φ(x) = 1, x, (x^2 -x+2)/2, ・・・・・, Σ[k=0,n] C[x-1,k], ・・・・
936132人目の素数さん:2009/06/06(土) 14:43:04
>>932 (上)

辺CD上に、CA '= QD なる点A 'をとる。
辺CDの延長線上に、B 'D = DQ なる点B 'をとる。
 A 'B '= AB かつ A 'B '//BA ゆえ、ABA 'B 'は平行4辺形。
AD上の点Pは、その内部にある。〔系〕により
  BP +PB '< BA + AB ',
∴  BP +PQ < BA + AQ,


〔補題〕 点Pが △XYZ の内部にあれば PY + PZ < XY + XZ,
 (略証)  YPの延長線とXZの交点をP 'とおく。三角不等式により PY + PZ < P 'Y + P 'Z < XY + XZ,

〔系〕 点Pが 平行四辺形ABA 'B 'の内部にあれば、BP + PB ' < BA + AB ',
937132人目の素数さん:2009/06/06(土) 15:04:12
〔問題〕
 △PQR が △XYZ に含まれるならば PQ+QR+RP ≦ XY+YZ+ZX,
938132人目の素数さん:2009/06/06(土) 23:09:59
>>937
(・∀・) ジメイ…のような気もするが・・・
939132人目の素数さん:2009/06/06(土) 23:19:48
>>938
 △PQR を相似拡大して △XYZ に内接させる。
(△XYZ を相似縮小して △PQR に外接させる)
かな?
940132人目の素数さん:2009/06/06(土) 23:40:00
凸多角形が閉曲線に囲まれた図形に含まれているとき
凸多角形の辺ABのAの側の延長と閉曲線の共有点の一つをC
Bの側の延長と閉曲線の共有点の一つをDとすると
閉曲線のCDの部分を線分CDに置き換えると閉曲線の長さは長くはならない。
この置き換えを凸多角形の全ての辺に対して順に行う。
最初の閉曲線の長さ≧最後の閉曲線の長さ=凸多角形の周の長さ。
941132人目の素数さん:2009/06/07(日) 01:01:29
x+y+z=1を満たす正の数x,y,zについて以下の不等式が成立することを示せ。
(1+x^2)/x(y+z)+(1+y^2)/y(z+x)+(1+z^2)/z(x+y)≧3
942132人目の素数さん:2009/06/07(日) 01:12:45
>>941
x,y,z≠0も追加で
943132人目の素数さん:2009/06/07(日) 13:00:55
>>942
>>941 で正の数x,y,z
っていってるから、
それはなくていいんでわ?
944132人目の素数さん:2009/06/07(日) 20:48:48
【トレビアン動画】朝日が台湾を「核保有国」に分類した件で紙面で「おことわり」掲載! 購読者が電話攻撃!

朝日新聞5月26日朝刊の6面に掲載された「核兵器をめぐる現状」という地図に「核保有5大国」にアメリカ、ロシア、イギリス、フランス、中国に赤色に染められているほか、
なんと台湾まで赤色になっているのだ。台湾は中国領土という見解なのか、6月5日に「おことわり」として紙面に掲載。その内容は以下のようなものだ。

おことわり
5月26日付「闇市場に関与指摘次々」の記事で核不拡散条約(NPT)で認められた核保有5大国などを地図に示しました。
その中で台湾については核保有国と同様の色分けでしたが、台湾は核兵器を保有していません。(原文ママ)

このことに疑問に思った購読者が朝日新聞に電話突撃攻撃。録音した内容を『YouTube』や『ニコニコ動画』にアップしている。
朝日新聞の対応も酷く受話器を放置してそのまま仕事をしたり「名前は名乗っていませんー」と名前も名乗らない対応。
電突者が「一流企業の広報とは思えない対応」というとその後は音信不通になりまたも受話器を放置される始末。

対応の状況をまとめると以下の様な感じだ。
・「おことわり」の意味を聞いても「読んで理解しろ」と言われる
・「おことわり」は訂正では無い(動画10:25〜)
・ガキレベルの対応(動画10:25〜)
・「ほかにも電話入っているので失礼します!」と強制的に切ろうとする(動画12:50)
・お名前は? 「名前は名乗っておりませんー」(動画15:27)

19分と長い戦いになるが、この動画を観れば大企業、朝日新聞社の対応の凄さがわかるぞ。
何回も電話を掛け直し、この対応に耐え抜いた忍耐力は凄いものである。
http://news.livedoor.com/article/detail/4190772/

★動画:朝日新聞に電凸 6月5日
http://www.nicovideo.jp/watch/sm7255107
http://www.youtube.com/watch?v=ThGlsJBtlM0 http://www.youtube.com/watch?v=WQxexTEOduQ
945132人目の素数さん:2009/06/08(月) 01:26:51
>>941
対称性から考えようとしたけどうまくいかない。。。

1/x + 1/y + 1/z -3 + 2/(1-x) + 2/(1-y) + 2/(1-z)

までは変形したんだけど
946132人目の素数さん:2009/06/08(月) 02:20:00
(1+x^2)/x(1−x)>1。

(1+x^2)/x(3−x)≧(1+x)/2。
947132人目の素数さん:2009/06/08(月) 21:11:03
>>945
 1/x, 1/(1-x) は下に凸から、あるいは相加・調和平均から
 1/x + 1/y + 1/z = (9/s) + (st-9u)/(su) ≧ 9/s,
 1/(1-x) + 1/(1-y) + 1/(1-z) ≧ 9/(3-s),
 (左辺) ≧ (9/s) -3 +18/(3-s),

>>946
 (1+x^2)/{x(1-x)} = 1/x -1 +2/(1-x)
は下に凸だから x=1/3 での接線の上側にある。
∴ (1+x^2)/{x(1-x)} = (13-9x)/2 + (1-3x)^2・{(2-x)/(2x(1-x))} ≧ (13-9x)/2,
 (左辺) ≧ 3(13-3s)/2,
948132人目の素数さん:2009/06/08(月) 22:05:51
∫[0→π]{(e^x)(sinx)^2}dx>8
であることを示せ.ただし,π=3.14…は円周率,e=2.71…は自然対数の底である.

エレガントな解を求む.
949132人目の素数さん:2009/06/08(月) 22:13:00
                       ,, -−──−-、、
                _,, -−─‐,r'",r''"´ ̄ ̄`"''−、` ー 、
             ,r'"        〈            ヽ   ヽ、
          /          ヽ、                !
           , ィ             \               ゙ 、
       ,r' /                !:.             く) ⌒ヽ、_              .厂 ̄i
        ノ ,イ               ゙、::.          r         ̄"'''ー--------一'"    ,'
     // ,'    ..::'"⌒ヽ、         !::..        ,'    , ヘ、__                 _ノ
   / /  !:.         ゙、          ト、::..      ノ:::..._厶_   _>゙ーーー‐‐‐‐‐‐‐一''"´
   ( (   |:::..        i::.           !::`'''ー-一<´厂     ̄
   ヽ、)  !::::...          !::::...       ゙、::..       ゙ 、
       人:::::::..        ゙、::::::::.......___,,ゝ、:::..      ヽ
      /::::...\::::::....       ヾ ̄ ̄    /::::..ヽ、:::..      \
     ,'::::::::::::....ヽ、:::::....       ゙、    ノ::::::::::::::...\::::...    ヽ
     〈:::::::::::::::::::::/`ヽ、:::::.....    〉   〈:::::::::::::::::::::...人:::::::::.....   〉
     `ー─一'"    `ー─一'゙      `ー−一'"   `ー─一'゙

950132人目の素数さん:2009/06/08(月) 22:59:25
>>949
誰がエレファントなAAを貼れと言った!
951132人目の素数さん:2009/06/09(火) 02:57:13
a[1],…,a[n]>0において
(a[1]/a[2])+(a[2]/a[3])+…+(a[n]/a[1])
≧{(a[1]+a[2])/(a[2]+a[3])}+{(a[2]+a[3])/(a[3]+a[4])}+…+{(a[n]+a[1])/(a[1]+a[2])}

f(x)は微分可能かつf'(x)が連続で,f(0)=f(π)=0のとき
∫[0,π](f(x))^2dx≦∫[0,π](f'(x))^2dx

a,b,c>0,ab+bc+ca=1において
{(1-a^2)/(1+a^2)}+{(1-b^2)/(1+b^2)}+{(1-c^2)/(1+c^2)}≦3/2
952132人目の素数さん:2009/06/09(火) 23:44:18
>>948
地道にやると・・・
∫ e^x・(sinx)^2 dx = ∫ e^x・{1-cos(2x)}/2 dx = e^x・{(1/2) - (1/10)cos(2x) -(1/5)sin(2x)},
(与式) = (2/5)(e^π - 1) だが、 この後が・・・・

>>951 (下)
 a^2 + b^2 + c^2 = ab+bc+ca + F_0 ≧ ab+bc+ca,
 (左辺) = 2/(1+a^2) + 2/(1+b^2) + 2/(1+c^2) -3
  ≦ 6/{1 + (a^2 + b^2 + c^2)/3} -3   (← 2/(1+x) は下に凸)
  ≦ 6/{1 + (ab+bc+ca)/3} -3,
953132人目の素数さん:2009/06/10(水) 16:04:31
>>951
a[i]/a[i+1]=x[i]、a[n]/a[1]=x[n]とおくと
(右辺)=(1+x[1])/(1+1/x[2])+(1+x[2])/(1+1/x[3])+……+(1+x[n])/(1+1/x[1])≦(1+x[1])/(1+1/x[1])+(1+x[2])/(1+1/x[2])+……+(1+x[n])/(1+1/x[n]) (チェビシェフ)
=x[1]+x[2]+……+x[n]=(左辺)

>>951
左辺を整理すると
1+4abc/(b+c)(c+a)(a+b)
よりabc/(b+c)(c+a)(a+b)≦1/8
をしめせばよいが
2√bc≦b+c,2√ca≦c+a,2√ab≦a+b
を辺々掛ければ明らか
(a=tanA,b=tanB,c=tanCとおいても解ける)
954132人目の素数さん:2009/06/11(木) 21:06:25
955132人目の素数さん:2009/06/11(木) 22:25:59
>>948
 e^2.302585・・・ = 10,
 π = 2.302585・・・ + 0.83900・・・> 2.302585・・・ + 5/6,
 e^(5/6) ≧ 1 + (5/6) + (1/2)(5/6)^2 > 1 + (5/6) + 1/3 > 2 + 1/6, 
 e^π > (e^2.302585)・e^(5/6) > 10・(2 + 1/6) = 21 + 2/3,
 (2/5)(e^π -1) > 8 + 4/15 > 8,

>>952
 無理筋ですた・・・・・orz

>>953 下 (続き)
 cot(A+B+C) = {1-(ab+bc+ca)}/(a+b+c-abc) =0,
より A+B+C = π/2,
 (左辺) = cos(2A) + cos(2B) + cos(2C) = 1 + 4sin(A)sin(B)sin(C)
 ≦ 1 + 4{[sin(A)+sin(B)+sin(C)]/3}^3    (相乗・相加平均)
 ≦ 1 + 4{sin((A+B+C)/3)}^3          (上に凸)
 = 1 + 4{sin(π/6)}^3
 = 1 + 4(1/2)^3
 = 3/2,
956132人目の素数さん:2009/06/11(木) 23:43:43
x,y,z>0,x^2<y<logzのとき
xy^4<z^2


a,b,c,d∈N,r=1-(a/b)-(c/d),a+c≦1982,r>0のとき
r>(1/1983)^3
957132人目の素数さん:2009/06/12(金) 04:01:37
a,b,c≧1のとき
{a^3-(1/a)^3}+{b^3-(1/b)^3}+{c^3-(1/c)^3}≧3{abc-(1/abc)}

a>b>c>0のとき
[1/{(a-b)(a-c)√a}]+[1/{(b-c)(b-a)√b}]+[1/{(c-a)(c-b)√c}]>0
958132人目の素数さん:2009/06/12(金) 11:57:35
a_k(k=1,2,3,..n)は正の数
Π[k=1,n]a_k^a_k≧(Π[k=1,n]a_k)^(Σa_k/n)を示せ
959132人目の素数さん:2009/06/13(土) 00:21:06
>>957
 a+b+c=s, ab+bc+ca=t, abc=u とおくと、
 a^3 + b^3 +c^3 -3abc = (a+b+c){(a-b)^2 +(b-c)^2 +(c-a)^2}/2
 ≧ 3{(a-b)^2 +(b-c)^2 +(c-a)^2}/2          (← a,b,c≧1)
 ≧ (1/a + 1/b + 1/c){[(a-b)/ab]^2 + [(b-c)/bc]^2 + [(c-a)/ca]^2}/2  (← 1≧1/a,1/b,1/c)
 ≧ 1/(a^3) + 1/(b^3) + 1/(c^3) - 3/(abc),

>>957
 (a-c)/{(b-c)(b-a)} = -1/(a-b) - 1/(b-c) より
 (左辺)*(a-c) = {1/(a-b)}(1/√a - 1/√b) + {1/(b-c)}(1/√c - 1/√b)
     = - 1/(a√b + b√a) + 1/(c√b + b√c) > 0,    (← a>c)

>>958
 対数を考えれ。チェビシェフより
 Σ[k=1,n] (a_k)log(a_k) ≧ {Σ[i=1,n] log(a_i)}(Σ[j=1,n] a_j)/n,
960959:2009/06/13(土) 00:42:37
>>957
 a^3 + b^3 +c^3 -3abc = (a+b+c){(a-b)^2 +(b-c)^2 +(c-a)^2}/2
 ≧ 3{(a-b)^2 +(b-c)^2 +(c-a)^2}/2          (← a,b,c≧1)
 ≧ (1/a + 1/b + 1/c){[(a-b)/ab]^2 + [(b-c)/bc]^2 + [(c-a)/ca]^2}/2  (← 1≧1/a,1/b,1/c)
 = 1/(a^3) + 1/(b^3) + 1/(c^3) - 3/(abc),

>>957
 √a = A, √b = B, √c = C とおくと、
 (左辺)*(a-c) = (A-C)(A+B+C)/{(A+B)(B+C)ABC},
 (左辺) = (A+B+C)/{(A+B)(B+C)(C+A)ABC} >0,
961132人目の素数さん:2009/06/13(土) 02:28:42
f(a)=f(b)=0
f’’(x)≧0 (a≦x≦b)
なら,なぜ
f(x)≦0 (a≦x≦b)なんですか?
962132人目の素数さん:2009/06/13(土) 02:50:39
不等式ヲタ=関数方程式ヲタ=整数ヲタ=CのΣヲタ
963132人目の素数さん:2009/06/13(土) 03:45:15
>>961
ほとんど明らか
964132人目の素数さん:2009/06/13(土) 04:49:24
π>3.05であることを示せ。
965132人目の素数さん:2009/06/13(土) 05:37:48
>>962
ほとんど明らか
966132人目の素数さん:2009/06/13(土) 09:33:18
>>961
ロルの定理から、
 f '(ξ) = 0,
なるξが (a,b) にある。
 a<x≦ξ では f '(x) = f '(ξ) -∫[x,ξ] f "(x)dx ≦ f'(ξ) = 0,
      f(x) = f(a) + ∫[a,x] f '(y)dy ≦ f(a) = 0,
 ξ≦x<b では f '(x) = f '(ξ) +∫[ξ,x] f "(x)dx ≧ f'(ξ) = 0,
      f(x) = f(b) - ∫[x,b] f '(y)dy ≦ f(b) = 0,
967132人目の素数さん:2009/06/13(土) 13:18:11
これって入試にそのまま使っていいのか悩んだ記憶がある
968132人目の素数さん:2009/06/13(土) 17:06:22
最近じゃヘロンの公式も入試で使っていいのかダメなのか議論されている
969132人目の素数さん:2009/06/13(土) 17:47:53
使っていいに決まってんじゃん
970132人目の素数さん:2009/06/13(土) 19:28:20
それが最近はダメだという意見もあるそうだ
971132人目の素数さん:2009/06/13(土) 20:55:32
ロルの定理使ったらダメなら平均値の定理も使ったらダメになるwww
972132人目の素数さん:2009/06/13(土) 23:09:24
>>970
どこのヌケ作が言っているんだ?ボケ!
973132人目の素数さん:2009/06/13(土) 23:33:34
プロレスの三沢光晴さん、リングで頭強打し死亡

 13日午後8時45分頃、広島市中区の広島県立総合体育館グリーンアリーナで、
プロレスリング・ノアの試合中、社長でプロレスラーの三沢光晴さん(46)が
相手選手にバックドロップをかけられ、頭部を強打した。
 三沢さんは救急車で市内の病院に運ばれたが、間もなく死亡した。
 三沢さんは2代目タイガーマスクとして人気を集め、
全日本プロレスやプロレスリング・ノアで中心選手として活躍してきた。

(2009年6月13日23時24分 読売新聞)

http://www.yomiuri.co.jp/sports/news/20090613-OYT1T01053.htm?from=top
974132人目の素数さん:2009/06/13(土) 23:50:11
不等式で頭を挟み撃ちにされたわけだな
975132人目の素数さん:2009/06/14(日) 16:04:44
976132人目の素数さん:2009/06/14(日) 23:06:06
>>974
かわいいオニャノコに、挟み撃ちにされたいです
977132人目の素数さん:2009/06/15(月) 11:04:47
>>972
荒らすなヌケ作ボケ!
978132人目の素数さん:2009/06/15(月) 19:10:00
979132人目の素数さん:2009/06/15(月) 23:24:53
nは自然数とする
(sinx)^n+(cosx)^n
の最大値、最小値を求めよ


Kを非負の定数とする
区間[t1,t2]で定義された負でない連続関数f(t),g(t)が
f(t)≦K+∫[t1→t]g(s)f(s)ds (t1≦t≦t2)
を満たすならば
f(t)≦Kexp(∫[t1→t]g(s)ds) (t1≦t≦t2)
が成り立つことを示せ
980132人目の素数さん:2009/06/16(火) 05:00:00
二年三十四日。
981132人目の素数さん:2009/06/16(火) 14:43:59
カーッ(゚Д゚≡゚д゚)、ペッ >>977,980
982132人目の素数さん:2009/06/16(火) 16:19:53
A,B,C>0,A+B+C=πのとき
sinA+sinB+sinC≦4sinAsinBsinC
を示せ
983132人目の素数さん:2009/06/16(火) 18:56:12
>>981
荒らすなヌケ作ボケ!
984132人目の素数さん:2009/06/16(火) 22:57:58
0 < a, b, c, d < 1 のとき、以下の不等式を示せ!

(1) 1+ab > a+b
(2) 1+abc > ab+bc+ca
(3) 1+abcd > abc+abd+acd+bcd
(4) 一般化せよ

お久しぶりです。( ゚∀゚) テヘッ!
985132人目の素数さん:2009/06/16(火) 23:00:45
>>982
改造せずにはいられない… (*゚∀゚)=3 ハァハァ

三角形ABCに対して、
0 < sin2A + sin2B + sin2C < sinA + sinB + sinC ≦ (3√3)/2
986132人目の素数さん:2009/06/17(水) 05:00:00
二年三十五日。
987132人目の素数さん:2009/06/17(水) 21:37:10
>>982

相乗・相加平均 と 上に凸 より
 {sin(A/2)sin(B/2)sin(C/2)}^(1/3) ≦ {sin(A/2) + sin(B/2) + sin(C/2)}/3 ≦ sin((A+B+C)/6) = sin(π/6) = 1/2,
∴ 1 ≧ 8sin(A/2)sin(B/2)sin(C/2),
∴ sin(A) + sin(B) + sin(C) = 4cos(A/2)cos(B/2)cos(C/2) ≧ 4sin(A)sin(B)sin(C),
 (不等号が逆向き・・・・)

>>985
(左側&中央)
 sin(2A) + sin(2B) + sin(2C) = 4sin(A)sin(B)sin(C),

(右側) sin は上に凸だから
 sin(A) + sin(B) + sin(C) ≦ 3sin((A+B+C)/3) = 3sin(π/3) = (3√3)/2,
988132人目の素数さん:2009/06/18(木) 00:49:47
>>984
成り立たない
989132人目の素数さん:2009/06/18(木) 05:00:00
二年三十六日。
990132人目の素数さん:2009/06/18(木) 16:37:06
>>984
nは正整数
0<a[i]<1で
n-1+Π[k=1_n]a[k]≧Σ[k=1_n]a[k]
と言いたかったのかな
991132人目の素数さん:2009/06/18(木) 18:45:57
>>990
俺はこう思った
1+ab>2(a+b)/2
1+abc>2(ab+bc+ca)/3
1+abcd>2(abc+abd+acd+bcd)/4
1+abcde>2(abcd+abce+abde+acde+bcde)/5
992132人目の素数さん:2009/06/18(木) 20:46:56
新数学スタンダード演習にあった問題です
1. x<y<zのときxyy-xxy+yzz-yyz+zxx-zzx>0を示せ
2. 1<a<b<cのとき log[a](c/b)+log[b](a/c)+logc[b/a]>0を示せ.
解答では1.でxについて整理、2.では1.を利用としています.
ここの住人の方々にエレガントで驚愕できる美しい解答をお願いしたいです.
993132人目の素数さん:2009/06/18(木) 21:11:29
>>992
とりあえず、1. の方だけ。
A=y-x B=z-y とおいて式を整理すると、
xy^2 - x^2y + yz^2 - y^2z + zx^2 - z^2x
= xyA + yzB - zx(A+B) = x(y-z)A + z(y-x)B = (z-x)AB > 0
994132人目の素数さん:2009/06/18(木) 22:57:05
>>984
すまぬ、こう書きたかった…

0 < a, b, c, d < 1 のとき、以下の不等式を示せ!

(1) 1+ab > a+b
(2) 1+2abc > ab+bc+ca
(3) 1+3abcd > abc+abd+acd+bcd
995132人目の素数さん:2009/06/18(木) 22:57:20
ダメだ,対称性がありすぎて普通にしか解けん...

1. x<y<zより 0<y-x, z-y, z-x なので乗じて (y-x)(z-y)(z-x)>0.
これを展開すれば与式となる.

2. 1<a<b<c の自然対数(常用対数でも可)をとって 0<log a<log b<log c
それぞれx,y,zと見立てて 第1式に代入し, logA-logB=log(A/B) という規則を用いたのち, (log a)(log b)(log c)>0 で両辺割れば
{log(c/b)}/log(a)+{log(a/c)}/log(b){log(b/a)}/log(c)>0
最後に logB/logA=log[A](B) という規則を用いれば与式となる.
996132人目の素数さん:2009/06/19(金) 00:40:43
>>994
(1) 1+ab > a+b:
(1-a)(1-b)>0を展開.

(2) 1+2abc > ab+bc+ca
(1)式両辺にcを乗じると c+abc > ca+bc=(与右辺)-ab
両辺に (1-c+abc) を足すと (与左辺)>(与右辺)-ab+(1-c+abc)
最後の部分 abc+1-ab-c は(1)を用いると0より大きいので,結局上式の右辺は与右辺より大きくなり(2)が成立.
(∵ 0<a,b,c<1より0<ab,c<1)

一般化すると
1+(n-1)(a1a2…an)>Σ[j=1,n]{(a1a2…an)/aj} (0<ai<1) ――― (*)
右辺は a1〜an の積が1項欠けたモノの和.

帰納法で示す.n=2 の場合は(1)で示した.
今 n で成り立つとする.
(*)の両辺に a(n+1) を乗じると
a(n+1)+(n-1)(a1a2…an・a(n+1)) > a(n+1) {Σ[j=1,n]{(a1a2…an)/aj}} = Σ[j=1,n+1]{(a1a2…an・a(n+1))/aj}-(a1a2…an)
両辺に 1-a(n+1)+(a1a2…an・a(n+1) を足すと
(左辺)=1+n(a1a2…an・a(n+1))
(右辺)=Σ[j=1,n+1]{(a1a2…an・a(n+1))/aj}+[1+(a1a2…an・a(n+1)-a(n+1)-(a1a2…an)]
0<ai<1 より 0<a(n+1),(a1a2…an)<1 がいえて,右辺第2項に(1)を用いれば
(右辺)>Σ[j=1,n+1]{(a1a2…an・a(n+1))/aj}
以上合わせて 1+n(a1a2…an・a(n+1))>Σ[j=1,n+1]{(a1a2…an・a(n+1))/aj}
となり n+1 でも成立

【コメント】
(2)(3)ともにaについて整理してて,前の結果を変形してやれば (n-1)(a1a2…an) が出ることに気付いた.
(a1a2…an) 足りないけど気にせずやってたらいい感じに.
997132人目の素数さん:2009/06/19(金) 05:00:00
二年三十七日。
998132人目の素数さん:2009/06/19(金) 05:01:00
二年三十七日一分。
999132人目の素数さん:2009/06/19(金) 05:02:00
二年三十七日二分。
1000132人目の素数さん:2009/06/19(金) 05:03:00
二年三十七日三分。
10011001
このスレッドは1000を超えました。
もう書けないので、新しいスレッドを立ててくださいです。。。